Download as pdf or txt
Download as pdf or txt
You are on page 1of 163

See discussions, stats, and author profiles for this publication at: https://www.researchgate.

net/publication/274069842

Inequalities

Book · January 2013


DOI: 10.13140/RG.2.1.3735.2485

CITATION READS

1 267

1 author:

Andrzej Nowicki
Nicolaus Copernicus University
110 PUBLICATIONS 826 CITATIONS

SEE PROFILE

All content following this page was uploaded by Andrzej Nowicki on 26 March 2015.

The user has requested enhancement of the downloaded file.


Podróże po Imperium Liczb

Część 13

Nierówności
Andrzej Nowicki

Wydanie drugie, uzupełnione i rozszerzone

Olsztyn, Toruń, 2013


NRW - 41(1350) - 04.05.2013
Spis treści
Wstęp 1

1 Nierówności i funkcje wypukłe 5


1.1 Funkcje wypukłe i ich własności . . . . . . . . . . . . . . . . . . . . . . . . . . . . . . 5
1.2 Funkcje J-wypukłe . . . . . . . . . . . . . . . . . . . . . . . . . . . . . . . . . . . . . . 10
1.3 Funkcje wypukłe i ciągłość . . . . . . . . . . . . . . . . . . . . . . . . . . . . . . . . . 11
1.4 Funkcje wypukłe i druga pochodna . . . . . . . . . . . . . . . . . . . . . . . . . . . . 11
1.5 Nierówność Karamaty . . . . . . . . . . . . . . . . . . . . . . . . . . . . . . . . . . . . 13

2 Klasyczne nierówności 17
2.1 Średnia arytmetyczna i średnia geometryczna . . . . . . . . . . . . . . . . . . . . . . . 17
2.2 Średnia harmoniczna i średnia kwadratowa . . . . . . . . . . . . . . . . . . . . . . . . 22
2.3 Średnie złożone . . . . . . . . . . . . . . . . . . . . . . . . . . . . . . . . . . . . . . . 23
2.4 Średnie potęgowe . . . . . . . . . . . . . . . . . . . . . . . . . . . . . . . . . . . . . . 23
2.5 Nierówność Bernoulliego . . . . . . . . . . . . . . . . . . . . . . . . . . . . . . . . . . 25
2.6 Nierówność Cauchy’ego . . . . . . . . . . . . . . . . . . . . . . . . . . . . . . . . . . . 26
2.7 Różne klasyczne nierówności . . . . . . . . . . . . . . . . . . . . . . . . . . . . . . . . 29

3 Twierdzenie Muirheada 33
3.1 Podziały . . . . . . . . . . . . . . . . . . . . . . . . . . . . . . . . . . . . . . . . . . . 33
3.2 Wielomian symetryczny stowarzyszony z podziałem . . . . . . . . . . . . . . . . . . . 36
3.3 Twierdzenie Muirheada i jego dowód . . . . . . . . . . . . . . . . . . . . . . . . . . . 38
3.4 Twierdzenie Muirheada dla podziałów liczb całkowitych . . . . . . . . . . . . . . . . . 41
3.5 Twierdzenie Muirheada dla podziałów liczb wymiernych . . . . . . . . . . . . . . . . . 42
3.6 Nierówności cykliczne . . . . . . . . . . . . . . . . . . . . . . . . . . . . . . . . . . . . 43

4 Jednorodne nierówności wielomianowe 47


4.1 Jednorodne nierówności wielomianowe n zmiennych . . . . . . . . . . . . . . . . . . . 47
4.2 Problem Janousa . . . . . . . . . . . . . . . . . . . . . . . . . . . . . . . . . . . . . . 49
4.3 Jednorodne nierówności wielomianowe dwóch zmiennych . . . . . . . . . . . . . . . . 51
4.4 Jednorodne nierówności wielomianowe trzech zmiennych stopnia 2 . . . . . . . . . . . 52
4.5 Jednorodne nierówności wielomianowe trzech zmiennych stopnia 3 . . . . . . . . . . . 53
4.6 Jednorodne nierówności wielomianowe trzech zmiennych stopnia 4 . . . . . . . . . . . 55
4.7 Jednorodne nierówności wielomianowe trzech zmiennych stopnia 5 . . . . . . . . . . . 56
4.8 Jednorodne nierówności wielomianowe trzech zmiennych stopnia 6 . . . . . . . . . . . 56
4.9 Jednorodne nierówności wielomianowe trzech zmiennych stopnia > 6 . . . . . . . . . 57
4.10 Jednorodne nierówności wielomianowe trzech zmiennych stopnia n . . . . . . . . . . . 57
4.11 Jednorodne nierówności wielomianowe czterech zmiennych . . . . . . . . . . . . . . . 58

5 Niejednorodne nierówności wielomianowe 59


5.1 Nierówności wielomianowe n zmiennych . . . . . . . . . . . . . . . . . . . . . . . . . . 59
5.2 Nierówności wielomianowe jednej zmiennej . . . . . . . . . . . . . . . . . . . . . . . . 60
5.3 Nierówności wielomianowe dwóch zmiennych . . . . . . . . . . . . . . . . . . . . . . . 60
5.4 Nierówności wielomianowe trzech zmiennych . . . . . . . . . . . . . . . . . . . . . . . 61
5.5 Nierówności wielomianowe dla boków trójkąta . . . . . . . . . . . . . . . . . . . . . . 64
5.6 Nierówności wielomianowe czterech zmiennych . . . . . . . . . . . . . . . . . . . . . . 65
5.7 Nierówności z liczbami an + bn . . . . . . . . . . . . . . . . . . . . . . . . . . . . . . . 66
5.8 Nierówności wielomianowe dla liczb całkowitych . . . . . . . . . . . . . . . . . . . . . 67
5.9 Różne nierówności wielomianowe . . . . . . . . . . . . . . . . . . . . . . . . . . . . . . 71

i
6 Jednorodne nierówności wymierne 73
6.1 Jednorodne nierówności wymierne n zmiennych . . . . . . . . . . . . . . . . . . . . . . 73
6.2 Nierówność Nesbitta i jej uogólnienia . . . . . . . . . . . . . . . . . . . . . . . . . . . 76
6.3 Jednorodne nierówności wymierne dwóch zmiennych . . . . . . . . . . . . . . . . . . . 79
6.4 Jednorodne nierówności wymierne trzech zmiennych . . . . . . . . . . . . . . . . . . . 79
6.5 Jednorodne nierówności wymierne czterech zmiennych . . . . . . . . . . . . . . . . . . 86

7 Różne nierówności wymierne 89


7.1 Nierówności wymierne ze stałym iloczynem . . . . . . . . . . . . . . . . . . . . . . . . 89
7.2 Nierówności wymierne n zmiennych . . . . . . . . . . . . . . . . . . . . . . . . . . . . 93
7.3 Nierówności wymierne jednej zmiennej . . . . . . . . . . . . . . . . . . . . . . . . . . 95
7.4 Nierówności wymierne dwóch zmiennych . . . . . . . . . . . . . . . . . . . . . . . . . 95
7.5 Nierówności wymierne trzech zmiennych . . . . . . . . . . . . . . . . . . . . . . . . . 97
7.6 Nierówności wymierne czterech zmiennych . . . . . . . . . . . . . . . . . . . . . . . . 102
7.7 Nierówności wymierne dla liczb całkowitych . . . . . . . . . . . . . . . . . . . . . . . 103

8 Nierówności z pierwiastkami 109


8.1 Nierówności z pierwiastkami n zmiennych . . . . . . . . . . . . . . . . . . . . . . . . . 109
8.2 Nierówności z pierwiastkami dwóch zmiennych . . . . . . . . . . . . . . . . . . . . . . 110
8.3 Jednorodne nierówności z pierwiastkami trzech zmiennych . . . . . . . . . . . . . . . 110
8.4 Niejednorodne nierówności z pierwiastkami trzech zmiennych . . . . . . . . . . . . . . 115
8.5 Nierówności z pierwiastkami czterech zmiennych . . . . . . . . . . . . . . . . . . . . . 118
8.6 Nierówności z pierwiastkami i liczbami naturalnymi . . . . . . . . . . . . . . . . . . . 118
8.7 Różne nierówności z pierwiastkami . . . . . . . . . . . . . . . . . . . . . . . . . . . . . 120

9 Różne nierówności 121


9.1 Pochodna i nierówności . . . . . . . . . . . . . . . . . . . . . . . . . . . . . . . . . . . 121
9.2 Nierówności z max i min . . . . . . . . . . . . . . . . . . . . . . . . . . . . . . . . . . 122
9.3 Nierówności z bezwzględną wartością . . . . . . . . . . . . . . . . . . . . . . . . . . . 124
9.4 Nierówności z częścią całkowitą . . . . . . . . . . . . . . . . . . . . . . . . . . . . . . 124
9.5 Nierówności z częścią ułamkową . . . . . . . . . . . . . . . . . . . . . . . . . . . . . . 126
9.6 Nierówności potęgowe . . . . . . . . . . . . . . . . . . . . . . . . . . . . . . . . . . . . 127
9.7 Nierówności z logarytmami . . . . . . . . . . . . . . . . . . . . . . . . . . . . . . . . . 128
9.8 Nierówności z sumą lub iloczynem cyfr . . . . . . . . . . . . . . . . . . . . . . . . . . 129
9.9 Nierówności z silniami . . . . . . . . . . . . . . . . . . . . . . . . . . . . . . . . . . . . 130
9.10 Nierówności z symbolami Newtona . . . . . . . . . . . . . . . . . . . . . . . . . . . . . 132
9.11 Nierówności z nwd i nww . . . . . . . . . . . . . . . . . . . . . . . . . . . . . . . . . . 133
9.12 Nierówności z liczbami pierwszymi . . . . . . . . . . . . . . . . . . . . . . . . . . . . . 136
9.13 Nierówności z klasycznymi funkcjami arytmetycznymi . . . . . . . . . . . . . . . . . . 138
9.14 Nierówności i ciągi rekurencyjne . . . . . . . . . . . . . . . . . . . . . . . . . . . . . . 143
9.15 Nierówności z funkcjami trygonometrycznymi . . . . . . . . . . . . . . . . . . . . . . . 144
9.16 Inne nierówności . . . . . . . . . . . . . . . . . . . . . . . . . . . . . . . . . . . . . . . 145

Spis cytowanej literatury 146

Skorowidz nazwisk 153

Skorowidz 156

ii
Wstęp

Głównym tematem prezentowanej serii książek są liczby i ich przeróżne własności. Autor
od najmłodszych lat zbierał wszelkie fakty i ciekawostki dotyczące najpierw liczb całkowitych
i wielomianów o współczynnikach całkowitych, a następnie dotyczące również liczb wymier-
nych, rzeczywistych, zespolonych oraz wielomianów nad tymi zbiorami liczbowymi. Nazbie-
rało się sporo interesującego materiału, którego wybrane fragmenty będą tu przedstawione.
Materiał pochodzi z wielu różnych źródeł. Są tu zadania i problemy, które znajdziemy w
popularnych czasopismach matematycznych. Wśród tych czasopism jest wychodzące od 1894
roku (przeważnie 10 numerów w roku) The American Mathematical Monthly. Są wśród tych
czasopism również: angielskie czasopismo Mathematical Gazette, , kanadyjskie Crux Mathe-
maticorum, rosyjskie Kwant, chińskie Mathematical Excalibur, itp. Godnymi uwagi są również
polskie czasopisma popularno-naukowe: Delta, czasopismo dla nauczycieli Matematyka oraz
inne.
Istotną rolę w prezentowanym materiale odegrały zadania z olimpiad i konkursów mate-
matycznych całego świata. Każdego roku pojawiają się opracowania, książki oraz artykuły
dotyczące zadań z różnych zawodów matematycznych. Wspomnijmy tylko o prestiżowych
seriach książek z zawodów International Mathematical Olympiad (IMO) oraz Putnam Mathe-
matical Competition. Sporo oryginalnych zadań znajduje się w opracowaniach dotyczących
olimpiad matematycznych w Rosji lub w państwach byłego Związku Radzieckiego. Polska
również ma wartościowe serie tego rodzaju książek.
Zebrany materiał pochodzi również z różnych starych oraz współczesnych podręczników
i książek z teorii liczb. Wykorzystano liczne książki popularno-naukowe oraz prace naukowe
publikowane w różnych czasopismach specjalistycznych. Są tu też pewne teksty pochodzące
z internetu.
Większość prezentowanych faktów ma swoje odnośniki do odpowiedniej literatury. Odnoś-
niki te wskazują tylko wybrane miejsca, w których można znaleźć albo informacje o danym
zagadnieniu, albo rozwiązanie zadania, albo odpowiedni dowód. Bardzo często omawiany
temat jest powtarzany w różnych pozycjach literatury i często trudno jest wskazać oryginalne
źródła. Jeśli przy danym zagadnieniu nie ma żadnego odnośnika do literatury, to oznacza to,
że albo omawiany fakt jest oczywisty i powszechnie znany, albo jest to własny wymysł autora.
Elementarna teoria liczb jest wspaniałym źródłem tematów zachęcających do pisania
własnych programów komputerowych, dzięki którym można dokładniej poznać badane prob-
lemy. Można wykorzystać znane komputerowe pakiety matematyczne: MuPad, Mathematica,
CoCoA, Derive, Maple i inne. W prezentowanej serii książek znajdziemy sporo wyników i
tabel uzyskanych głównie dzięki pakietowi Maple.

We wszystkich książkach z serii ”Podróże po Imperium Liczb” stosować będziemy jedno-


lite oznaczenia. Zakładamy, że zero nie jest liczbą naturalną i zbiór {1, 2, 3, . . . }, wszystkich
liczb naturalnych, oznaczamy przez N. Przez N0 oznaczamy zbiór wszystkich nieujemnych
liczb całkowitych, czyli zbiór N wzbogacony o zero. Zbiory liczb całkowitych, wymiernych,
rzeczywistych i zespolonych oznaczamy odpowiednio przez Z, Q, R oraz C. Zbiór wszystkich
liczb pierwszych oznaczamy przez P.

1
Największy wspólny dzielnik liczb całkowitych a1 , . . . , an oznaczamy przez nwd(a1 , . . . , an )
lub, w przypadkach gdy to nie prowadzi do nieporozumienia, przez (a1 , . . . , an ). Natomiast
najmniejszą wspólną wielokrotność tych liczb oznaczamy przez nww(a1 , . . . , an ) lub [a1 , . . . , an ].
Zapis a | b oznacza, że liczba a dzieli liczbę b. Piszemy a ∤ b w przypadku, gdy a nie dzieli
b. Część całkowitą liczby rzeczywistej x oznaczamy przez [x]. Jeśli m jest liczbą natural-
ną, to ϕ(m) jest liczbą wszystkich liczb naturalnych mniejszych lub równych m i względnie
pierwszych z liczbą m. Liczbę elementów skończonego zbioru A oznaczamy przez |A|.
Pewne zamieszczone tutaj fakty przedstawione są wraz z ich dowodami. Początek dowodu
oznaczono przez D.. Pojawiają się również symbole R., U., W. oraz O. informujące odpo-
wiednio o początku rozwiązania, uwagi, wskazówki i odpowiedzi. Wszystkie tego rodzaju
teksty zakończone są symbolem ⊠. Skrót ”Odp.” również oznacza odpowiedź.
Spis cytowanej literatury znajduje się na końcu tej książki (przed skorowidzami). Liczby
pomiędzy nawiasami h oraz i, występujące w tym spisie, oznaczają strony, na których dana
pozycja jest cytowana. W pewnych podrozdziałach podano również literaturę dodatkową lub
uzupełniającą. Informuje o tym symbol ⋆.

Seria ”Podróże po Imperium Liczb” składa się z piętnastu nastpujących książek.


01. Liczby wymierne;
02. Cyfry liczb naturalnych;
03. Liczby kwadratowe;
04. Liczby pierwsze;
05. Funkcje arytmetyczne;
06. Podzielność w zbiorze liczb całkowitych;
07. Ciągi rekurencyjne;
08. Liczby Mersenne’a, Fermata i inne liczby;
09. Sześciany, bikwadraty i wyższe potęgi;
10. Liczby i funkcje rzeczywiste;
11. Silnie i symbole Newtona;
12. Wielomiany;
13. Nierówności;
14. Równanie Pella;
15. Liczby, funkcje, zbiory, geometria.
Wszystkie książki z serii ”Podróże po Imperium Liczb” napisano w edytorze LATEX.
Spisy treści tych książek oraz pewne wybrane rozdziały moża znaleźć na internetowej stronie
autora: http://www.mat.uni.torun.pl/~anow.
Wszystkie książki z serii ”Podróże po Imperium Liczb” zostały wydane przez Wydaw-
nictwo Naukowe Olsztyńskiej Wyższej Szkoły Informatyki i Zarządzania im. prof. Tadeusza
Kotarbińskiego. Pierwsze wydania tych książek pojawiły się w latach 2008 − 2011. Autor
otrzymał sporo interesujących listów z uwagami i komentarzami dotyczącymi omawianych
zagadnień. Były też listy, w których wytknięto szereg pomyłek, błędów i niedokładności.
Autorom tych wszystkich listów należą się szczere i serdeczne podziękowania.
Teraz, w tym drugim wydaniu książek serii ”Podróże po Imperium Liczb”, przesłane
uwagi zostały uwzględnione. Naprawiono błędy, dołączono pewne dowody oraz podano nową
aktualną literaturę. Wydanie to jest rozszerzone, uzupełnione i wzbogacone o pewne nowe
rozdziały lub podrozdziały.

2
ooooo

W trzynastej książce z serii ”Podróże po Imperium Liczb” zajmujemy się różnymi nie-
równościami liczbowymi.
Książka składa się z dziewięciu rozdziałów. W rozdziale pierwszym podajemy, wraz z
istotnymi dowodami, podstawowe informacje o funkcjach wypukłych. Informacje te odgry-
wać będą ważną rolę w dowodach wielu nierówności podanych we wszystkich następnych
rozdziałach. W szczególności zajmujemy się nierównościami Karamaty.
W rozdziale drugim mowa jest o powszechnie znanych nierównościach klasycznych. Mówi-
my tu o nierównościach pomiędzy wszelkiego rodzaju średnimi oraz mówimy o nierównościach:
Bernoulliego, Cauchy’ego, Höldera, Minkowskiego, Czebyszewa i różnych innych. Do klasycz-
nych nierówności zalicza się również pewne nierówności wynikające z twierdzenia Muirheada
z 1903 roku. Tym zagadnieniem zajmujemy się szczegółowo w rozdziale trzecim.
Wszystkie następne rozdziały, od czwartego do ostatniego włącznie, zawierają liczne serie
przykładów znanych i mniej znanych nierówności. Przykłady te posortowano i podzielono
na pewne grupy. Oddzielne grupy stanowią nierówności wielomianowe i nierówności wymier-
ne. W tych grupach wyróżnia się jeszcze nierówności jednorodne, posegregowane względem
stopnia jednorodności. Jest też grupa dotycząca nierówności z pierwiastkami, która zajmu-
je cały rozdział ósmy. W ostatnim rozdziale są, między innymi, nierówności potęgowe oraz
nierówności z bezwzględnymi wartościami i logarytmami.
Pewnymi nierównościami zajmowaliśmy się już w innych książkach z serii ”Podróże po
Imperium Liczb”. W książce [N-2] jest podrozdział o nierównościach z sumami cyfr liczb
naturalnych. W książce [N-5] są trzy podrozdziały poświęcone nierównościom dla podsta-
wowych funkcji arytmetycznych. Podrozdział o nierównościach dla największych wspólnych
dzielników i najmniejszych wspólnych wielokrotności znajdziemy w książce [N-6]. Nierówności
są również w [N-7] (patrz, na przykład, podrozdział o nierównościach dla liczb Fibonaccie-
go). Sporo różnych nierówności znajdziemy w książce [N10]. Są tam nierówności z częściami
całkowitymi i ułamkowymi oraz różnego rodzaju nierówności trygonometryczne. W książce
[N11] są oddzielne podrozdziały o nierównościach z silniami i symbolami Newtona. Wymie-
niliśmy tylko kilka książek. We wszystkich pozostałych książkach prezentowanej serii również
pojawiają się nierówności.

3
⊡⊡⊡⊡⊡⊡⊡⊡⊡⊡⊡⊡⊡⊡⊡⊡⊡⊡⊡⊡⊡⊡⊡⊡⊡⊡⊡⊡⊡⊡⊡⊡⊡⊡⊡⊡⊡⊡⊡⊡⊡⊡⊡⊡⊡⊡⊡⊡⊡⊡⊡⊡
1 Nierówności i funkcje wypukłe
⊡⊡⊡⊡⊡⊡⊡⊡⊡⊡⊡⊡⊡⊡⊡⊡⊡⊡⊡⊡⊡⊡⊡⊡⊡⊡⊡⊡⊡⊡⊡⊡⊡⊡⊡⊡⊡⊡⊡⊡⊡⊡⊡⊡⊡⊡⊡⊡⊡⊡⊡⊡

oooooooooooooooooooooooooooooooooooooooooooooooooooooooooooooooooooooo
1.1 Funkcje wypukłe i ich własności
oooooooooooooooooooooooooooooooooooooooooooooooooooooooooooooooooooooo
We wszystkich książkach serii ”Podróże po Imperium Liczb” przez R oznaczamy zbiór
wszystkich liczb rzeczywistych. Pewne szeczególne podzbiory tego zbioru nazywają się prze-
działami. Niech a, b ∈ R, a < b. Zbiory
[a, b] = {x ∈ R; a 6 x 6 b}, (a, b) = {x ∈ R; a < x < b},
to odpowiednio przedział domknięty i przedział otwarty. Są też przedziały półotwarte (lub
półdomknięte): (a, b] = {x ∈ R; a < x 6 b}, [a, b) = {x ∈ R; a 6 x < b}. Mamy również
przedziały nieskończone:
[a, ∞) = {x ∈ R; a 6 x}, (−∞, a] = {x ∈ R; x 6 a},
(a, ∞) = {x ∈ R; a < x}, (−∞, a) = {x ∈ R; x < a}
oraz (−∞, ∞) = R. Przedział [0∞), czyli zbiór wszystkich nieujemnych liczb rzeczywistych,
oznaczać będziemy przez R+ . Wymienione podzbiory zbioru liczb rzeczywistych mają pewną
wspólną cechę pozwalającą przyjąć następującą definicję. Przedziałem liczb rzeczywistych
nazywamy każdy niepusty zbiór I ⊆ R taki, że jeśli a, b ∈ I, to [a, b] ⊆ I.
Niech a < b. Liczby należące do przedziału [a, b] posiadają następującą szczególną wła-
sność.
1.1.1. Jeśli liczba x należy do przedziału [a, b], to istnieją jednoznacznie wyznaczone liczby
rzeczywiste α > 0 i β > 0 takie, że α + β = 1 oraz x = αa + βb. Liczbami tymi są:
b−x x−a
α= , β= .
b−a b−a
b−x x−a b−x+x−a b−a
D. Niech α = ,β= . Wtedy α > 0, β > 0, α + β = = = 1 oraz
b−a b−a b−a b−a
1   1  
αa + βb = (b − x)a + (x − a)b = x(b − a) = x.
b−a b−a
Załóżmy teraz, że x = αa + βb, gdzie α > 0, β > 0, α + β = 1. Wtedy
b − x = b − (αa + βb) = (1 − β)b − αa = αb − αa = α(b − a)
b−x
i stąd α = b−a . Ponadto,
x − a = αa + βb − a = βb − (1 − α)a = βb − βa = β(b − a)
x−a
i stąd β = . Liczby α i β są więc wyznaczone jednoznacznie. ⊠
b−a
Z tej uwagi wynikają następujące równości.
n o n o
1.1.2. [a, b] = αa + βb; α > 0, β > 0, α + β = 1 = λa + (1 − λ)b; λ ∈ [0, 1] .

5
6 Nierówności 1. Nierówności i funkcje wypukłe

Niech I ⊆ R będzie dowolnym przedziałem. Mówimy, że funkcja f : I → R jest wypukła,


jeśli dla dowolnych x, y ∈ I i dla dowolnych α > 0 i β > 0 takich, że α + β = 1, zachodzi
nierówność
f (αx + βy) 6 αf (x) + βf (y) .
Jeśli powyższa nierówność jest ostra (w przypadku gdy liczby α i β są większe od zera), to
mówimy, że funkcja f jest ściśle wypukła.
Mówimy, że funkcja f : I → R jest wklęsła, jeśli dla dowolnych x, y ∈ I i dla dowolnych
α > 0 i β > 0 takich, że α + β = 1, zachodzi nierówność

f (αx + βy) > αf (x) + βf (y) .

Jeśli powyższa nierówność jest ostra (w przypadku gdy liczby α i β są większe od zera), to
mówimy, że funkcja f jest ściśle wklęsła.
Przez −f oznaczamy funkcję przeciwną do funkcji f . Jej dziedziną jest dziedzina funkcji f
oraz dla wszystkich x z dziedziny mamy:

(−f )(x) = −f (x).

Jest oczywiste, że funkcja f jest wklęsła (odpowiednio ściśle wklęsła) wtedy i tylko wtedy,
gdy funkcja −f jest wypukła (odp. ściśle wypukła). Ta prosta uwaga pozwala nam w wielu
wypadkach ograniczyć się do badania tylko funkcji wypukłych lub ściśle wypukłych.

Przykładem funkcji wypukłej jest każda funkcja liniowa, tzn. funkcja h : I → R (gdzie I
jest przedziałem) taka, że
h(x) = ux + v
dla x ∈ I, gdzie u i v są ustalonymi liczbami rzeczywistymi. Jeśli bowiem a, b ∈ I, α, β ∈ R,
α > 0, β > 0 oraz α + β = 1, to

h(αa + βb) = u(αa + βb)u + v


= α(ua + v) − αv + β(ub + v) − βv + v
= αh(a) + βh(b) + (α + β)v − v
= αh(a) + βh(b).

Mamy tu nawet równość


h(αa + βb) = αh(a) + βh(b).
Funkcja h jest więc wypukła i jednocześnie wklęsła. Można udowodnić:

1.1.3. Niech I ⊆ R będzie przedziałem. Jeśli funkcja h : I → R jest jednocześnie wypukła i


wklęsła, to istnieją liczby rzeczywiste u, v takie, że

h(x) = ux + v,

dla wszystkich x ∈ I. ([Buln] 25).

W szczególnym przypadku, gdy u = 0, rozważane h jest funkcją stałą. Każda więc funkcja
stała jest jednocześnie wypukła i wklęsła.
Nierówności 1. Nierówności i funkcje wypukłe 7

1.1.4. Niech I ⊆ R będzie przedziałem. Jeśli funkcja f : I → R nie jest różnowartościowa i


jest jednocześnie wypukła i wklęsła, to jest funkcją stałą.

Podamy teraz następne przykłady funkcji wypukłych. W dowodach dotyczących warunku


wypukłości korzystać będziemy tylko z definicji.

1.1.5. Funkcja f : R → R, określona wzorem f (x) = x2 dla x ∈ R, jest wypukła.


D. Niech a, b, α, β ∈ R, α > 0, β > 0, α + β = 1. Wtedy:
αf (a) + βf (b) − f (αa + βb) = αa2 + βb2 − (αa + βb)2
= αa2 + βb2 − α2 a2 − β 2 b2 − 2αβab
= (α − α2 )a2 + (β − β 2 )b2 − 2αβab
= α(1 − (α)a2 + β(1 − β)b2 − 2αβab
= αβa2 + βαb2 − 2αβab = αβ(a2 − 2ab + b2 )
= αβ(a − b)2 > 0,
a zatem f (αa + βb) 6 αf (a) + βf (b). ⊠

1.1.6. Funkcja f : (0, ∞) → R, określona wzorem


1
f (x) =
x
dla x ∈ (0, ∞), jest wypukła.
D. Niech a, b, α, β ∈ R, a > 0, b > 0, α > 0, β > 0, α + β = 1. Wtedy:
α β 1 αb(αa + βb) + a(αa + βb) − ab
αf (a) + βf (b) − f (αa + βb) = + − =
a b αa + βb ab(αa + βb)
1  
= α2 ab + αβb2 + αβa2 + β 2 ab − ab
ab(αa + βb)
1  
= α2 ab + αβb2 + αβa2 + β 2 ab − (α + β)ab
ab(αa + βb)
1  
= (α2 − α)ab + αβb2 + αβa2 + (β 2 − β)ab
ab(αa + βb)
1  
= α(α − 1)ab + αβb2 + αβa2 + β(β − 1)ab
ab(αa + βb)
1  
= − αβab + αβb2 + αβa2 − βαab
ab(αa + βb)
αβ   αβ(a − b)2
= a2 − 2ab + b2 = > 0,
ab(αa + βb) ab(αa + βb)
a zatem f (αa + βb) 6 αf (a) + βf (b). ⊠

Suma funkcji wypukłych jest oczywiście funkcją wypukłą. Natomiast iloczyn funkcji wy-
pukłych nie musi być funkcją wypukłą. Jak już wiemy funkcje

f (x) = x2 i g(x) = −1

są wypukłe. Ich iloczynem f · g jest funkcja h(x) = −x2 , która nie jest wypukła.
8 Nierówności 1. Nierówności i funkcje wypukłe

1.1.7. Jeśli f, g są funkcjami wypukłymi i g jest funkcją rosnącą, to złożenie g ◦f jest funkcją
wypukłą.

Załóżmy, że I jest przedziałem. W definicji wypukłości funkcji f : I → R są nierówności,


w których występują dwie nieujemne liczby rzeczywiste α i β. W prosty sposób otrzymujemy
stąd nierówności z trzema nieujemnymi liczbami rzeczywistymi. Niech f : I → R będzie funk-
cją wypukłą i niech x, y, z ∈ I oraz niech α, β, γ będą nieujemnymi liczbami rzeczywistymi
takimi, że α + β + γ = 1. Wtedy

f (αx + βy + γz) 6 αf (x) + βf (y) + γf (z).

Podobne nierówności zachodzą dla czterech, pięciu, sześciu i większej liczby nieujemnych liczb
rzeczywistych z sumą równą jeden. Mówi o tym następujące twierdzenie Jensena.

1.1.8 (Jensen). Niech I ⊆ R będzie przedziałem. Jeśli f : I → R jest funkcją wypukłą, to


dla dowolnych liczb x1 , . . . , xn należących do I (gdzie n > 2) oraz dla dowolnych nieujemnych
liczb α1 , . . . , αn takich, że α1 + · · · + αn = 1 zachodzi nierówność

f (α1 x1 + · · · + αn xn ) 6 α1 f (x1 ) + · · · + αn f (xn ).

Nierówność ta jest często nazywana nierównością Jensena.


D. (Indukcja za względu na n). Dla n = 2 wynika to wprost z definicji funkcji wypukłej. Niech
n > 2 i załóżmy, że dla n jest już to udowodnione. Niech x1 , . . . , xn+1 ∈ I i niech α1 , . . . , αn+1
będą nieujemnymi liczbami rzeczywistymi, których suma jest równa 1. Oznaczmy: α = αn+1 oraz
β = 1 − α = α1 + · · · + αn . Jeśli α = 1, to α1 = · · · = αn = 0 i nie ma czego dowodzić. Załóżmy więc,
że α < 1. Oznaczmy również: γi = αi /β dla i = 1, . . . , n. Wtedy γ1 , . . . , γn są nieujemnymi liczbami
rzeczywistymi,
n
X n+1
X n
X
γi = 1 oraz αj xj = βu + αxn+1 , gdzie u = γi xi .
i=1 j=1 i=1

Mamy zatem:
 
n+1
X
f αj xj  = f (βu + αxn+1 ) 6 βf (u) + αf (xn+1 )
j=1

n
! n
!
X X αi
= βf γi xi + αf (xn+1 ) 6 β f (xi ) + αf (xn+1 )
i=1 i=1
β
n+1
X
= αj f (xj )
j=1

i to kończy dowód. ⊠

Podamy teraz kilka własności funkcji wypukłych.

1.1.9. Niech f : I → R będzie funkcją (gdzie I ⊆ R jest przedziałem). Funkcja f jest


wypukła wtedy i tylko wtedy, gdy dla dowolnych x1 , x2 , x3 ∈ I, takich że x1 < x2 < x3 ,
zachodzi nierówność
x3 − x2 x2 − x1
f (x2 ) 6 f (x1 ) + f (x3 )
x2 − x1 x3 − x1
Nierówności 1. Nierówności i funkcje wypukłe 9

lub równoważnie, w bardziej symetrycznej postaci


f (x1 ) f (x2 ) f (x3 )
+ + >0
(x1 − x2 )(x1 − x3 ) (x2 − x3 )(x2 − x1 ) (x3 − x1 )(x3 − x2 )
lub w postaci wyznacznikowej
1 x1 f (x1 )
1 x2 f (x2 ) > 0.
1 x3 f (x3 )
([Fich], [MiV] s.15, [Buln] s.26).

x3 − x x2 − x1
D. Wynika to z równości x2 = αx1 + βx3 , gdzie α = , β= . (Oczywiście α > 0,
x3 − x1 x3 − x1
β > 0 i α + β = 1). ⊠

1.1.10. Niech f : I → R będzie funkcją wypukłą. Jeśli z < y < x, to


f (y) − f (z) f (x) − f (z) f (x) − f (y)
6 6 .
y−z x−z x−y
y−z x−y y−z x−y
D. Ponieważ y = x+ z, więc f (y) 6 f (x) + f (z), czyli
x−z x−z x−z x−z
(∗) (x − z)f (y) 6 (y − z)f (x) + (x − y)f (z).

Stąd mamy (x − z)f (y) 6 ((x − y) − (x − y))f (x) + (x − y)f (z), czyli

(x − y)(f (x) − f (z)) 6 (x − z)(f (x) − f (y))

f (x) − f (z) f (x) − f (y)


i stąd 6 .
x−z x−y
Zapisując nierówność (∗) w postaci (x−z)f (y) 6 (y −z)f (x)+((x−z)−(y −z))f (z) otrzymujemy:

(x − z)(f (y) − f (z)) 6 (y − z)(f (x) − f (z)),

f (y) − f (z) f (x) − f (z)


czyli 6 .⊠
y−z x−z

1.1.11 ([Khr2]). Niech f : I → R (gdzie I ⊆ R jest przedziałem) będzie funkcją wypukłą.


Jeśli x1 > x2 , y1 > y2 , x1 6= y1 i x2 6= y2 , to
f (x1 ) − f (y1 ) f (x2 ) − f (y2 )
> .
x1 − y1 x2 − y2
D. Zamieniając ewentualnie x1 na y1 oraz x2 na y2 , możemy założyć, że x1 > y1 . Mamy wtedy
x1 > y1 > y2 . Zachodzi wówczas jeden z następujących 8 przypadków.

(1) x1 = x2 > y1 > y2 , (5) x1 > x2 = y1 > y2 ,


(2) x1 = x2 > y1 = y2 , (6) x1 > y1 > x2 > y2 ,
(3) x1 > x2 > y1 > y2 , (7) x1 > y1 = y2 > x2 ,
(4) x1 > x2 > y1 = y2 , (8) x1 > y1 > y2 > x2 .

Dla każdego z tych przypadków rozpatrujemy nierówności podane w 1.1.10 i otrzymujemy tezę. ⊠
10 Nierówności 1. Nierówności i funkcje wypukłe

1.1.12. Niech I ⊆ R będzie przedziałem, f : I → R funkcją wypukłą oraz A odcinkiem


domkniętym zawartym we wnętrzu przedziału I. Spełniony jest wówczas warunek Lipschitza,
tzn. istnieje dodatnia liczba M taka, że

|f (y) − f (x)| < M |y − x|,

dla wszystkich x, y ∈ A. ([Buln] s.27).

1.1.13. Jeśli f1 , . . . , fn : I → R+ są funkcjami wklęsłymi, to ich średnia geometryczna


p
f= n
f1 · · · fn

również jest funkcją wklęsłą. ([IMO] Longlist 1978).

⋆ E. F. Beckenbach, An inequality of Jensen, [Mon] 53(9)(1946) 501-505.


J. Górnicki, Funkcje wypukłe i nierówność Jensena, [Gorn], 118-122.
O. Izboldin, L. Kurlandczyk, Nierówność Jensena, [Kw] 4/1990 57-62.
Hojoo Lee, Jensen’s inequality, [LeH2] 42-44.
D. S. Mitrinović, J. E. Pećarić, A. M. Finc, Convex funct. and Jensen’s inequality, [M-pf].
oooooooooooooooooooooooooooooooooooooooooooooooooooooooooooooooooooooo
1.2 Funkcje J-wypukłe
oooooooooooooooooooooooooooooooooooooooooooooooooooooooooooooooooooooo
Niech I ⊆ R będzie przedziałem. Mówimy, że funkcja f : I → R jest J-wypukła lub
wypukła w sensie Jensena, jeśli dla wszystkich x, y ∈ I zachodzi nierówność
 
x+y f (x) + f (y)
f 6 .
2 2
Mówimy, że funkcja f : I → R jest J-wklęsła lub wklęsła w sensie Jensena, jeśli funkcja −f
jest J-wypukła. Takie definicje mamy, na przykład, w: [MiV] s.11, [Mitr], [Mit2] s.54.

1.2.1 (Jensen 1905, [MiV] s.12). Jeśli f : I → R jest funkcją J-wypukłą, to dla dowolnych
liczb x1 , . . . , xn należących do I (gdzie n > 2) oraz dla dowolnych nieujemnych liczb wymier-
nych α1 , . . . , αn takich, że α1 + · · · + αn = 1, zachodzi nierówność

f (α1 x1 + · · · + αn xn ) 6 α1 f (x1 ) + · · · + αn f (xn ).

1.2.2. Jeśli J-wypukła funkcja jest ciągła, to jest wypukła.

1.2.3 (Jensen 1905). Jeśli I jest odcinkiem domkniętym, to każda ograniczona J-wypukła
funkcja f : I → R jest ciągła. ([MiV] s.15).
Nierówności 1. Nierówności i funkcje wypukłe 11

W 1893 roku J. Hadamard zajmował się następującą nierównością, zwaną dzisiaj nierów-
nością Hadamarda.

1.2.4. Jeśli f : [a, b] → R jest funkcją wypukłą, to

  Zb
a+b 1 f (a) + f (b)
f 6 f (x)dx 6 .
2 b−a 2
a

⋆ B. G. Pachpatte, Hadamard’s inequalities, [Pach] 53-64.


oooooooooooooooooooooooooooooooooooooooooooooooooooooooooooooooooooooo
1.3 Funkcje wypukłe i ciągłość
oooooooooooooooooooooooooooooooooooooooooooooooooooooooooooooooooooooo
W ogólności funkcje wypukłe nie muszą być ciągłe.

1.3.1. Funkcja f : [0, 1] → R, okrślona wzorem


(
0, gdy x ∈ [0, 1),
f (x) =
1, gdy x = 1,

jest wypukła i nie jest ciągła.

Dla przedziałów otwartych taka sytuacja już się nie może zdarzyć.

1.3.2. Jeśli I ⊆ R jest przedziałem otwartym, to każda funkcja wypukła f : I → R jest


ciągła. (Patrz na przykład [Kras] 136).

⋆ G. H. Hardy, J. E. Littlewood, G. Polya, Continuous convex functions, [H-52], 71-72.


G. H. Hardy, J. E. Littlewood, G. Polya, Discontinuous convex functions, [H-52], 96-101.
D. S. Mitrinović, P. M. Vasić, Continuity of Jensen convex functions, [MiV], 14-15.
oooooooooooooooooooooooooooooooooooooooooooooooooooooooooooooooooooooo
1.4 Funkcje wypukłe i druga pochodna
oooooooooooooooooooooooooooooooooooooooooooooooooooooooooooooooooooooo

1.4.1. Niech f : I → R będzie funkcją (gdzie I ⊆ R jest przedziałem) posiadającą w I drugą


pochodną. Jeśli f ′′ (x) > 0 dla wszystkich x ∈ I, to f jest funkcją wypukłą.

D. Niech x1 , x2 ∈ I, x1 < x2 i niech 0 < λ < 1. Oznaczmy x = λx1 + (1 − λ)x2 . Wtedy x ∈ I oraz
x1 < x < x2 . Rozważmy przedziały [x1 , x] i [x, x2 ]. Z twierdzenia Lagrange’a (o wartości średniej)
istnieją c1 ∈ (x1 , x), c2 ∈ (x, x2 ) takie, że

f (x) − f (x1 ) f (x2 ) − f (x)


f ′ (c1 ) = , f ′ (c2 ) = .
x − x1 x2 − x

Oczywiście c1 < c2 . Ponieważ f ′′ (x) > 0, funkcja f ′ jest niemalejąca. Zatem f ′ (c1 ) 6 f ′ (c2 ), czyli

f (x) − f (x1 ) f (x2 ) − f (x)


6 .
x − x1 x2 − x
12 Nierówności 1. Nierówności i funkcje wypukłe

Stąd otrzymujemy nierówność

(x2 − x)(f (x) − f (x1 )) 6 (x − x1 )(f (x2 ) − f (x))

i następnie nierówność (x2 − x1 )f (x) 6 (x − x1 )f (x2 ) + (x2 − x)f (x1 ), czyli


x2 − x x − x1
f (x) 6 f (x1 ) + f (x2 ).
x2 − x1 x2 − x1
x2 − x x − x1
Zauważmy teraz, że λ = oraz 1 − λ = . Zatem
x2 − x1 x2 − x1

f (λx1 + (1 − λ)x2 ) < λf (x1 ) + (1 − λ)f (x2 ),

a to oznacza, że f jest funkcją wypukłą. ⊠

W ten sam sposób dowodzimy następne twierdzenie.

1.4.2. Niech f : I → R będzie dowolną funkcją (gdzie I ⊆ R jest przedziałem) posiadającą


w I drugą pochodną. Jeśli f ′′ (x) > 0 dla wszystkich x ∈ I, to f jest funkcją ściśle wypukłą.

1.4.3. Niech f : I → R będzie dowolną funkcją (gdzie I ⊆ R jest przedziałem) różniczkowalną


w I. Następujące warunki są równoważne.
(1) Funkcja f jest wypukła.
(2) Pochodna f ′ jest funkcją niemalejącą.
(3) f (x) > f (x0 ) + f ′ (x0 )(x − x0 ) dla dowolnych x, x0 ∈ I.
Jeśli dodatkowo funkcja f posiada drugą pochodną, to dochodzi jeszcze warunek
(4) f ′′ (x) > 0 dla dowolnego x ∈ I. (Patrz np. [Stee] 90, [Kras] 134).

Korzystając z powyższych twierdzeń łatwo można podać następne przykłady funkcji wy-
pukłych.

1.4.4. Przykłady wypukłych funkcji f : R → R :


(1) f (x) = |x|;
(2) f (x) = ex i ogólniej f (x) = ax dla a > 0;
(3) f (x) = xn , gdy n jest parzystą liczbą naturalną.

1.4.5. Przykłady wypukłych funkcji f : (0, ∞) → R :


(1) f (x) = xn dla n ∈ N;

(2) f (x) = − n x dla n > 2;
1
(3) f (x) = .
x

1.4.6. Funkcja f (x) = − sin x jest wypukła na odcinku [0, π].

Stąd w szczególności wynikają następujące nierówności.


Nierówności 1. Nierówności i funkcje wypukłe 13

1.4.7.
ex + ey
x+y
(1) e 2 6 , dla x, y ∈ R.
2
 
x + y 5 x5 + y 5
(2) 6 , dla x, y > 0.
2 2

1.4.8. n2 (x31 + · · · + x3n ) > (x1 + · · · + xn )3 , dla x1 , . . . , xn > 0. (Patrz 4.1.6).

1.4.9. Niech I ⊆ R będzie przedziałem i niech f : I → R będzie funkcją taką, że f (x) > 0
dla x ∈ I. Załóżmy, że funkcja f jest rosnąca lub wypukła. Wtedy

f (x)(x − y)(x − z) + f (y)(y − z)(y − x) + f (z)(z − x)(x − y) > 0

dla wszystkich x, y, z ∈ I z wyjątkiem przypadku x = y = z. (Patrz 4.5.6).

⋆ T. Andreescu, B. Enescu, Inequalities with convex functions, [AnE] 21-25.


P. S. Bullen, Convex functions, [Buln], 25-59.
P. S. Bullen, D. S. Mitrinović, P. M. Vasić, Convex functions, [B-mv], 21-33.
J. Górnicki, Nierówności, wypukłość i ekstrema, [Msn], 11(1993), 44-49.
G. H. Hardy, J. E. Littlewood, G. Polya, Convex functions, [H-52], 70-71.
G. H. Hardy, J. E. Littlewood, G. Polya, Further general prop. of convex functions, [H-52].
D. S. Mitrinović, Funkcje wypukłe, [Mitr], [Mit2] 54-62.
D. S. Mitrinović, P. M. Vasić, Convex functions, [MiV], 10-26.
B. G. Pachpatte, Inequalities involving convex functions, [Pach] 11-112.
Podstawowe własności funkcji wypukłych przedstawione są w wielu książkach z analizy matema-
tycznej (patrz na przykład: [Fich], [Kurt], [Mau1]). Godnymi polecenia są współczesne książki: [Druz],
[Kras], [Kry1], [Plos].
oooooooooooooooooooooooooooooooooooooooooooooooooooooooooooooooooooooo
1.5 Nierówność Karamaty
oooooooooooooooooooooooooooooooooooooooooooooooooooooooooooooooooooooo
Na podstawie [Khr2] oraz [Khr1].

Niech A ⊆ R będzie przedziałem. Dla każdej liczby naturalnej n oznaczmy przez A(n)
zbiór wszystkich ciągów (x1 , . . . , xn ) takich, że x1 , . . . , xn ∈ A oraz x1 > x2 > · · · > xn .
Wprowadzamy w zbiorze A(n) częściowy porządek , określony w następujący sposób. Niech
x = (x1 , . . . , xn ) oraz y = (y1 , . . . , yn ) będą ciągami należącymi do zbioru S(n) . Wówczas
x  y jeśli:
x1 > y1 ,
x1 + x2 > y1 + y2 ,
x1 + x2 + x3 > y1 + y2 + y3 ,
..
.
x1 + · · · + xn−1 > y1 + · · · + yn−1 ,
x1 + · · · + xn = y1 + · · · + yn .
Jeśli x  y i x 6= y, to piszemy x ≻ y. Stosować będziemy również oznaczenia y  x oraz
y ≺ x, w przypadkach gdy odpowiednio x  y oraz x ≺ y. Spójrzmy na kilka przykładów.

1.5.1. (1, 0, . . . , 0) ≻ ( 21 , 12 , 0, . . . , 0) ≻ ( 31 , 13 , 31 , 0, . . . , 0) ≻ · · · ≻ ( n1 , n1 , . . . , n1 ).
14 Nierówności 1. Nierówności i funkcje wypukłe

1.5.2. Niech A = [0, 1]. Jeśli x = (x1 , . . . , xn ) ∈ A(n) i x1 + x2 + · · · + xn = 1, to


 
1 1
(1, 0, . . . , 0)  x  ,..., .
n n
1.5.3. Jeśli x = (x1 , . . . , xn ) ∈ A(n) i y = (y1 , . . . , yn ), gdzie y1 = y2 = · · · = yn =
x1 + x2 + · · · + xn
, to y ∈ A(n) oraz x  y.
n
1.5.4. Rozważany porządek  nie jest na ogół porządkiem liniowym, tzn. jeśli x, y ∈ A(n) , to
nie musi być prawdą, że x  y lub y  x. Tak jest na przykład dla x = (5, 5, 0) , y = (6, 2, 2).

1.5.5 (Twierdzenie Karamaty). ([Khr2], [Khr1]). Niech A ⊆ R będzie dowolnym przedziałem i


niech f : A → R będzie funkcją wypukłą. Jeśli x = (x1 , . . . , xn ) i y = (y1 , . . . , yn ) są takimi
ciągami należącymi do A(n) , że x  y, to

f (x1 ) + f (x2 ) + · · · + f (xn ) > f (y1 ) + f (y2 ) + · · · + f (yn ).


D. ([Khr2]). Korzystamy z tożsamości Abela:
n
X n−1
X
ak bk = uk (bk − bk+1 ) + un bn ,
k=1 k=1

gdzie uk = a1 + · · · + ak . (Tutaj a1 , . . . , an , b1 , . . . , bn są dowolnymi liczbami).


Jeśli dla pewnego k ∈ {1, . . . , n} zachodzi równość xk = yk , to xk i yk możemy wyeliminować,
gdyż f (xk ) i f (yk ) występują po obu stronach dowodzonej nierówności. Możemy zatem założyć, że
xk 6= yk dla wszystkich k = 1, . . . , n. Oznaczmy:
f (xk ) − f (yk )
Xk = x1 + · · · + xk , Yk = y1 + · · · + yk , Dk = ,
xk − yk
dla wszystkich k = 1, . . . , k. Ponieważ x  y, więc Xk > Yk dla wszystkich k < n, Xn = Yn oraz (na
mocy 1.1.11) Dk > Dk+1 dla k < n. Mamy zatem oczywistą nierówność
n−1
X
(∗) (Xk − Yk )(Dk − Dk+1 ) + (Xn − Yn )Dn > 0.
k=1

Niech ak = xk − yk , bk = Dk , dla k = 1, . . . , n. Wówczas z tożsamości Abela i nierówności (∗)


otrzymujemy:
n
X n
X n−1
X
(xk − yk )Dk = ak bk = uk (bk − bk+1 ) + un bn
k=1 k=1 k=1
n−1
X
= (Xk − Yk )(Dk − Dk+1 ) + (Xn − Yn )Dn > 0,
k=1

n
X n
X n
X n
f (xk ) − f (yk ) X
czyli (xk −yk )Dk > 0. Zatem, (f (xk )−f (yk )) = (xk −yk ) = (xk −yk )Dk >
xk − yk
k=1 k=1 k=1 k=1
n
X n
X
0 i mamy: f (xk ) > f (yk ). ⊠
k=1 k=1

Powyższą nierówność można uogólnić w następujący sposób.


Nierówności 1. Nierówności i funkcje wypukłe 15

1.5.6 ([Fu47], [Khr1]). Niech f : A → R będzie funkcją wypukłą (gdzie A ⊆ R jest przedzia-
łem) i niech p1 , . . . , pn będą dodatnimi liczbami. Niech (x1 , . . . , xn ), (y1 , . . . , yn ) będą ciągami
o wyrazach należących do A takimi, że

x1 > x2 > · · · > xn , y1 > y2 > · · · > yn


k
X k
X n
X n
X
oraz pi xi > pi yi dla k = 1, . . . , n − 1 i pi xi = pi yi . Wtedy
i=1 i=1 i=1 i=1

p1 f (x1 ) + p2 f (x2 ) + · · · + pn f (xn ) > p1 f (y1 ) + p2 f (y2 ) + · · · + pn f (yn ).

Przedstawimy teraz kilka zastosowań nierówności Karamaty.

1.5.7.([Khr2], [Khr1]). Niech f : A → R będzie funkcją wypukłą (gdzie A ⊆ R jest przedziałem).


Wówczas dla dowolnych a, b, c ∈ A zachodzi nierówność

f (a + b) + f (b + c) + f (c + a) 6 f (2a) + f (2b) + f (2c).

D. Niech a, b, c ∈ A. Możemy założyć, że a > b > c. Wtedy

(2a, 2b, 2c)  (a + b, a + c, b + c)

i teza wynika z 1.5.5. ⊠

1.5.8. Niech f : (0, ∞) → R będzie funkcją wypukłą. Jeśli a, b, c są długościami boków


trójkąta, to

f (a) + f (b) + f (c) 6 f (a + b − c) + f (b + c − a) + f (c + a − b).

([Khr2], [Khr1]).

1 1 1
D. Niech x = (b + c − a), y = (a + c − b), z = (a + b − c). Wtedy x + y = c, x + z = c,
2 2 2
y + z = a. Wówczas, na mocy 1.5.7, mamy:

f (x + y) + f (y + z) + f (z + a) 6 f (2x) + f (2y) + f (2z),

czyli f (a) + f (b) + f (c) 6 f (a + b − c) + f (b + c − a) + f (c + a − b). ⊠

1.5.9 (Szegö 1950). Niech f : A → R będzie funkcją wypukłą (gdzie A ⊆ R jest przedziałem).
Jeśli
a1 > a2 > · · · > aan−1 > 0
są liczbami należącymi do A, to

f (a1 ) − f (a2 ) + f (a3 ) − · · · + f (a2n−1 ) > f (a1 − a2 + a3 − · · · + a2n−1 ).

([Khr2], [Khr1], [AnE] s.23).


16 Nierówności 1. Nierówności i funkcje wypukłe

D. Niech b = a1 − a2 + a3 − · · · + a2n−1 . Wtedy nierówność ta ma postać

f (a1 ) + f (a3 ) + · · · + f (a2n−1 ) > f (b) + f (a2 ) + · · · + f (a2n−2 )

i to wynika z nierówności 1.5.5, gdyż (a1 , a3 , . . . , a2n−1 )  (a2 , a4 , . . . , a2n−2 , b). ⊠

⋆ Jovan Karamata żył w latach 1902 − 1967; matematyk jugosłowiański.


Nazwanie nierówności z twierdzenia 1.5.5 nierównością Karamaty jest (jak czytamy w [Khr2] i
[Khr1]) sprawą sporną. Nierówność ta została udowodniona w 1932 roku przez Karamatę w pracy
[Kara], w której podano również pewne jej uogólnienia. Trzy lata wcześniej Hardy, Littlewood i Polya
udowodnili ją w [H-lp]. Podobna nierówność pojawiła się jeszcze wcześniej (I. Schur 1923).
E. F. Beckenbach, R. Bellman, Nierówność Karamaty, [BeB], 48-50.
S. Gueron, Substitutions, inequalities, and history, [Crux], 2002, 88-90.
S. Gueron, R. Tessler, Majorization ant the Karamata inequality, Etgar-Gilionot Mathematica,
48-49(1999), 4-10.
⊡⊡⊡⊡⊡⊡⊡⊡⊡⊡⊡⊡⊡⊡⊡⊡⊡⊡⊡⊡⊡⊡⊡⊡⊡⊡⊡⊡⊡⊡⊡⊡⊡⊡⊡⊡⊡⊡⊡⊡⊡⊡⊡⊡⊡⊡⊡⊡⊡⊡⊡⊡
2 Klasyczne nierówności
⊡⊡⊡⊡⊡⊡⊡⊡⊡⊡⊡⊡⊡⊡⊡⊡⊡⊡⊡⊡⊡⊡⊡⊡⊡⊡⊡⊡⊡⊡⊡⊡⊡⊡⊡⊡⊡⊡⊡⊡⊡⊡⊡⊡⊡⊡⊡⊡⊡⊡⊡⊡

oooooooooooooooooooooooooooooooooooooooooooooooooooooooooooooooooooooo
2.1 Średnia arytmetyczna i średnia geometryczna
oooooooooooooooooooooooooooooooooooooooooooooooooooooooooooooooooooooo
x1 + · · · + xn
Średnią arytmetyczną liczb x1 , . . . , xn nazywamy liczbę . Średnią geomet-
√ n
ryczną dodatnich liczb x1 , . . . , xn nazywamy liczbę x1 · · · xn .
n

2.1.1. Dla dowolnych dodatnich liczb rzeczywistych x1 , . . . , xn , zachodzi nierówność

x1 + x2 + · · · + xn √
> n x1 x2 · · · xn .
n
Równość zachodzi wtedy i tylko wtedy, gdy x1 = x2 = · · · = xn .

Istnieje wiele przeróżnych dowodów powyższej nierówności. Przedstawimy teraz jeden ze


znanych dowodów. W tym celu udowodnimy najpierw:

2.1.2. Jeśli a1 , . . . , as są dodatnimi liczbami rzeczywistymi takimi, że a1 · · · an = 1, to

a1 + a2 + · · · + an > n .

Równość zachodzi wtedy i tylko wtedy, gdy a1 = · · · = an = 1.

D. ([JeL], indukcja matematyczna względem n). Dla n 6 2 jest to oczywiste. Załóżmy, że dla
pewnego n > 2 suma n dowolnych liczb dodatnich jest nie mniejsza od n, gdy ich iloczyn jest równy 1.
Mamy wykazać, że jeżeli a1 > 0, a2 > 0, . . . , an+1 > 0 oraz a1 a2 · · · an+1 = 1, to

a1 + a2 + · · · + an + an+1 > n + 1.

Liczby a1 , . . . , an+1 można tak ponumerować, aby a1 6 a2 6 · · · 6 an+1 . Mamy a1 6 1. Istotnie,


gdyby a1 > 1, to iloczyn a1 · · · an+1 byłby większy od 1, wbrew założeniu. Podobnie wykazujemy, że
an+1 > 1.
Ponieważ a2 a3 · · · an (an+1 a1 ) = 1, więc na mocy założenia a2 + a3 + · · · + an + an+1 a1 > n. Mamy
zatem:
a1 + a2 + · · · + an+1 − (n + 1) = (a2 + a3 + · · · + an + an+1 a1 ) + a1 + an+1 − a1 an+1 − (n + 1)
> n + a1 + an+1 − a1 an+1 − n − 1
= (1 − a1 )(an+1 − 1) > 0,

gdyż, jak zauważyliśmy, a1 6 1, an+1 > 1. W ten sposób nierówność a1 + a2 + · · · + an + an+1 > n + 1
została udowodniona.
Z dowodu wynika, że równość a1 + · · · + an + an+1 = n + 1 zajdzie wtedy i tylko wtedy, gdy
a1 = an+1 = 1, a więc gdy wszystkie liczby a1 , . . . , an+1 są równe 1. ⊠

17
18 Nierówności 2. Klasyczne nierówności

xi √
Dowód nierówności 2.1.1. Niech ai = , dla i = 1, . . . , n, gdzie g = n x1 · · · xn . Wtedy
g
a1 , . . . , an są dodatnimi liczbami rzeczywistymi takimi, że a1 · · · an = 1. Korzystając z 2.1.2, mamy
a1 + · · · + an > n, a więc
x1 + x2 + · · · + xn √
> n x1 x2 · · · xn .
n
Równość zachodzi wtedy i tylko wtedy, gdy a1 + · · · + an = n, czyli tylko wtedy gdy a1 = · · · = an = 1
(patrz 2.1.2), a to implikuje, że x1 = · · · = xn . ⊠

Interesujący dowód nierówności 2.1.1 podał Sławomir Cynk w [Dlt] 8/1991 (patrz [CiCP]
22-23). Wykorzystał on tak zwaną indukcję wsteczną.
Drugi dowód nierówności 2.1.1.
(a). Udowodnimy najpierw, że z prawdziwości nierówności 2.1.1 dla n wynika jej prawdziwość dla
n − 1. Niech bowiem x1 , . . . , xn−1 będą liczbami dodatnimi. Przyjmijmy:

xn = n−1 x1 x2 · · · xn1 .
x1 + · · · + xn √
Korzystając z prawdziwości dla n mamy nierówność > n x1 · · · xn . Ale
n
q

xnn−1 · xn = xn ,
n
n
x1 · · · xn =

x1 + · · · xn−1 √
więc x1 + · · · + xn > nxn i wobec tego > xn = n−1
x1 , · · · xn−1 .
n−1
(b). Teraz wykażemy, że nierówność 2.1.1 jest prawdziwa gdy n = 2s jest potęgą dwójki. Dla s = 1
jest to dobrze znana nierównoć dla dwóch dodatnich liczb. Niech s > 2 i załóżmy, że nierówność 2.1.1
jest prawdziwa dla n = 2s−1 . Niech x1 , . . . , x2n będą liczbami dodatnimi. Przyjmijmy:
x2k−1 + x2k √
yk = , zk = x2k−1 x2k ,
2
dla k = 1, 2, . . . , n. Oczywiście yk > zk dla wszystkich k = 1, . . . , n. Mamy wtedy:
q
√ √ z1 + · · · + z n y1 + · · · + y n
2n
x1 x2 . . . , x2n = 2n z12 z22 · · · zn2 = n z1 z2 · · · zn 6 6
n n
x1 +x2 x3 +x4 x2n−1 +x2n
2 + 2 + ··· + 2 x1 + x2 + · · · x2n
= =
n 2n
i stąd wynika teza. ⊠
Inne dowody nierówności 2.1.1 znajdziemy w następnych rozdziałach tej książki. Udowod-
niliśmy nierówność 2.1.1 przy pomocy nierówności 2.1.2. Zauważmy, że te dwie nierówności
są równoważne. Nierówność 2.1.2 wynika natychmiast z nierówności 2.1.1.
Nierówność 2.1.1 jest szczególnym przypadkiem następującej ogólniejszej nierówności.
n
X
2.1.3. Niech p1 , . . . , pn będą liczbami dodatnimi takimi, że pi = 1. Wówczas
i=1

p1 x1 + p2 x2 + · · · + pn xn > xp11 xp22 · · · xpnn ,

dla x1 , . . . , xn > 0. Równość zachodzi wtedy i tylko wtedy, gdy x1 = x2 = · · · = xn .


(Patrz, na przykład, [Fich], [Stee] 22).
Nierówności 2. Klasyczne nierówności 19

α β
2.1.4. xα+β + y α+β > xα y β , dla α, β, x, y > 0.
α+β α+β
α β
D. ([Stee] 32, 232). Podstawiamy: p1 = , p2 = , a1 = xα+β , a2 = y α+β .
α+β α+β
Wtedy p1 + p2 = 1 i mamy:
α β
xα+β + y α+β = p1 a1 + p2 a2 > ap11 ap22 = xα y β .
α+β α+β
Wykorzystaliśmy nierówność 2.1.3. ⊠

2.1.5. xp+1 + y p+1 > xp y + xy p , dla p, x, y > 0.


D. Wynika to z nierówności 2.1.4:
   
p 1 1 p
xp y + xy p 6 xp+1 + y p+1 + xp+1 + y p+1 = xp+1 + y p+1 . ⊠
p+1 p+1 p+1 p+1

Z nierówności 2.1.2 wynikają nierówności:


a b
2.1.6. + > 2, dla a, b > 0.
b a
a b c
2.1.7. + + > 3, dla a, b, c > 0.
b c a
2.1.8. Dla dowolnych dodatnich liczb rzeczywistych x1 , . . . , xn zachodzi nierówność

x1 x2 xn−1 xn
+ + ··· + + >n .
x2 x3 xn x1
Równość zachodzi wtedy i tylko wtedy, gdy x1 = · · · = xn .

2.1.9. Z nierówności 2.1.8 wynika nierówność 2.1.2.

D. Niech a1 , . . . , an będą dodatnimi liczbami rzeczywistymi, których iloczyn jest równy 1. Niech
xi = ai ai+1 · · · an , dla i = 1, . . . , n.
x1 x2 xn−1
Wtedy = a1 , = a2 , . . . , = an−1 oraz
x2 x3 xn
xn an an
= = = an .
x1 a1 · · · an 1
x1 xn
Mamy więc (na mocy 2.1.8): a1 = · · · + an = + ··· + > n. ⊠
x2 x1

Zanotujmy kilka nierówności wynikających w prosty sposób z nierówności 2.1.2.

2.1.10. ab + bc + ca > 3, dla a, b, c > 0, abc = 1.

2.1.11. a(b + 1) + b(c + 1) + c(a + 1) > 6, dla a, b, c > 0, abc = 1. ([OMm] 1996/1997).
20 Nierówności 2. Klasyczne nierówności

2.1.12. a2 + b2 + c2 + d2 + ab + ac + ad + bc + bd + dc > 10, dla a, b, c, d > 0, abcd = 1.


([OM] Rosja).

2.1.13. Jeśli x1 , . . . , xn są liczbami dodatnimi takimi, że x1 · · · xn = 1, to:


1 1
(1) + ··· + > n.
x1 xn
(2) xα1 + · · · + xαn > n, dla dowolnej liczby rzeczywistej α.

Następna nierówność również wynika z nierówności 2.1.2.

2.1.14. Jeśli x1 , . . . , xn są dodatnimi liczbami takimi, że x1 · · · xn = 1, to

2n 6 (1 + x1 )(1 + x2 ) · · · (1 + xn ).

([IMO] Longlist 1959-1960, [Djmp] s.36, [OM] Rosja 1984/1985).

D. (Sposób I). Wynika to z nierówności 2.1.2, gdyż iloczyn


(1 + x1 )(1 + x2 ) · · · (1 + xn )

jest sumą 2n składników, których iloczyn jest równy 1.


(Sposób II. [Rias]). Korzystając z nierówności pomiędzy średnimi, otrzymujemy ciąg nierówności:
√ √ √
1 + x1 > 2 x1 , 1 + x2 > 2 x2 , . . . , 1 + xn > 2 x2 .

Mnożymy te wszystkie nierówności stronami przez siebie i otrzymujemy tezę. ⊠


Nierówność 2.1.14 jest szczególnym przypadkiem następującej ogólniejszej nierówności
Huygensa.

2.1.15. Dla dowolnych dodatnich liczb rzeczywistych x1 , . . . , xn zachodzi nierówność


 √ n
1+ n
x1 · · · xn 6 (1 + x1 )(1 + x2 ) · · · (1 + xn ).

([IMO] Longlist 1971).

Z tej nierównośći wynika następna nierówność.

2.1.16. Jeśli x1 , . . . , xn są dodatnimi liczbami takimi, że x1 · · · xn = 1, to

3n 6 (2 + x1 )(2 + x2 ) · · · (2 + xn ).

([OM] Chiny 1989/1990, [Liu1] 130-131).


   
D. (2 + x1 )(2 + x2 ) · · · (2 + xn ) = 1 + (1 + x1 ) · · · 1 + (1 + xn )
p  n
> n
(1 + x1 )(1 + x2 ) · · · (1 + xn )
1+
 √ n
1 + 2n = 3n .
n
>
Wykorzystaliśmy dwa razy nierówność 2.1.15. ⊠
W nierówności 2.1.15 występuje średnia geometryczna. Średnią arytmetyczną też tu moż-
na dodać.
Nierówności 2. Klasyczne nierówności 21

2.1.17. Dla dowolnych dodatnich liczb rzeczywistych x1 , . . . , xn zachodzą nierówności


 n  n
√ x1 + · · · + xn
1+ n
x1 · · · xn 6 (1 + x1 )(1 + x2 ) · · · (1 + xn ) 6 1 + .
n
([IMO] Longlist 1971, [Djmp] s.77).

2.1.18. Dla dowolnych dodatnich liczb rzeczywistych a oraz x1 , . . . , xn zachodzą nierówności


 n  n
√ x1 + · · · + xn
a+ n
x1 · · · xn 6 (a + x1 )(a + x2 ) · · · (a + xn ) 6 a + .
n
(T. Mitev, [Crux] z.2938).

D. Prawa nierówność wynika z nierówności pomiędzy średnimi:


x1 + · · · + xn (a + x1 ) + · · · + (a + xn ) p
a+ = > n (a + x1 ) · · · (a + xn ).
n n
Teraz wykazujemy nierówność po lewej stronie:
 r
n x1   xn   x1 xn n  √ n
(a + x1 ) · · · (a + xn ) = a 1+ ··· 1 + > an 1 + n
··· = a + n x1 · · · xn .
a a a a
Wykorzystaliśmy nierówność Huygensa 2.1.15. ⊠

Niech x = (xn ) będzie nieskończonym ciągiem liczb dodatnich. Jeśli n jest liczbą natu-
ralną, to przez An (x) i Gn (x) oznaczamy odpowiednio średnią arytmetyczną i geometryczną
liczb x1 , . . . , xn , tzn.:

x1 + · · · + xn √
An (x) = , Gn (x) = n
x1 · · · xn .
n
  n 
Gn−1 (x) An−1 (x) Gn (x)
2.1.19. An (x) = (n − 1) + , dla n > 2. ([Mit2] s.20).
n Gn−1 (x) Gn−1 (x)

2.1.20. q
G1 (x) + G2 (x) + · · · + Gn (x)
6 n A1 (x)A2 (x) · · · An (x).
n
Innymi słowy, dla dowolnych dodatnich liczb rzeczywistych x1 , . . . , xn zachodzi nierówność
√ √ r
x1 + x1 x2 + · · · + n x1 x2 · · · xn x1 + x2 x1 + · · · + xn
6 n x1 · ··· .
n 2 n
(Kiran Kedlaya, [Mild]).

2.1.21. n (An (x) − Gn (x)) > (n − 1) (An−1 (x) − Gn−1 (x)) , dla n > 2. Stąd wynika, że jeśli
n > m, to
n (An (x) − Gn (x)) > m (Am (x) − Gm (x)) .
([MM] 45(3)(1972) 172-173, [Mit2] s.19).

 n  n−1
Gn (x) Gn−1 (x)
2.1.22. 6 , dla n > 2. ([Mit2] s.73).
An (x) An−1 (x)
22 Nierówności 2. Klasyczne nierówności

a+b+c √
3 3
2.1.23. Niech a, b, c > 0, A = , G = abc, H = −1 . Wtedy
3 a + b + c−1
−1

 3
A 1 3A
> + .
G 4 4H
([IMO] Longlist 1992).

⋆ P. S. Bullen, The geometric mean-arithmetic mean inequality, [Buln], 71-124.


P. S. Bullen, Refinements of the geometric mean-arithmetic mean inequality, [Buln], 125-154.
G. H. Hardy, J. E. Littlewood, G. Polya, The theorem of the arithm. and geom. means, [H-52].
L. Kurlandczyk, Średnia arytmetyczna i geometryczna, [Ko00], 127-135.
N. Lord, On inequalities equivalent to the inequality of the means, [MG] 525(2008) 529-533.
H. Pawłowski, Nierówność o średnich i jej uogólnienie, [Dlt] 2/1994 8-11.
R. M. Young, Another simple proof of the theorem of the means, [MG] 520(2007) 107-109.
oooooooooooooooooooooooooooooooooooooooooooooooooooooooooooooooooooooo
2.2 Średnia harmoniczna i średnia kwadratowa
oooooooooooooooooooooooooooooooooooooooooooooooooooooooooooooooooooooo
Jeśli x = (x1 , . . . , xn ) jest ciągiem liczb dodatnich, to przez H(x), G(x), A(x) i K(x)
oznaczamy odpowiednio średnie harmoniczną, geometryczną, arytmetyczną i kwadratową te-
go ciągu. Średnie te definiuje się następująco.
 −1
1 1 1
H(x) = n + + ··· + ,
x1 x2 xn

G(x) = n
x1 x2 · · · xn ,
1
A(x) = (x1 + x2 + · · · + xn ),
n
s
1 2
K(x) = (x + x22 + · · · + x2n ).
n 1

2.2.1. Pomiędzy tymi średnimi zachodzą nierówności:

H(x) 6 G(x) 6 A(x) 6 K(x).

W każdej z tych nierówności równość ma miejsce wtedy i tylko wtedy, gdy x1 = x2 = · · · = xn .

2.2.2 (Sierpiński 1909). A(x)H(x)n−1 6 G(x)n 6 A(x)n−1 H(x). ([M-pf] 21).

2.2.3. A(x)n+2 > G(x)n K(x)2 . ([MM] 63(3)(1990) 193).

2.2.4. H(x1 + 1, . . . , xn + 1) > H(x1 , x2 , . . . , xn ) + 1, dla x1 , . . . , xn > 0. ([OM] Mołdawia 2001).

⋆ P. S. Bullen, The arithmetic, geometric and harmonic means, [Buln], 60-174.


P. S. Bullen, D. S. Mitrinović, P. M. Vasić, The arithm., geom. and harmonic means, [B-mv].
D. S. Mitrinović, Nierówności zachodzące dla podstawowych średnich, [Mit2], 19-27.
L. Kurlandczyk, Klasyczne nierówności pomiędzy średnimi, [Ko00], 138-142.
Nierówności 2. Klasyczne nierówności 23

oooooooooooooooooooooooooooooooooooooooooooooooooooooooooooooooooooooo
2.3 Średnie złożone
oooooooooooooooooooooooooooooooooooooooooooooooooooooooooooooooooooooo

2.3.1. Dla danych liczb nieujemnych a i b definiujemy dwa nieskończone ciągi (an ) i (gn ) w
następujący sposób:
a+b √ an + gn √
a1 = , g1 = ab, an+1 = , gn+1 = an gn .
2 2
Wtedy dla każdej liczby naturalnej n zachodzą nierówności

min(a, b) 6 g1 6 g2 6 · · · 6 gn 6 an 6 an−1 6 · · · 6 a1 6 max(a, b).

Równości zachodzą wtedy i tylko wtedy, gdy a = b. ([Mit2] s.28).

2.3.2. Dla danych liczb nieujemnych a i b definiujemy dwa nieskończone ciągi (an ) i (hn ) w
następujący sposób:
a+b 2 an + hn 2
a1 = , h1 = , an+1 = , hn+1 = .
2 1
a + 1b 2 1
an + 1
hn

Wtedy dla każdej liczby naturalnej n zachodzą nierówności

min(a, b) 6 h1 6 h2 6 · · · 6 hn 6 an 6 an−1 6 · · · 6 a1 6 max(a, b).

Równości zachodzą wtedy i tylko wtedy, gdy a = b. ([Mit2] s.29).

⋆ D. S. Mitrinović, Średnie złożone, [Mit2], 27-29.


oooooooooooooooooooooooooooooooooooooooooooooooooooooooooooooooooooooo
2.4 Średnie potęgowe
oooooooooooooooooooooooooooooooooooooooooooooooooooooooooooooooooooooo
Jeśli x = (x1 , . . . , xn ) jest ciągiem liczb dodatnich i t ∈ R ∪ {−∞, +∞}, to przez Mt (x)
oznaczamy liczbę rzeczywistą, zwaną średnią potęgową stopnia t ciągu x, zdefiniowaną nastę-
pująco
  1

 1 t t


 (x + xt2 + ··· + xtn ) , gdy 0 6= t ∈ R,


 n 1
 √
Mt (x) = 
n
x1 x2 · · · xn , gdy t = 0,



 min{x1 , . . . , xn } gdy t = −∞,




max{x1 , . . . , xn } gdy t = +∞.
W szczególności:
M−1 (x) = H(x) średnia harmoniczna,
M0 (x) = G(x) średnia geometryczna,
M1 (x) = A(x) średnia arytmetyczna,
M2 (x) = K(x) średnia kwadratowa.
24 Nierówności 2. Klasyczne nierówności

Zatem,
M−1 (x) 6 M0 (x) 6 M1 (x) 6 M2 (x)
(patrz 2.2.1).

Określenie średniej potęgowej stopnia zero, jako średniej geometrycznej, jest naturalne,
gdyż:

2.4.1. Jeśli x = (x1 , . . . , xn ) jest ustalonym ciągiem liczb dodatnich, to

lim Mt (x) = M0 (x).


t→0

([B-mv], [Buln]).

To samo dotyczy średnich M−∞ (x) i M∞ (x):

2.4.2. lim Mt (x) = M−∞ (x), lim Mt (x) = M∞ (x). ([B-mv], [Buln]).
t→−∞ t→∞

2.4.3. Niech x = (x1 , . . . , xn ) będzie ustalonym ciągiem liczb dodatnich i niech t, s będą
dowolnymi liczbami rzeczywistymi. Wtedy:
(1) min(x) 6 Mt (x) 6 max(x);
(2) jeśli s < t, to Ms (x) 6 Mt (x);
(3) jeśli s < t, to Ms (x) = Mt (x) ⇐⇒ x1 = x2 = · · · = xn . ([B-mv], [Buln]).

Powyższe fakty były już znane w dziewiętnastym wieku (patrz [B-mv], [Buln] s.203). W
szczególności mamy:

2.4.4. Jeśli x1 , . . . , xn > 0, to


r r r
1 2 1 3 1 4
(x + x22 + · · · + x2n ) 6 (x + x32 + · · · + x3n ) 6 (x + x42 + · · · + x4n ) 6 . . . .
3 4

n 1 n 1 n 1
q q
2.4.5. x21 + x22 + ··· + x2n > 3
x31 + x32 + · · · + x3n . ([Nord] 1990).

√ √
2.4.6. m
xm + y m > n
xn + y n , dla x, y > 0, gdy m 6 n. ([OM] Moskwa 1997/1998, [Ko03] 146).

⋆ P. S. Bullen, The power means, [Buln], 175-265.


P. S. Bullen, D. S. Mitrinović, P. M. Vasić, The power means, [B-mv], 132-214.
G. H. Hardy, J. E. Littlewood, G. Polya, General properties of the means, [H-52], 26-30.
L. Kurlandczyk, Nierówności pomiędzy średnimi potęgowymi, [Ko00], 142-146.
Hojoo Lee, Power means, [LeH2] 45-46.
D. S. Mitrinović, Nierówności dla średnich rzędu r, [Mitr], [Mit2] 43-45.
D. S. Mitrinović, J. E. Pećarić, A. M. Finc, Some recent results involving means, [M-pf], 21-63.
D. S. Mitrinović, P. M. Vasić, Means, [MiV], 74-95.
J. M. Steele, The ladder of power means, [Stee] 120-134.
Nierówności 2. Klasyczne nierówności 25

oooooooooooooooooooooooooooooooooooooooooooooooooooooooooooooooooooooo
2.5 Nierówność Bernoulliego
oooooooooooooooooooooooooooooooooooooooooooooooooooooooooooooooooooooo
Nierównością Bernoulliego nazywa się każdą z następujących nierówności.

2.5.1. Jeśli x, α są liczbami rzeczywistymi, to

(1 + x)α 6 1 + αx , dla 0 < α < 1, x > −1.

(1 + x)α > 1 + αx , dla α > 1, x > −1.


W obu nierównościach równość zachodzi wtedy i tylko wtedy, gdy x = 0.

Gdy α = n jest liczbą naturalną i x > 0, nierówność Bernoulliego (1 + x)n 6 1 + nx


wynika natychmiast z dwumianu Newtona:
! !
n n n 2
(1 + x) = 1 + x+ x + · · · > 1 + x.
1 2

Stąd wynikają następne nierówności, które również są nazywane nierównościami Bernoulliego.


n(n − 1) 2
2.5.2. (1 + x)n > 1 + nx + x , dla x > 0, n > 0.
2
n(n − 1) 2 n(n − 2)(n − 2) 3
2.5.3. (1 + x)n > 1 + nx + x + x , dla x > −1, n > 0. ([JeL]).
2 6

W dowodach podanych niżej nierówności wykorzystuje się nierówności Bernoulliego.


 n
1
2.5.4. 1+ 6 3, dla n > 1.
n
D. ([JeL]). Korzystamy z nierówności 2.5.3:
 −n  n
1 1 n n(n − 1) n(n − 1)(n − 2)
1+ = 1− >1− + −
n n=1 n + 1 2(n + 1)2 6(n + 1)3
2n3 + 9n2 + 7n + 6 2n3 + 6n2 + 6n + 2 1
= > = ,
6(n + 1)3 6(n + 1)3 3
 n
1
a zatem 1+ 6 3. ⊠
n

√ q
2.5.5. n
n61+ n/2, dla n > 1.

D. ([JeL]). Przyjmijmy n
n = 1 + x. Wówczas oczywiście x > 0. Stosując nierówność 2.5.2
otrzymujemy
n(n − 1) 2 n(n − 1) 2
n = (1 + x)n > 1 + nx + x >1+ x ,
2 2
√ p
skąd x2 6 2/n, a zatem n n = 1 + x 6 1 + 2/n. ⊠
26 Nierówności 2. Klasyczne nierówności

!n
n+1 n2 n
2.5.6. 6 6 , dla n > 2.
n n2 − 1 n−1

D. ([JeL]). Korzystamy z nierówności 2.5.1:


 n  n
n2 1 n n n+1
= 1+ >1+ >1+ 2 = ,
n −1
2 n −1
2 n2 −1 n n

 −n  n
n2 1 n n−1
= 1− >1− = ,
n2 − 1 n2 n 2 n
 n
n+1 n2 n
skąd 6 6 .⊠
n n −1
2 n−1

⋆ J. Górnicki, Nierówność Bernoulliego, [Gorn], 122-125.


L. Kurlandczyk, Nierówność Bernoulliego, [Ko00], 135-138.
D. S. Mitrinović, Nierówność Bernoulliego, [Mitr], [Mit2] 31-33.
D. S. Mitrinović, J. E. Pećarić, A. M. Finc, Bernoulli’s inequality, [M-pf], 65-81.
D. S. Mitrinović, P. M. Vasić, Bernoulli’s inequality and its generalizations, [MiV], 34-36.
oooooooooooooooooooooooooooooooooooooooooooooooooooooooooooooooooooooo
2.6 Nierówność Cauchy’ego
oooooooooooooooooooooooooooooooooooooooooooooooooooooooooooooooooooooo

2.6.1 (Cauchy). Dla dowolnych liczb rzeczywistych x1 , . . . , xn , y1 , . . . , yn zachodzi nierów-


ność
 2   
x1 y1 + · · · + xn yn 6 x21 + ··· + x2n y12 + ··· + yn2 .

Jeśli wszystkie liczby x1 , . . . , xn są różne od zera, to równość zachodzi wtedy i tylko wtedy, gdy
y1 y2 yn
= = ··· = .
x1 x2 xn

Nierówność 2.6.1 nazywa się nierównością Cauchy’ego lub nierównością Cauchy’ego-Bu-


niakowskiego lub nierównością Cauchy’ego-Schwarza lub nierównością Cauchy’ego-Buniakow-
skiego-Schwarza.
Znanych jest wiele różnych dowodów nierówności 2.6.1.
n
X n
X n
X
D. (I). Oznaczmy A = x2i , B = xi yi , C = yi2 . Należy udowodnić, że B 2 6 AC.
i=1 i=1 i=1
n
X
Startujemy od oczywistej nierówności (xi − tyi )2 > 0, zachodzącej dla każdej liczby rzeczywistej t.
i=1
Po przekształceniu nierówność ta ma postać

Ct2 − 2Bt + A > 0.

To oznacza, że ∆ = 4B 2 − 4AC 6 0, czyli B 2 6 AC. ⊠


Nierówności 2. Klasyczne nierówności 27

v v
u n u n
uX uX
D. (II). Niech A = t x2i , B = t yi2 . Podstawiając w nierówności x2 + y 2 > 2xy liczby
i=1 i=1
x = xi /A, y = yi /B otrzymujemy  
xi yi 1 x2i yi2
6 + ,
AB 2 A2 B2
dla i = 1, . . . , n. Sumując te nierówności stronami mamy
Pn  Pn Pn 
i=1 xi yi 1 2
i=1 xi
2
i=1 yi
6 + = 1,
AB 2 A2 B2
n
X
czyli xi yi 6 AB. ⊠
i=1

D. (III). ([Ko00], z nierówności Jensena). Ponieważ


n
X n
X n
X
xi yi 6 |xi yi | = |xi ||yi |,
i=1 i=1 i=1

n
X
wystarczy udowodnić to tylko dla dodatnich liczb x1 , . . . , xn , y1 , . . . , yn . Niech ai = xi /yi , B = yi2 ,
i=1
λi = yi2 /B, dla i = 1, . . . , n. Wtedy λ1 , . . . , λn są liczbami dodatnimi i ich suma jest równa 1. Z
nierówności Jensena 1.1.8, dla wypukłej funkcji f (x) = x2 , mamy:
n
!2 n
!2 n
!
X X X
2
xi yi = λi ai B = f λi ai B 2
i=1 i=1 i=1
n
! n
!
X X
2
6 λi f (ai ) B = x2i B
i=1 i=1
n
! n
!
X X
= x2i yi2 ,
i=1 i=1

n
!2 n
! n
!
X X X
czyli xi yi 6 x2i yi2 .⊠
i=1 i=1 i=1

Z nierówności 2.6.1 wynikają następujące znane nierówności.

2.6.2. Dla dowolnych nieujemnych liczb rzeczywistych a1 , . . . , an , b1 , . . . , bn :


q q 2
a1 b1 + · · · + an bn 6 (a1 + · · · + an ) (b1 + · · · + bn )
√ √ √ √
D. Podstawiamy: x1 = a1 , . . . , xn = an , y1 = b1 , . . . , yn = bn i korzystamy z
nierówności 2.6.1. ⊠

2.6.3 (Nierówność trójkąta). Dla dowolnych liczb rzeczywistych x1 , . . . , xn , y1 , . . . , yn :


q q q
(x1 + y1 )2 + · · · + (xn + yn )2 6 x21 + · · · + x2n + y12 + · · · + yn2 .
28 Nierówności 2. Klasyczne nierówności

P P P P P pP pP P
D. (xi + yi )2 = x2i + 2 xi yi + yi2 6 x2i + 2 x2i · yi2 + yi2
 pP pP  2
= x2i + yi2 .
i stąd wynika teza. Wykorzystaliśmy nierówność 2.6.1. ⊠

2.6.4 (Nierówność trójkąta). Dla dowlnych liczb rzeczywistych x1 , . . . , xn , y1 , . . . , yn , oraz


z1 , . . . , zn , zachodzi nierówność
p p p
(x1 − z1 )2 + · · · + (xn − zn )2 6 (x1 − y1 )2 + · · · + (xn − yn )2 + (y1 − z1 )2 + · · · + (yn − zn )2 .

D. W nierówności 2.6.3 wstawiamy zamiast xi , yi odpowiednio xi − yi , oraz yi − zi . ⊠

2.6.5. Dla dowolnych liczb rzeczywistych a1 , . . . , an :


 2   
a31 + · · · + a3n 6 a21 + · · · + a2n a41 + · · · + a4n

D. W nierówności 2.6.1 wstawiamy: x1 = a1 , . . . , xn = an , y1 = a21 , . . . , yn = a2n . ⊠


X ∞
X ∞
X
2.6.6. Jeśli szeregi a2n oraz b2n są zbieżne, to zbieżny jest również szereg |an bn |.
n=1 n=1 n=1
Ponadto, v v
∞ u∞ u∞
X uX uX
|an bn | 6 t an · t
2 b2n . ([Stee] 4-5).
n=1 n=1 n=1

1 4 1 16
2.6.7. + + > , dla a, b, c > 0.
a b c a+b+c
D. Wykorzystujemy nierówność 2.6.2:
  
1 4 1 √ √ √ 2
(a + b + c) + + > 1 + 4 + 1 = 42 = 16. ⊠
a b c

1 1 4 16 64
2.6.8. + + + > . ([Fom] 47/88, [Mild]).
a b c d a+b+c+d
D. Wykorzystujemy nierówność 2.6.2:
  
1 1 4 16 √ √ √ √ 2
(a + b + c + d) + + + > 1 + 1 + 4 + 16 = 82 = 64. ⊠
a b c d

2.6.9. Niech f (x) będzie wielomianem o nieujemnych współczynnikach. Jeśli f (1) > 1, to
 
1 1
f >
x f (x)
dla x > 0. Innymi słowy, jeśli powyższa nierówność zachodzi dla x = 1, to zachodzi dla
wszystkich x > 0. ([Mild]).
Nierówności 2. Klasyczne nierówności 29

D. ([Mild]). Niech f (x) = an xn +an−1 xn−1 +· · ·+a1 x+a0 . Korzystając z nierówności Cauchy’ego
mamy:
1
  
f (x)f x = an xn + an−1 xn−1 + · · · + a1 x + a0 an x1n + · · · + a1 x1 + a1
2
> (an + · · · + a1 + a0 ) = f (1)2 > 1
i stąd wynika teza. ⊠
Nierówność Cauchy’ego ma następujące uogólnienie dla liczb zespolonych.

2.6.10 (Cauchy). Dla dowolnych liczb zespolonych x1 , . . . , xn , y1 , . . . , yn zachodzi nierówność


  
|x1 y1 + · · · + xn yn |2 6 |x1 |2 + · · · + |xn |2 |y1 |2 + · · · + |yn |2 . ([Khr3]).

⋆ E. F. Beckenbach, R. Bellman, Nierówność Cauchy’ego, [BeB] 10-12.


W. K. Cmyszlajew, Zastosowanie nierówności Buniakowskiego-Cauchy do rozwiązywania pewnych
zadań, [Kw] 1/1972 33-35.
L. Kurlandczyk, Nierówność Cauchy’ego, [Ko00], 146-151.
D. S. Mitrinović, Nierówność Cauchy’ego, [Mitr], [Mit2] 36-38.
D. S. Mitrinović, J. E. Pećarić, A. M. Finc, Cauchy’s and related inequalities, [M-pf], 83-98.
D. S. Mitrinović, P. M. Vasić, Cauchy’s inequality, [MiV], 30-32.
D. S. Mitrinović, P. M. Vasić, Cauchy’s and related inequalities, [MiV], 41-44.
J. M. Steele, Starting with Cauchy, [Stee] 1-18.
oooooooooooooooooooooooooooooooooooooooooooooooooooooooooooooooooooooo
2.7 Różne klasyczne nierówności
oooooooooooooooooooooooooooooooooooooooooooooooooooooooooooooooooooooo
W tym podrozdziale przedstawimy pewne stare i dobrze znane nierówności. Każda z
tych nierówności ma wiele przeróżnych elementarnych dowodów. Dowodami nie będziemy się
zajmować. Istnieje obszerna literatura na ten temat; są liczne książki, artykuły i opracowania.
Różne dowody przedstawionych tu nierówności znajdziemy, na przykład, w: [H-52], [Mitr],
[Mit2], [M-pf], [MiV], [Ko00], [BeB], [LeH2], [Mild].
Pewnym uogólnieniem nierówności Cauchy’ego 2.6.1 jest następująca nierówność Höldera.

1 1
2.7.1. Jeśli p > 0, q > 0 są liczbami takimi, że + = 1, to
p q
 1/p  1/q
x1 y1 + · · · + xn yn 6 xp1 + ··· + xpn y1q + ··· + ynq ,
dla dowolnych nieujemnych liczb x1 , . . . , xn , y1 , . . . , yn .

Jeśli, w szczególności, p = q = 2, to nierówność Höldera staje się nierównością Cau-


chy’ego. Stosując odpowiednio dwa razy nierówność Höldera, łatwo można otrzymać nastę-
pujące uogólnienie nierówności Cauchy’ego.

2.7.2. Dla dowolnych dodatnich liczb rzeczywistych x1 , . . . , xn , y1 , . . . , yn , z1 , . . . , zn zachodzi


nierówność

(x1 y1 z1 + · · · + xn yn zn )3 6 (x31 + · · · + x3n )(y13 + · · · + yn3 )(z13 + · · · + zn3 ).


30 Nierówności 2. Klasyczne nierówności

Ta nierówność jest szczególnym przypadkiem następującej ogólniejszej nierówności, która


również wynika z nierówności Höldera.

2.7.3. Niech x1i , x2i , . . . , xsi , dla i = 1, 2, . . . , n, będą liczbami dodatnimi i niech p1 , . . . , ps
będą liczbami dodatnimi, których suma odwrotności jest równa 1. Wtedy:

n s n
!1/pk
X Y X pk
x1i x2i · · · xsi 6 xki .
i=1 k=1 i=1

Następne dwie nierówności nazywane są są nierównościami Minkowskiego.

2.7.4. Jeśli x1 , . . . , xn , y1 , . . . , yn są liczbami dodatnimi, to


√ √ q
n
n
x1 x2 · · · xn + n
y1 y2 · · · yn 6 (x1 + y1 )(x2 + y2 ) · · · (xn + yn ).
Równość zachodzi wtedy i tylko wtedy, gdy
y1 y2 yn
= = ··· = .
x1 x2 xn
√ √ r r
n
x1 · · · xn + n y1 · · · yn x1 xn y1 yn
D. ([Stee] 234). n
p = n
··· + n ···
(x1 + y1 ) · · · (xn + yn ) x 1 + y 1 xn + y n x1 + y 1 xn + yn
   
1 x1 xn 1 y1 yn
6 + ··· + + + ··· + = 1.
n x1 + y1 x1 + yn n x1 + y1 x1 + yn
Wykorzystaliśmy dwa razy nierówność pomiędzy średnimi. ⊠

2.7.5. Jeśli x1 , . . . , xn , y1 , . . . , yn są liczbami dodatnimi i p > 1, to

n
!1/p n
!1/p n
!1/p
X X p
X p
p
(xi + yi ) 6 xi + yi .
i=1 i=1 i=1

Kolejne dwie nierówności nazywane są nierównościami Czebyszewa.

2.7.6. Jeśli x1 6 x2 6 · · · 6 xn , y1 6 y2 6 · · · 6 yn , to

(x1 + · · · + xn )(y1 + · · · + yn ) 6 n(x1 y1 + · · · + xn yn ).

Jeśli x1 6 x2 6 · · · 6 xn , y1 > y2 > · · · > yn , to

(x1 + · · · + xn )(y1 + · · · + yn ) > n(x1 y1 + · · · + xn yn ).

Następna nierówność dotyczy dwóch skończonych ciągów jednakowo uporządkowanych.


Jej angielska popularna nazwa, to rearrangement inequality.
Nierówności 2. Klasyczne nierówności 31

2.7.7. Niech x1 , . . . , xn , y1 , . . . , yn będą dodatnimi liczbami rzeczywistymi takimi, że

x1 > x2 > · · · > xn oraz y 1 > y2 > · · · > y n .

Wtedy, dla dowolnej permutacji τ zbioru {1, 2, . . . , n}, zachodzą nierówności:

x1 y1 + · · · + xn yn > x1 yτ (1) + · · · + xn yτ (n) > x1 yn + x2 yn−1 + · · · + xn y1 .

Zanotujmy jeszcze kilka innych klasycznych nierówności.


a c a a+c c
2.7.8. Jeśli a, b, c, d > 0, < , to < < .
b d b b+d d
a1 a2 a3 a1 a1 + a2 + a3 a3
2.7.9. Jeśli a1 , a2 , a3 , b1 , b2 , b3 > 0, < < , to < < . ([Ismj] J11.2).
b1 b2 b3 b1 b1 + b2 + b3 b3

2.7.10. Jeśli a1 , . . . , an są dowolnymi liczbami rzeczywistymi oraz b1 , . . . , bn są liczbami do-


datnimi, to zachodzi nierówność
   
a1 an a1 + · · · + an a1 an
min ,..., 6 6 max ,..., .
b1 bn b1 + · · · + b n b1 bn
([Mit2] s.24, [Stee] 82, 243).

W 1960 roku Z. Opial udowodnił następującą nierówność, zwaną dzisiaj nierównością


Opiala.

2.7.11. Jeśli f : [0, b] → R jest funkcją klasy C 1 taką, że f (x) > 0 w przedziale (0, b) oraz
f (0) = f (b) = 0, to
Zb Zb
′ b
|f (x)f (x)|dx 6 |f ′ (x)|2 dx.
4
0 0

⋆ I. Ashiba, M. Nihei, On a unified method of solving some inequalities, [MG] 512(2004) 292-298.
P. S. Bullen, A Dictionary of Inequalities, [Bull].
P. Hajłasz, O pewnej metodzie dowodzenia nierówności, [Dlt] 3/1986 12-15.
A. I. Khrabrov, Classical inequalities and its reversals, (Russian). Appendix to: St Petersburg
mathematical olympiad, 2000. Izdat. St Petersburg State Univ., St Petersburg, 2000.
A. I. Khrabrov, Proofs of inequalities using quasilinearization method, (Russian). Appendix to: St
Petersburg mathematical olympiad, 1999. Izdat. St Petersburg State Univ., St Petersburg, 1999.
Hojoo Lee, Dowodzenie nierówności, [Dlt], 6/2002, 9-11.
V. Levin, Parabola i nierówności, [Kw] 4/1976 14-18.
G. Łukaszewicz Wokół nierówności Younga, [Dlt] 7/1995 4-7.
B. G. Pachpatte, Opial–type inequalities, [Pach] 263-380.
J. M. Steele, Hölder inequality, [Stee] 135-155.
M. Siebriuk, Wariacje na temat klasycznych nierówności, [Kw] 5/1979 18-21.
32 Nierówności 2. Klasyczne nierówności
⊡⊡⊡⊡⊡⊡⊡⊡⊡⊡⊡⊡⊡⊡⊡⊡⊡⊡⊡⊡⊡⊡⊡⊡⊡⊡⊡⊡⊡⊡⊡⊡⊡⊡⊡⊡⊡⊡⊡⊡⊡⊡⊡⊡⊡⊡⊡⊡⊡⊡⊡⊡
3 Twierdzenie Muirheada
⊡⊡⊡⊡⊡⊡⊡⊡⊡⊡⊡⊡⊡⊡⊡⊡⊡⊡⊡⊡⊡⊡⊡⊡⊡⊡⊡⊡⊡⊡⊡⊡⊡⊡⊡⊡⊡⊡⊡⊡⊡⊡⊡⊡⊡⊡⊡⊡⊡⊡⊡⊡

Jeśli x, y, z, t są dodatnimi liczbami rzeczywistymi, to:

x2 + y 2 > 2xy,
x5 + y 5 > x3 y 2 + y 3 x2 ,
x2 + y 2 + z 2 > xy + yz + zx,
x3 + y 3 + z 3 > 3xyz,
x2 y 2 + y 2 z 2 + z 2 x2 > x2 yz + xy 2 z + xyz 2 ,
x3 + y 3 + z 3 + t3 > xyz + xyt + xzt + yzt.

Pewne z tych nierówności są prawdziwe nawet dla dowolnych liczb rzeczywistych (nieko-
niecznie dodatnich). Rozważać będziemy jednak tylko liczby dodatnie.
Pokażemy, że wszystkie powyższe nierówności są szczególnymi przypadkami pewnego
twierdzenia udowodnionego w 1903 roku przez R. E. Muirheada Przed wysłowieniem tego
twierdzenia wprowadzimy najpierw kilka nowych pojęć i oznaczeń.
oooooooooooooooooooooooooooooooooooooooooooooooooooooooooooooooooooooo
3.1 Podziały
oooooooooooooooooooooooooooooooooooooooooooooooooooooooooooooooooooooo
Podziałem długości k liczby naturalnej n nazywamy każdy ciąg α = (α1 , . . . , αk ) nieujem-
nych liczb całkowitych spełniających następujące dwa warunki:
(1) α1 > α2 > · · · > αk ,
(2) α1 + α2 + · · · + αk = n.
Zbiór wszystkich podziałów długości k liczby n oznaczać będziemy przez P(n, k). W szcze-
gólności zbiór P(4, 3) składa się z 4 elementów:

(4, 0, 0), (3, 1, 0), (2, 2, 0), (2, 1, 1).

Natomiast zbiór P(7, 4) ma 11 elementów:

(7, 0, 0, 0), (6, 1, 0, 0), (5, 2, 0, 0), (5, 1, 1, 0), (4, 3, 0, 0), (4, 2, 1, 0),
(4, 1, 1, 1), (3, 3, 1, 0), (3, 2, 2, 0), (3, 2, 1, 1), (2, 2, 2, 1).

Jeśli n < 10, to elementy zbioru P(n, k) zapisywać będziemy bez nawiasów i bez przecinków.
Elementami zbioru P(4, 3) są więc podziały: 400, 310, 220, 211, a elementami zbioru P(7, 4)
podziały:

7000, 6100, 5200, 5110, 4300, 4210 4111, 3310, 3220, 3211, 2221.

33
34 Nierówności 3. Twierdzenie Muirheada

Załóżmy, że α = (α1 , . . . , αk ), β = (β1 , . . . , βk ) są podziałami należącymi do zbioru


P(n, k). Mówić będziemy, że podział α jest większy lub równy od podziału β, co zapisywać
będziemy jako ”α > β”, jeśli:

α1 > β1 ,
α1 + α2 > β1 + β2 ,
α1 + α2 + α3 > β1 + β2 + β3 ,
..
.
α1 + α2 + α3 + · · · + αk−1 > β1 + β2 + β3 + · · · + βk−1 ,
α1 + α2 + α3 + · · · + αk−1 + αk > β1 + β2 + β3 + · · · + βk−1 + βk .

Ta ostatnia nierówność jest oczywiście równością. Spójrzmy na przykłady. Ciągi 421 i 322
są podziałami długości 3 liczby 7. Zachodzi nierówność 421 > 322, gdyż:

4 > 3,
4 + 2 > 3 + 2,
4 + 2 + 1 = 3 + 2 + 2.

W ten sam sposób sprawdzamy, że:

720 > 522, 521 > 431, 52100 > 43100, 22220 > 22211.

Łatwo udowodnić:

3.1.1. Niech α, β, γ będą podziałami należącymi do zbioru P(n, k). Wtedy:


(1) α > α;
(2) jeśli α > β i β > α, to α = β;
(3) jeśli α > β i β > γ, to α > γ.

W zbiorze P(3, 3) mamy elementy 300, 210, 111 i zachodzi: 300 > 210 > 111. Wszystkie
elementy zbioru P(4, 3) uporządkowane są następująco: 400 > 310 > 220 > 211. Podobnie
jest w zbiorze P(5, 3):
500 > 410 > 320 > 311 > 221.
Widzimy tutaj, że każde dwa elementy α, β zbioru P(n, k) są w relacji: albo α > β albo β > α.
Na ogół tak jednak nie musi być. Elementy α = 411 i β = 330 zbioru P(6, 3) nie są w żadnej
relacji; nie jest prawdą, że α > β i nie jest prawdą, że β > α.
Niech α = (α1 , . . . , αk ), β = (β1 , . . . , βk ) będą podziałmi należącymi do zbioru P(n, k).
Jeśli podziały te są różne, to istnieje co najmniej jedno i ∈ {1, 2, . . . , k} takie, że αi 6=
βi . Oznaczmy przez A(α, β) zbiór tych wszystkich indeksów i ∈ {1, 2, . . . , k}, dla których
zachodzi nierówność αi 6= βi . Dla przykładu, jeśli α = (4, 3, 3, 2, 1), β = (5, 4, 3, 1, 0) są
podziałami liczby 13, to
A(α, β) = {1, 2, 4, 5},
gdyż podziały te różnią się na miejscach 1, 2, 4 i 5.
Nierówności 3. Twierdzenie Muirheada 35

Jest jasne, że zbiór A(α, β) jest pusty wtedy i tylko wtedy, gdy α = β. Załóżmy, że α 6= β.
Wtedy αi 6= βi , dla pewnego i ∈ {1, . . . , k}. Ale

α1 + α2 + · · · + αk = n = β1 + β2 + · · · + βk ,

więc musi istnieć jeszcze jeden indeks j ∈ {1, . . . , k}, różny od i, dla którego zachodzi nie-
równość αj 6= βj . Mamy zatem następujące oczywiste stwierdzenie.

3.1.2. Jeśli α i β są różnymi podziałami należącymi do zbioru P(n, k), to zbiór A(α, β) ma
co najmniej dwa różne elementy.

Mówić będziemy, że α jest ostro większe od β i pisać α > β, jeśli α > β i α 6= β. Ponadto,
pisać będziemy α ⊲ β w przypadku, gdy α > β i pomiędzy α i β nie ma żadnych innych
podziałów, tzn. gdy nie istnieje żaden taki podział γ ∈ P(n, k), że α > γ > β. W tej sytuacji
mówić będziemy, że podziały α i β są sąsiednie. Łatwo sprawdzić, że podziały (3, 2, 1) i
(2, 2, 2), liczby 6, są sąsiednie. Mamy więc: (3, 2, 1) ⊲ (2, 2, 2). Inny przykład: (5, 4, 3, 2, 1) ⊲
(4, 4, 3, 3, 1).
Ponieważ zbiór P(n, k) jest skończony, więc mamy następujące stwierdzenie.

3.1.3. Niech α, β ∈ P(n, k), α > β. Istnieją wtedy podziały γ (0) , γ (1) , . . . , γ (s) , należące do
zbioru P(n, k) takie, że

α = γ (0) ⊲ γ (1) ⊲ γ (2) ⊲ · · · ⊲ γ (s) = β.

Liczba naturalna s, pojawiająca się w powyższym stwierdzeniu, nie musi być wyznaczona
jednoznacznie. Spójrzmy na następujący przykład.

3.1.4. Rozważmy podziały α = (4, 2, 1, 0) oraz β = (3, 2, 1, 1), należące do zbioru P(7, 4).
Zachodzi nierówność α > β i mamy:

α = (4, 2, 1, 0) ⊲ (4, 1, 1, 1) ⊲ (3, 2, 1, 1) = β,


α = (4, 2, 1, 0) ⊲ (3, 3, 1, 0) ⊲ (3, 2, 2, 0) ⊲ (3, 2, 1, 0) = β.

Są to dwa ciągi różnych długości.

Zanotujmy kilka informacji o podziałach sąsiednich.

3.1.5. Jeśli α ⊲ β, to zbiór A(α, β) ma dokładnie dwa elementy.

D. Oznaczmy: A = A(α, β). Wiemy (patrz 3.1.2), że |A| > 2. Przypuśćmy, że |A| > 3. Niech p
będzie najmniejszym elementem w zbiorze A. Wtedy αp 6= βp oraz, w przypadku gdy p > 1, wszystkie
liczby α1 , . . . , αp−1 są odpowiednio równe liczbom β1 , . . . , βp−1 . Z nierówności α > β wynika, że
αp > βp .
Wśród liczb βp+1 , βp+2 , . . . , βk co najmniej jedna musi być większa od zera. Gdyby bowiem wszyst-
kie te liczby były równe zero, mielibyśmy sprzeczność:
k
X k
X
n= βi = β1 + · · · + βp < αi = n.
i=1 i=1
36 Nierówności 3. Twierdzenie Muirheada

Niech q będzie największą liczbą ze zbioru {p + 1, p + 2. . . . , k} taką, że βq > 0.


Rozpatrzmy ciąg γ = (γ1 , . . . , γk ), nieujemnych liczb całkowitych zdefiniowanych następująco:

 βi ,
 gdy i 6∈ {p, q},
γi = βp + 1, gdy i = p,


βq − 1, gdy i = q.
Z powyższych założeń wynika, że γ jest podziałem należącym do zbioru P(n, k) i to takim, że α > γ >
β. Ponadto, |A(γ, β)| = 2. Ponieważ α ⊲ β, więc nie może zachodzić nierówność α > γ. Zatem γ = α.
Otrzymaliśmy więc sprzeczność: 3 6 |A| = |A(α, β)| = |A(γ, β)| = 2. ⊠

3.1.6. Niech α = (α1 , . . . , αk ), β = (β1 , . . . , βk ) będą podziałmi należącymi do zbioru P(n, k).
Jeśli α ⊲ β, to w zbiorze {1, 2, . . . , k} istnieją dwie liczby p < q takie, że

 βi ,

 gdy i 6∈ {p, q},
αi = β + 1, gdy i = p,
p


 β − 1, gdy i = q.
q

D. Już wiemy (patrz 3.1.5), że zbiór A(α, β) ma dokładnie dwa elementy. Istnieją więc dwie liczby
p < q, należące do zbioru {1, . . . , k} takie, że αp 6= βp , αq 6= βq oraz αi = βi dla i 6∈ {p, q}. Ponieważ
α > β oraz
Xk X k
αi = n = βi ,
i=1 i=1
więc αp = βp +a, αq = βq −a, gdzie a > 0 jest pewną liczbą naturalną. Wystarczy więc tylko pokazać,
że a = 1.
Przypuśćmy, że a > 2 i niech γ = (γ1 , . . . , γk ) będzie ciągiem nieujemnych liczb całkowitych takich,
że γp = βp + 1, γq = βq − 1 oraz γi = αi = βi dla i 6∈ {p, q}. Wtedy γ jest podziałem, należącym do
zbioru P(n, k) takim, że α > γ > β. Jest to sprzeczne z tym, że podziały α i β są sąsiednie. Zatem
a = 1 i to kończy dowód. ⊠
U. Stwierdzenie odwrotne do stwierdzenia 3.1.6 nie musi zachodzić. Rozważmy podziały α =
(4, 2, 1, 0) i β = (3, 2, 1, 1) należące do zbioru P(7, 4). Mamy tu: α > β, A(α, β) = {1, 4} (czyli p = 1,
q = 4), α1 = β1 + 1, α4 = β4 − 1 oraz α2 = β2 , α3 = β3 . Podziały te nie są jednak sąsiednie (patrz
3.1.4). ⊠
oooooooooooooooooooooooooooooooooooooooooooooooooooooooooooooooooooooo
3.2 Wielomian symetryczny stowarzyszony z podziałem
oooooooooooooooooooooooooooooooooooooooooooooooooooooooooooooooooooooo
Jeżeli k jest ustaloną liczbą naturalną, to przez Sk oznaczać będziemy zbiór wszystkich
permutacji zbioru {1, 2, . . . , k}. Przypomnijmy, że zbiór Sk ma k! elementów.
Niech f (x1 , . . . , xk ) będzie wielomianem zmiennych x1 , . . . , xk . Mówimy, że wielomian ten
jest symetryczny, jeśli dla każdej permutacji σ należącej do zbioru Sk zachodzi równość
   
f xσ(1) , xσ(2) , . . . , xσ(k) = f x1 , x2 , . . . , xk .
Załóżmy, że mamy tylko dwie zmienne x1 i x2 (tzn. k = 2). Zmienne te oznaczmy odpowiednio
przez x i y. W tym przypadku wielomian f (x, y) jest symetryczny dokładnie wtedy, gdy
f (y, x) = f (x, y).
Nierówności 3. Twierdzenie Muirheada 37

W szczególności wielomiany xy, x5 +y 5 , x3 +y 3 −13xy są symetryczne. Natomiast wielomiany


x + 4y, x2 + y 3 , xy + 5y 2 nie są symetryczne.
Rozważmy teraz trzy zmienne x = x1 , y = x2 , z = x3 (tzn. k = 3). W tym przypadku
wielomian f (x, y, z) jest symetryczny dokładnie wtedy, gdy

f (x, y, z) = f (x, z, y) = f (y, x, z) = f (y, z, x) = f (z, x, y) = f (z, y, x).

Wielomiany xyz, x9 + y 9 + z 9 , xy + yz + zx, 5xyz − 12x2 − 12y 2 − 12z 2 są symetryczne.


Wielomiany xyz 2 , x + 5y + z, x2 + y 2 + z nie są symetryczne.

Niech α = (α1 , . . . , αk ) będzie podziałem długości k liczby naturalnej n. Oznaczmy przez


Aα jednomian zmiennych x1 , . . . , xk zdefiniowany następująco:

Aα = Aα (x1 , . . . , xk ) = xα1 1 xα2 2 · · · xαk k .


Przykłady: A21 (x, y) = x2 y, A432 (x, y, z) = x4 y 3 z 2 , A5100 (x, y, z, t) = x5 y.

W dalszym ciągu istotną rolę odgrywać będzie wielomian symetryczny, zmiennych x1 ,


. . . , xk , którego oznaczać będziemy przez Tα lub Tα (x1 , . . . , xk ). Wielomian ten definiujemy
następująco:
X
Tα = Tα (x1 , . . . , xk ) = Aα (xσ(1) , xσ(2) , . . . , xσ(k) ),
σ∈Sk

gdzie Sk jest zbiorem wszystkich permutacji zbioru {1, 2, . . . , k}. W szczególności, dla dwóch
zmiennych x i y, mamy:
Tα (x, y) = Aα (x, y) + Aα (y, x),
natomiast dla trzech zmiennych x, y, z:

Tα (x, y, z) = Aα (x, y, z) + Aα (x, z, y) + Aα (y, x, z) + Aα (y, z, x) + Aα (z, x, y) + Aα (z, y, x).

3.2.1. Przykłady:

T32 (x, y) = x3 y 2 + y 3 x2 ,
T321 (x, y, z) = x3 y 2 z + x3 z 2 y + y 3 x2 z + y 3 z 2 x + z 3 x2 y + z 3 y 2 x,
T3300 (x, y, z, t) = 4x3 y 3 + 4x3 z 3 + 4x3 t3 + y 3 z 3 + 4y 3 t3 + 4z 3 t3 ,
T4110 (x, y, z, t) = 2x4 (yz + yt + zt) + 2y 4 (xz + xt + zt)+
2z 4 (xy + xt + yt) + 2t4 (xy + xz + yz),
T50000 (x, y, z, t, u) = 24(x5 + y 5 + z 5 + t5 + u5 ),
T11111 (x, y, z, t, u) = 120xyztu.
38 Nierówności 3. Twierdzenie Muirheada

W dalszym ciągu wykorzystamy następujące dwa lematy.

3.2.2. Jeśli p > q są liczbami naturalnymi, to dla dowolnych nieujemnych liczb rzeczywistych
x, y zachodzi nierówność

xp+1 y q−1 + xp−1 y q+1 > xp y q + xq y p .

D. Oznaczmy przez r liczbę naturalną p − q + 1. Mamy wtedy:



xp+1 y q−1 + xq−1 y p+1 − xp y q + xq y p = (x − y) xp−q+1 − y p−q+1 xq−1 y q−1
= (x − y) (xr − y r ) xq−1 y q−1

= (x − y)2 xr−1 + xr−1 y + · · · + xy r−2 + y r−1 xq−1 y q−1 .

Ostatnia liczba jest oczywiście nieujemna. ⊠

3.2.3. Niech α, β będą podziałami należącymi do zbioru P(n, k). Jeśli α⊲β (tzn. jeśli α > β i
podziały te są sąsiednie), to dla dowolnych nieujemnych liczb rzeczywistych r1 , . . . , rk zachodzi
nierówność
Tα (r1 , . . . , rk ) > Tβ (r1 , . . . , rk ).

D. Niech α = (α1 , . . . , αk ), β = (β1 , . . . , βk ). Załóżmy, że α ⊲ β. Wiemy (patrz 3.1.6), że wtedy


w zbiorze {1, 2, . . . , k} istnieją dwie liczby p < q takie, że αp = βp + 1, αq = βq − 1 oraz αi = βi dla
wszystkich i 6∈ {p, q}. Spójrzmy na wielomiany Tα i Tβ . Grupując odpowiednio wszystkie jednomiany
występujące w tych wielomianach widzimy, że
X
T α − Tβ = wij fij ,
i<j

gdzie wszystkie elementy wij są jednomianami, a elementy fij są wielomianami zmiennych xi , xj


postaci
fij = xp+1
i xq−1
j + xq−1
i xp+1
j − xpi xqj − xqi xpj .

Załóżmy, że r1 , . . . , rk są dowolnymi nieujemnymi liczbami rzeczywistymi. Wtedy każda liczba postaci


wij (r1 , . . . , rk ) jest nieujemna oraz (na mocy 3.2.2) każda liczba postaci fij (r1 , . . . , rk ) jest również
nieujemna. Zatem Tα (r1 , . . . , rk ) − Tβ (r1 , . . . , rk ) > 0, czyli Tα (r1 , . . . , rk ) > Tβ (r1 , . . . , rk ). ⊠

oooooooooooooooooooooooooooooooooooooooooooooooooooooooooooooooooooooo
3.3 Twierdzenie Muirheada i jego dowód
oooooooooooooooooooooooooooooooooooooooooooooooooooooooooooooooooooooo
Teraz możemy już wysłowić zapowiedziane wcześniej twierdzenie Muirheada.

3.3.1 (Muirhead 1903). Niech α, β będą podziałami długości k liczby naturalnej n. Następu-
jące dwa warunki są równoważne.
(1) α > β.
(1) Dla dowolnych nieujemnych liczb rzeczywistych x1 , . . . , xk zachodzi nierówność

Tα (x1 , . . . , xk ) > Tβ (x1 , . . . , xk ).


Nierówności 3. Twierdzenie Muirheada 39

Dowód tego twierdzenia nie jest trudny. Można go znaleźć np. w [H-48], [MaO]. Pewne
fragmenty tego dowodu są opisane w [DwJ] lub [Ko03]. Dowód, który tutaj przedstawiamy,
został opracowany na podstawie książki [H-48].
D. (1) ⇒ (2). Załóżmy, że α > β i niech r = (r1 , . . . , rk ) będzie ciągiem nieujemnych liczb
rzeczywistych. Wiemy (patrz 3.1.3), że istnieją podziały γ (0) , γ (1) , . . . , γ (s) , należące do zbioru
P(n, k) takie, że
α = γ (0) ⊲ γ (1) ⊲ γ (2) ⊲ · · · ⊲ γ (s) = β.
Z faktu 3.2.3 wynika, że wtedy Tα (r) > Tγ (1) (r) > · · · > Tγ (s−1) (r) > Tβ (r). Zatem Tα (r) > Tβ (r).
(2) ⇒ (1). Załóżmy, że dla dowolnego ciągu r = (r1 , . . . , rk ), nieujemnych liczb rzeczywistych,
zachodzi nierówność Tα (r) > Tβ (r). Niech R[t] oznacza pieścień wielomianów jednej zmiennej t nad
ciałem R, liczb rzeczywistych.
Ustalmy jedną liczbę naturalną i0 taką, że 1 6 i0 < k. Niech w(t) będzie ciągiem równym
(t, t, . . . , t, 1, 1, . . . , 1); na początku występuje i0 razy zmienna t, a pozostałe wyrazy są równe 1.
Oznaczmy:

f (t) = Tα (w(t)) = Tα (t, t, . . . , t, 1, 1, . . . , 1), g(t) = Tβ (w(t)) = Tβ (t, t, . . . , t, 1, 1, . . . , 1).

Zauważmy, że f (t), g(t) są niezerowymi wielomianami należącymi do pierścienia R[t]. Wszystkie


niezerowe współczynniki tych wielomianów są liczbami większymi od zera (są nawet liczbami na-
turalnymi). Z założenia wynika, że dla każdej nieujemnej liczby rzeczywistej a zachodzi nierówność
f (a) > g(a). Stąd wnioskujemy, że deg f (t) > deg g(t). Ale deg f (t) = α1 + α2 + · · · + αi0 oraz
deg g(t) = β1 + β2 + · · · + βi0 . Mamy więc:

α1 + α2 + · · · + αi0 > β1 + β2 + · · · + βi0 .

Powtarzając to samo dla każdej liczby i0 ze zbioru {1, 2, . . . , k − 1}, otrzymujemy żądaną nierówność
α > β. ⊠

Wykażemy teraz jak z twierdzenia 3.3.1 wynikają nierówności wspomniane na początku


tego rozdziału.

3.3.2. x2 + y 2 > 2xy.

D. Rozpatrzmy podziały 20 i 11. Są to podziały długości 2 liczby 2. Poniewż 20 > 11, więc (na
mocy twierdzenia Muirheada) x2 + y 2 = T20 (x, y) > T11 (x, y) = 2xy. ⊠

3.3.3. x5 + y 5 > x3 y 2 + y 3 x2 .

D. Rozpatrzmy podziały 50 i 32. Są to podziały długości 2 liczby 5. Poniewż 50 > 32, więc (na
mocy twierdzenia Muirheada) x5 + y 5 = T50 (x, y) > T32 (x, y) = x3 y 2 + y 3 x2 . ⊠

3.3.4. x2 + y 2 + z 2 > xy + yz + zx. Dowód. α = 200, β = 110.

3.3.5. x3 + y 3 + z 3 > 3xyz. Dowód. α = 300, β = 111.

3.3.6. x2 y 2 + y 2 z 2 + z 2 x2 > x2 yz + xy 2 z + xyz 2 . Dowód. α = 220, β = 211.

3.3.7. x3 + y 3 + z 3 + t3 > xyz + xyt + xzt + yzt. Dowód. α = 3000, β = 1110.


40 Nierówności 3. Twierdzenie Muirheada

Dla każdej liczby naturalnej n zachodzi nierówność

(n, 0, 0, . . . , 0) > (1, 1, 1, . . . , 1).

Z twierdzenia Muirheada wynika zatem, że

xn1 + xn2 + · · · + xnn > nx1 x2 · · · xn ,

dla dodatnich liczb rzeczywistych x1 , . . . , xn . Zamieniają liczby x1 , . . . , xn odpowiednio licz-


√ √
bami n x1 , . . . , n xn , otrzymujemy nowy dowód twierdzenia o nierówności pomiędzy średnią
arytmetyczną i średnią geometryczną.

3.3.8. Dla dowolnych dodatnich liczb rzeczywistych x1 , . . . , xn , zachodzi nierówność

x1 + x2 + · · · + xn √
> n x1 x2 · · · xn .
n
Równość zachodzi wtedy i tylko wtedy, gdy x1 = x2 = · · · = xn .

3.3.9. Korzystając z twierdzenia Muirheada wykazać, że zachodzą następujące nierówności.


Wszystkie liczby rzeczywiste x, y, z, . . . , x1 , x2 , . . . , występujące w tych nierównościach, są do-
datnie.
(1) x4 + y 4 + z 4 > xyz(x + y + z).
(2) x5 + y 5 + z 5 > xyz(xy + yz + zx).
(3) (x2 + y 2 + z 2 )(x + y + z) > 9xyz.
x3 y3 z3 x2 + y 2 y 2 + z 2 x2 + z 2
(4) + + > + + > x + y + z.
yz xz xy 2z 2x 2y
x y z 3
(5) + + > .
y+z x+z x+y 2
x1 x2 xn n
(6) + + ··· + > .
x2 + x3 + · · · + xn x1 + x3 + · · · + xn x1 + x2 + · · · + xn−1 n−1
(7) 8(x4 + y 4 ) > (x + y)4 .
(8) (x3 − y 3 )2 > (x2 − y 2 )(x4 − y 4 ).
(9) (x + y)(x4 + y 4 ) > (x2 + y 2 )(x3 + y 3 ).
1 1 1 1 16
(10) + + + > .
x+y+z x+y+t x+z+t y+z+t x+y+z+t
x6 + y 6 x + y x2 + y 2 x3 + y 3
(11) > · · .
2 2 2 2
xn+m + y n+m xn + y n xm + y m
(12) > · .
2 2 2
Nierówności 3. Twierdzenie Muirheada 41

⋆ A. Berent, Twierdzenie Muirheada i nierówności symetryczne, [Pmgr] 1991.


P. S. Bullen, Symmetric polynomial means, [Buln], 321-367.
P. S. Bullen, D. S. Mitrinović, P. M. Vasić, Symmetric means, [B-mv], 283-342.
G. H. Hardy, J. E. Littlewood, G. Polya, Muirhead’s theorem, [H-52], 44-48.
L. C. Hin, Muirhead’s inequality, [ME] 11(1)(2006) 1-4.
K. Y. Li, Proofs of Muirhead’s inequality, [ME] 11(1)(2006).
A. W. Marshall, I. Olkin, Symetryzacja funkcji wypukłych i wypukłych w sensie Schura: twierdzenie
Muirheada, [MaO], 92-101.
A. Nowicki, Nierówności symetryczne, Zakopane, 1995.
oooooooooooooooooooooooooooooooooooooooooooooooooooooooooooooooooooooo
3.4 Twierdzenie Muirheada dla podziałów liczb całkowitych
oooooooooooooooooooooooooooooooooooooooooooooooooooooooooooooooooooooo
Do tej pory mówiliśmy o podziałach liczb naturalnych. W analogiczny sposób można
rozapatrywać podziały dowolnej liczby całkowitej. Podziałem długości k liczby całkowitej m
nazywamy każdy ciąg α = (α1 , . . . , αk ) liczb całkowitych spełniających następujące dwa
warunki:
(1) α1 > α1 > · · · > αk ,
(2) α1 + α2 + · · · + αk = m.
Zbiór wszystkich podziałów długości k liczby całkowitej m oznaczać będziemy, tak samo
jak poprzednio, przez P(m, k). Porządek > w zbiorze P(m, k) wprowadzamy tak samo, jak
dla podziałów liczb naturalnych. Jeśli α = (α1 , . . . , αk ), β = (β1 , . . . , βk ) są podziałami
należącymi do zbioru P(m, k), to α > β, jeśli

α1 > β1 , α1 + α2 > β1 + β2 , α1 + α2 + α3 > β1 + β2 + β3 , ....

Przykład: (0, −3) > (−1, −2).


Niech α = (α1 , . . . , αk ) ∈ P(m, k). Oznaczmy przez Aα = Aα (x1 , . . . , xk ) funkcję wymier-
ną xα1 1 xα2 2 · · · xαk k i niech
X
Tα = Tα (x1 , . . . , xk ) = Aα (xσ(1) , xσ(2) , . . . , xσ(k) ).
σ∈Sk

Mamy wówczas:

3.4.1 (Twierdzenie Muirheada dla całkowitych podziałów). Niech α, β będą podziałami dłu-
gości k liczby całkowitej m. Następujące dwa warunki są równoważne.
(1) α > β.
(1) Dla dowolnych dodatnich liczb rzeczywistych x1 , . . . , xk zachodzi nierówność

Tα (x1 , . . . , xk ) > Tβ (x1 , . . . , xk ).

D. Niech α = (α1 , . . . , αk ), β = (β1 , . . . , βk ) i niech r := −1+min(αk , βk ). Oznaczmy αi′ := αi −r,


βi′= βi − r, dla i = 1, . . . , k, i niech α′ = (α1′ , . . . , αk′ ), β ′ = (β1′ , . . . , βk′ ). Wtedy α′ , β ′ są podziałami
należącymi do P(n, k), gdzie n jest liczbą naturalną równą m − kr.
42 Nierówności 3. Twierdzenie Muirheada

Załóżmy, że α > β. Wtedy α′ > β ′ i z twierdzenia Muirheada 3.3.1 (dla podziałów liczb natural-
nych) mamy nierówność
X α′ α′1
X β′ β1′
xσ(1)
1
· · · xσ(1) > xσ(1)
1
· · · xσ(1) ,
σ∈Sn σ∈Sn

zachodzącą dla wszystkich dodatnich liczb rzeczywistych x1 , . . . , xk . Zauważmy, że dla każdej per-
mutacji σ ∈ Sk zachodzi równość (x1 x2 · · · xk )r = xrσ(1) xrσ(2) · · · xrσ(k) . Mnożąc stronami powyższą
nierówność przez dodatnią liczbę (x1 x2 · · · xk )r otrzymujemy: Tα (x1 , . . . , xk ) > Tβ (x1 , . . . , xk ).
Udowodniliśmy zatem, że z (1) wynika (2). Analogicznie dowodzimy implikację w przeciwnym
kierunku. ⊠
oooooooooooooooooooooooooooooooooooooooooooooooooooooooooooooooooooooo
3.5 Twierdzenie Muirheada dla podziałów liczb wymiernych
oooooooooooooooooooooooooooooooooooooooooooooooooooooooooooooooooooooo
W analogiczny sposób można rozpatrywać podziały dowolnej liczby wymiernej. Podzia-
łem długości k liczby wymiernej q nazywamy każdy ciąg α = (α1 , . . . , αk ) liczb wymiernych
takich, że α1 > α1 > · · · > αk i α1 + α2 + · · · + αk = q. Zbiór wszystkich podziałów
długości k liczby wymiernej q oznaczać będziemy, tak samo jak poprzednio, przez P(q, k).
Porządek > w zbiorze P(q, k) wprowadzamy tak samo, jak dla podziałów liczb naturalnych
lub całkowitych. Jeśli α = (α1 , . . . , αk ), β = (β1 , . . . , βk ) są podziałami należącymi do zbioru
P(q, k), to α > β, jeśli

α1 > β1 , α1 + α2 > β1 + β2 , α1 + α2 + α3 > β1 + β2 + β3 , ··· .

Przykłady:        
1 1 1 1 1 1 1 1
, > , , ,− > ,− .
3 6 4 4 2 2 3 3
Niech α = (α1 , . . . , αk ) ∈ P(q, k) i niech x1 , . . . , xk będą dodatnimi liczbami rzeczywisty-
mi. Oznaczmy przez Aα = Aα (x1 , . . . , xk ) dodatnią liczbę rzeczywistą xα1 1 xα2 2 · · · xαk k i niech
X
Tα = Tα (x1 , . . . , xk ) = Aα (xσ(1) , xσ(2) , . . . , xσ(k) ).
σ∈Sk

Mamy wówczas:

3.5.1 (Twierdzenie Muirheada dla wymiernych podziałów). Niech α, β będą podziałami dłu-
gości k liczby wymiernej q. Następujące dwa warunki są równoważne.
(1) α > β.
(1) Dla dowolnych dodatnich liczb rzeczywistych x1 , . . . , xk zachodzi nierówność

Tα (x1 , . . . , xk ) > Tβ (x1 , . . . , xk ).

D. Niech α = (α1 , . . . , αk ), β = (β1 , . . . , βk ) i niech w ∈ N będzie wspólnym mianownikiem


wszystkich liczb wymiernych α1 , . . . , αk , β1 , . . . , βk . Oznaczmy αi′ := wαi , βi′ = wβi , dla i = 1, . . . , k,
i niech α′ = (α1′ , . . . , αk′ ), β ′ = (β1′ , . . . , βk′ ). Wtedy α′ , β ′ są podziałami należącymi do P(m, k), gdzie
m jest liczbą całkowitą równą wq.
Załóżmy, że α > β. Wtedy α′ > β ′ i z twierdzenia Muirheada 3.4.1 (dla podziałów liczb całkowi-
tych) mamy nierówność
X α′ α′1
X β′ β1′
xσ(1)1
· · · xσ(1) > xσ(1)
1
· · · xσ(1) ,
σ∈Sn σ∈Sn
Nierówności 3. Twierdzenie Muirheada 43

zachodzącą dla wszystkich dodatnich liczb rzeczywistych x1 , . . . , xk . Zauważmy, że dla każdej permu-
tacji σ ∈ Sk zachodzi równość
1/w 1/w 1/w
(x1 x2 · · · xk )1/w = xσ(1) xσ(2) · · · xσ(k) .

Mnożąc stronami powyższą nierówność przez dodatnią liczbę (x1 x2 · · · xk )1/w otrzymujemy:
Tα (x1 , . . . , xk ) > Tβ (x1 , . . . , xk ). Udowodniliśmy, że z (1) wynika (2). Analogicznie dowodzimy impli-
kację w przeciwnym kierunku. ⊠
   
1 1 1 1
3.5.2. Z nierówności , > , wynika, na mocy twierdzenia 3.5.1, że
3 6 4 4

3
√ √ √ √
x 6 y + 3 y 6 y > 2 4 xy, dla x, y > 0.
U. Wstawiając x = a12 , y = b12 mamy stąd nierówność a4 b2 + a2 b4 > 2a3 b3 i po podzieleniu
przez a2 b2 otrzymujemy oczywistą nierówność a2 + b2 > 2ab. Ta ostatnia nierówność wynika np. z
twierdzenia Muirheada 3.3.1 zastosowanego do podziałów (2, 0) > (1, 1) liczby naturalnej 2.
Z dowodów twierdzeń 3.4.1 i 3.5.1 wynika, że wszystkie nierówności otrzymane za pomocą podzia-
łów liczb wymiernych można otrzymać z twierdzenia Muirheada 3.3.1 stosując odpowiednie podsta-
wienia. ⊠
oooooooooooooooooooooooooooooooooooooooooooooooooooooooooooooooooooooo
3.6 Nierówności cykliczne
oooooooooooooooooooooooooooooooooooooooooooooooooooooooooooooooooooooo
Niech r będzie dowolną liczbą rzeczywistą i niech n > 1 będzie liczbą naturalną. Oznacz-
my, tak jak poprzednio, przez P(n, r) zbiór wszystkich ciągów postaci α = (α1 , . . . , αn ), gdzie
α1 , . . . , αn są liczbami rzeczywistymi takimi, że α1 > α2 > · · · > αn oraz
α1 + · · · + αn = r.
W zbiorze P(n, r) wprowadzamy inny niż dotychczas porządek, który oznaczać będziemy
przez . Jeśli α = (α1 , . . . , αn ), β = (β1 , · · · , βn ) są ciągami należącymi do P(n, r), to
pisać będziemy α  β, jeśli istnieje ciąg (p1 , . . . , pn ) nieujemnych liczb wymiernych taki, że
p1 + · · · + pn = 1 oraz


 β1 = p1 α1 + p2 α2 + p3 α3 + · · · + pn αn ,



 β2 = p1 αn + p2 α1 + p3 α2 + · · · + pn αn−1 ,



(∗) β3 = p1 αn−1 + p2 αn + p3 α1 + · · · + pn αn−2 ,



 ..


 .


βn = p1 α2 + p2 α3 + p3 α4 + · · · + pn α1 .
Ciąg (p1 , . . . , pn ) nazywać będziemy w tym przypadku ciągiem wagowym nierówności α  β.

3.6.1. Jeśli r 6= 0 i (p1 , . . . , pn ) jest ciągiem niezerowych liczb wymiernych (niekoniecznie z


sumą równą 1) spełniającym układ (∗), to p1 + · · · + pn = 1.
D. Niech P = p1 + · · · + pn . Dodając wszystkie równości stronami otrzymujemy równość
β1 + · · · + βn = (α1 + · · · + αn )P,

czyli r = rP . Ale r 6= 0, więc P = 1. ⊠


44 Nierówności 3. Twierdzenie Muirheada

Definicję relacji  wygodnie jest zapisać w języku macierzowym. Jeśli u = (u1 , . . . , un )


jest dowolnym ciągiem liczb rzeczywistych, to przez M (u) oznaczać będziemy n × n macierz
 
u1 u2 u3 . . . un−1 un
 un u1 u2 . . . un−2 un−1 
 
 
M (u) =  un−1 un u1 . . . un−3 un−2 .
 
 .. 
 . 
u2 u3 u4 . . . un u1
Mamy zatem: α  β wtedy i tylko wtedy, gdy istnieje ciąg nieujemnych liczb wymiernych
(p1 , . . . , pn ) taki, że    
p1 β1
n    
X  p2   β2 
 ..  = 
pi = 1 oraz M (α)    .. .

i=1  .   . 
pn βn
W innej formie można to wyrazić następująco:

3.6.2. Jeśli α, β ∈ P(n, r), to α  β ⇐⇒ istnieje ciąg nieujemnych liczb wymiernych


p = (p1 , . . . , pn ) taki, że p1 + · · · + pn = 1 oraz

M (α) · M (p) = M (β).

Zanotujmy kilka znanych własności macierzy postaci M (u).

3.6.3. Jeśli u i v są ciągami (długości n) liczb rzeczywistych, to istnieje ciąg w (również


długości n) liczb rzeczywistych taki, że

M (u)M (v) = M (w).

3.6.4. Wyznacznik macierzy M (u) jest równy

f (ε1 )f (ε2 ) · · · f (εn ),

gdzie f (x) = u1 + u2 x + · · · + un xn−1 oraz ε1 , . . . , εn są pierwiastkami zespolonymi n-tego


stopnia z jedynki.

Korzystając z powyższych własności nie jest trudno wykazać, że  jest relacją częściowego
porządku w zbiorze P(n, r). Innymi słowy:

3.6.5. Niech r ∈ R i niech α, β, γ ∈ P(n, r). Wtedy:


(1) α  α;
(2) jeśli α  β oraz β  α, to α = β;
(3) jeśli α  β i β  γ, to α  γ.

Przypomnijmy, że jeśli α = (α1 , . . . , αn ), β = (β1 , . . . , βn ) są elementami zbioru P(n, r),


to piszemy α > β, jeśli

α1 > β1 , α1 + α2 > β1 + β2 , α1 + α2 + α3 > β1 + β2 + β3 , ....


Nierówności 3. Twierdzenie Muirheada 45

3.6.6. Jeśli α  β, to α > β.


D. Wykażemy to przykładowo dla n = 4. Dowód dla dowolnego n wygląda podobnie. Niech
α = (α1 , α2 , α3 , α4 ), β = (β1 , β2 , β3 , β4 ). Wtedy α1 > α2 > α3 > α4 , β1 > β2 > β3 > β4 i
α1 + α2 + α3 + α4 = r = β1 + β2 + β3 + β4 . Załóżmy, że α  β. Istnieje wtedy ciąg wagowy
(p1 , p2 , p3 , p4 ), tzn. wszystkie liczby pi są nieujemnymi liczbami wymiernymi, ich suma jest równa 1
oraz
β1 = p1 α1 + p2 α2 + p3 α3 + p4 α4 ,
β2 = p1 α4 + p2 α1 + p3 α2 + p4 α3 ,
β3 = p1 α3 + p2 α4 + p3 α1 + p4 α2 ,
β4 = p1 α2 + p2 α3 + p3 α4 + p4 α1 .
Mamy zatem β1 = p1 α1 +p2 α2 +p3 α3 +p4 α4 6 p1 α1 +p2 α1 +p3 α3 +p4 α1 = (p1 +p2 +p3 +p4 )α1 = α1 ,
czyli α1 > β1 .
β1 + β2 = (p1 α1 + p2 α2 + p3 α3 + p4 α4 ) + (p1 α4 + p2 α1 + p3 α2 + p4 α3 )
6 p1 α1 + p2 α2 + p3 α1 + p4 α1 + p1 α4 + p2 α1 + p3 α2 + p4 α3
= α1 + p2 α2 + p1 α4 + p3 α2 + p4 α3
6 α1 + p2 α2 + p1 α2 + p3 α2 + p4 α2
= α1 + α2 ,
czyli α1 + α2 > β1 + β2 .
β1 + β2 + β3 = (p1 α1 + p2 α2 + p3 α3 + p4 α4 ) + (p1 α4 + p2 α1 + p3 α2 + p4 α3 )
+(p1 α3 + p2 α4 + p3 α1 + p4 α2 ),
6 p1 α1 + p2 α2 + p3 α3 + p4 α4 + p1 α4 + p2 α1 + p3 α2 + p4 α3
+p1 α3 + p2 α4 + p3 α1 + p4 α1 ,
= α1 + p2 α2 + p3 α3 + p4 α4 + p1 α4 + p3 α2 + p4 α3 + p1 α3 + p2 α4 ,
6 α1 + p2 α2 + p3 α3 + p4 α4 + p1 α4 + p3 α2 + p4 α2 + p1 α2 + p2 α4 ,
= α1 + α2 + p3 α3 + p4 α4 + p1 α4 + p2 α4 ,
6 α1 + α2 + p3 α3 + p4 α3 + p1 α3 + p2 α3 ,
= α1 + α2 + α3 ,
czyli α1 +α2 +α3 > β1 +β2 +β3 . Ponieważ α1 +α2 +α3 +α4 = β1 +β2 +β3 +β4 = r, więc wykazaliśmy,
że α > β. ⊠

3.6.7. Jeśli α > β, to nie musi być prawdą, że α  β. Na przykład: (3, 1, 0) > (2, 2, 0) i nie
jest prawdą, że (3, 1, 0)  (2, 2, 0).
D. Przypuśćmy, że (3, 1, 0) > (2, 2, 0). Istnieją wówczas nieujemne liczby wymierne p1 , p2 , p3 takie,
że p1 + p2 + p3 = 1 oraz p1 3 + p2 1 + p3 0 = 2, p1 0 + p2 3 + p3 1 = 2 i p1 1 + p0 3 + p3 3 = 0. Trzy ostatnie
3 5 1
równości tworzą układ Cramera (o wyznaczniku równym 28). Mamy tu: p1 = , p2 = , p3 = − ,
7 7 7
czyli p3 < 0. ⊠

3.6.8. Niech α, β ∈ P(n, r). Jeśli n = 2, to α > β ⇐⇒ α  β.

Jeśli x = (x1 , . . . , xn ) jest ciągiem dodatnich liczb rzeczywistych oraz α = (α1 , . . . , αn ) ∈


P(n, r), to przez Rα (x) oznaczać będziemy liczbę rzeczywistą
Rα (x) = xα1 1 xα2 2 · · · xαnn + xα1 2 xα2 3 · · · xαn1 + · · · + xα1 n xα2 1 · · · xnαn−1 .
Na przykład: R(3,2,1) (x, y, z) = x3 y 2 z + y 3 z 2 x + z 3 x2 y.
46 Nierówności 3. Twierdzenie Muirheada

3.6.9 (Twierdzenie o nierówności cyklicznej). Niech r ∈ R oraz α, β ∈ P(n, r). Jeśli α  β,


to dla dowolnego ciągu dodatnich liczb rzeczywistych x = (x1 , . . . , xn ), zachodzi nierówność
Rα (x) > Rβ (x). ([Ko03] 48).

D. ([Ko03]). Ponieważ α  β, więc istnieją nieujemne liczby wymierne p1 , . . . , pn spełniające


układ (∗) i takie, że p1 + · · · + pn = 1. Co najmniej jedna z tych liczb jest więc większa od zera. Niech
w ∈ N będzie wspólnym mianownikiem wszystkich liczb p1 , . . . , pn . Mamy wtedy
c1 cn
p1 = , . . . , pn = ,
w w
gdzie c1 , . . . , cn są nieujemnymi liczbami całkowitymi, których suma jest równa w. Układ (∗) ma więc
postać: 
 wβ1 = c1 α1 + c2 α2 + c3 α3 + · · · + cn αn ,



 wβ2 = c1 αn + c2 α1 + c3 α2 + · · · + cn αn−1 ,


wβ3 = c1 αn−1 + c2 αn + c3 α1 + · · · + cn αn−2 ,



 ..


 .
wβn = c1 α2 + c2 α3 + c3 α4 + · · · + cn α1 .
Niech x = (x1 , . . . , xn ) będzie ciągiem dodatnich liczb rzeczywistych. Rozważmy teraz ciąg liczbowy
(u1 , u2 , . . . , uw ), w którym c1 początkowych liczb jest równych xα1 α2 αn
1 x2 · · · xn , następnych c2 liczb jest
α2 α3 αn α1
α1
równych x1 x2 · · · xn , itd., oraz końcowych cn liczb jest równych x1 x2 · · · xnαn−1 . Dla tego ciągu
rozważmy nierówność pomiędzy średnią arytmetyczną i średnią geometryczną. Mamy wtedy:
p1 xα1 α2 αn α2 α3 α1 αn α1 αn−1
1 x2 · · · xn + p2 x1 x2 · · · xn + · · · + pn x1 x2 · · · xn > xβ1 1 xβ2 2 · · · xβnn .
Czyniąc to samo, zastępując ciąg (α1 , α2 , . . . , αn ) ciągiem (α2 , α3 , . . . , αn , α1 ), otrzymujemy:
β2 β3
p1 xα2 α3 α1 α3 α4 α2 α1 α2 αn β1
1 x2 · · · xn + p2 x1 x2 · · · xn + · · · + pn x1 x2 · · · xn > x1 x2 · · · xn .

Kontynuując to postępowanie, otrzymamy n nierówności, które po dodaniu stronami utworzą żądaną


nierówność Rα (x) > Rβ (x). ⊠

Przedstawimy teraz kilka zastosowań twierdzenia 3.6.9.


x3 y3 z3 x2 y 2 z 2
3.6.10. + + > + + , dla x, y, z > 0. ([Ko03] 49).
yz zx xy y z x
D. Niech α = (3, −1, −1), β = (2, −1, 0). Wtedy α, β ∈ P(3, 1). Niech p1 = 34 , p2 = 14 , p3 = 0.
Liczby p1 , p2 , p3 są wymierne, nieujemne oraz
p1 3 + p2 (−1) + p3 (−1) = 2, p1 (−1) + p2 (−1) + p3 3 = −1, p1 (−1) + p2 3 + p3 (−1) = 0.

Zatem α  β i rozpatrywana nierówność wynika z twierdzenia 3.6.9. ⊠

3.6.11. x4 y + y 4 z + z 4 x > x3 yz + y 3 zx + z 3 xy, dla x, y, z > 0. ([Ko03] 51).

9 3 1
D. Niech α = (4, 1, 0), β = (3, 1, 1). Wtedy α, β ∈ P(3, 5). Niech p1 = 13 , p2 = 13 , p3 = 13 .
Liczby p1 , p2 , p3 są wymierne, nieujemne oraz
p1 4 + p2 1 + p3 0 = 3, p1 1 + p2 0 + p3 4 = 1, p1 0 + p2 4 + p3 1 = 1.

Zatem α  β i rozpatrywana nierówność wynika z twierdzenia 3.6.9. ⊠


⋆ J. Górnicki, Nierówności cykliczne, [Dlt] 10/1987, [Gorn].
J. M. Steele, Symmetric sums, [Stee] 178-190.
⊡⊡⊡⊡⊡⊡⊡⊡⊡⊡⊡⊡⊡⊡⊡⊡⊡⊡⊡⊡⊡⊡⊡⊡⊡⊡⊡⊡⊡⊡⊡⊡⊡⊡⊡⊡⊡⊡⊡⊡⊡⊡⊡⊡⊡⊡⊡⊡⊡⊡⊡⊡
4 Jednorodne nierówności wielomianowe
⊡⊡⊡⊡⊡⊡⊡⊡⊡⊡⊡⊡⊡⊡⊡⊡⊡⊡⊡⊡⊡⊡⊡⊡⊡⊡⊡⊡⊡⊡⊡⊡⊡⊡⊡⊡⊡⊡⊡⊡⊡⊡⊡⊡⊡⊡⊡⊡⊡⊡⊡⊡

oooooooooooooooooooooooooooooooooooooooooooooooooooooooooooooooooooooo
4.1 Jednorodne nierówności wielomianowe n zmiennych
oooooooooooooooooooooooooooooooooooooooooooooooooooooooooooooooooooooo

4.1.1. Dla dowolnych liczb rzeczywistych x1 , . . . , xn , zachodzi nierówność

(x1 + x2 + · · · + xn )2 6 n(x21 + x22 + · · · + x2n ).

([Kw] 4/1976 18).

4.1.2. W poniższych nierównościach x1 , . . . , xn są dowolnymi liczbami rzeczywistymi.


(1) x21 + x22 + · · · + x2n > x1 x2 + x2 x3 + · · · + xn−1 xn ,
4
(2) x21 + x22 + · · · + x2n > (x1 x2 + x2 x3 + · · · + xn−1 xn ), dla n > 3.
3
6
(3) x21 + x22 + · · · + x2n > (x1 x2 + x2 x3 + · · · + xn−1 xn ), dla n > 4. ([Kw] 12/1974 46).
5

4.1.3. x21 + x22 + x23 + x24 + x25 > x1 (x2 + x3 + x4 + x5 ), dla x1 , . . . , x5 ∈ R. ([OM] Mołdawia 1998).

4.1.4. Dla jakich n > 2 prawdziwa jest nierówność

x21 + x22 + · · · + x2n > xn (x1 + x2 + · · · + xn−1 ),

dla wszystkich x1 , . . . , xn ∈ R? Odp. Tylko dla n = 2, 3, 4, 5. ([Fom] 28/83).

4.1.5. Niech (xn ) będzie ciągiem dodatnich liczb rzeczywistych. Niech


1
yn = (x1 + x2 + · · · + xn ),
n
dla n ∈ N. Wtedy dla każdej liczby naturalnej n zachodzi nierówność

(y12 + y22 + · · · + yn2 ) 6 4(x21 + x22 + · · · + x2n ). ([OM] Korea 2005).

4.1.6. n2 (x31 + · · · + x3n ) > (x1 + · · · + xn )3 , dla x1 , . . . , xn > 0. ([MOc] 1998/1999).

D. Rozpatrzmy wypukłą funkcję f : (0, ∞) → R, f (x) = x3 . Z nierówności Jensena 1.1.8 mamy:


n
!3 n
1X 1X 3
xi 6 x
n i=1 n i=1 i

i stąd otrzymujemy rozpatrywaną nierówność. ⊠

47
48 Nierówności 4. Jednorodne nierówności wielomianowe

4.1.7. x31 + x32 + · · · + x3n > x21 x2 + x22 x3 + · · · + x2n x1 , dla n > 2 i x1 , . . . , xn > 0. (Patrz 4.1.13).

n
X n
X
4.1.8. x2i+1 xi > x2i xi+1 , dla xn > · · · > x1 > 0, przy czym xn+1 = x1 . ([Ko03] 107).
i=1 i=1

n
!4
X X
4.1.9. 8 xi xj (x2i + x2j ) 6 xi , dla n > 2 i x1 , . . . , xn > 0. ([IMO] 2000).
1¬i<j¬n i=1

4.1.10. x1 x42 + x2 x43 + · · · + xn x41 > x2 x41 + x3 x42 + · · · + x1 x4n , dla x1 < x2 < · · · < xn .
([OM] Iran 1999).

     
4.1.11. x1 +· · ·+xn x71 +· · ·+x7n > x31 +· · ·+x3n x51 +· · ·+x5n , dla x1 , x2 , . . . , xn > 0.
([Khr3] 23).

n
X  
4.1.12. x1 · · · xbi · · · xn x1 + · · · + x
bi + · · · xn > n(n − 1)x1 · · · xn , dla x1 , . . . , xn > 0.
i=1
([Str] 136).

4.1.13. xn1 + xn2 + · · · + xnn > xn−1


1 x2 + xn−1
2 x3 + · · · + xn−1
n x1 , dla x1 , . . . , xn > 0.

D. Korzystamy z twierdzenia 3.6.9. Mamy tu (n, 0, 0, . . . , 0)  (n − 1, 1, 0, . . . , 0). Ciągiem wago-


n−1 1
wym jest p1 = , p2 = , p3 = p4 = · · · = pn = 0. ⊠
n n

4.1.14. x1 x2 · · · xn (x1 + x2 + · · · + xn ) 6 xn+1


1 + xn+1
2 + · · · + xn+1
n , dla x1 , . . . , xn > 0.
([Mat] 2/1952 117, [Cmj] 23(4)(1992) 344).

 
4.1.15. x1 x2 . . . xn xs1 +xs2 +· · ·+xsn 6 xn+s
1 +xn+s
2 +· · ·+xn+s
n , dla n, s ∈ N, x1 , . . . , xn > 0.
([IMO] Longlist 1967, [Djmp] s.48(357)).

    
4.1.16. n xn1 + xn2 + · · · + xnn > x1 + x2 + · · · + xn xn−1
1 + xn−1
2 + · · · + xn−1
n , dla
x1 , . . . , xn > 0. (Len Bos, [Crux] z.1288).

4.1.17. Niech x1 , . . . , xn ∈ R. Niech


n
X
En = (xi − x1 )(xi − x2 ) · · · (x\
i − xi ) · · · (xi − xn ).
i=1

(1) Jeśli n = 3 lub n = 5, to En > 0.


(2) Dla pozostałych n, większych od 3, już tak nie jest. ([IMO] 1971).

x1 + x2 x2 + x3 x5 + x1 x1 + x2 + x3 x2 + x3 + x4 x5 + x1 + x2
4.1.18. · ··· 6 · ··· ,
2 2 2 3 3 3
dla x1 , . . . , x5 > 0. ([OM] St Petersburg 2002).
Nierówności 4. Jednorodne nierówności wielomianowe 49

x1 + x2 x2 + x3 xn + x1 x1 + x2 + x3 x2 + x3 + x4 xn + x1 + x2
4.1.19. · ··· 6 · ··· ,
2 2 2 3 3 3
gdy x1 > x2 > · · · > xn > 0. ([Khr2], [Khr1]).
x1 + x2 x2 + x3 xn + x1 x1 + x2 + x3 x2 + x3 + x4 xn + x1 + x2
4.1.20. · ··· 6 √ · √ ··· √ ,
2 2 2 2 2 2 2 2
dla x1 , . . . , xn > 0. ([OM] St Petersburg 2000).

4.1.21. ([Zw] 2003). Jeśli x1 , . . . , xn > 0 i n > 3, to


 √ n
2 (x1 + x2 )(x2 + x3 ) · · · (xn + x1 ) < (x1 + x2 + x3 )(x2 + x3 + x4 ) · · · (xn + x1 + x2 ).

oooooooooooooooooooooooooooooooooooooooooooooooooooooooooooooooooooooo
4.2 Problem Janousa
oooooooooooooooooooooooooooooooooooooooooooooooooooooooooooooooooooooo
Matematyk austriacki Walther Janous zaproponował w 1992 roku, w czasopiśmie Crux
Mathematicorum, następujące zadanie.

4.2.1 (Janous). Niech n i k będą liczbami naturalnymi takimi, że 2 6 k < n i niech x1 , . . . , xn


będą nieujemnymi liczbami rzeczywistymi spełniającymi równość x1 +· · ·+xn = 1. Udowodnić
lub podać kontrprzykład na to, że
 
1 1
x1 x2 · · · xk + x2 x3 · · · xk+1 + · · · + xn x1 x2 · · · xk−1 6 max , .
k k nk−1
([Crux] 1992 z.1754).

Dalej, po sformułowaniu tego zadania, Janous zaznacza, że przypadek k = 2 jest znany i


opisany w rozwiązaniu wcześniejszego zadania z Crux Mathematicorum 1992 (Problem 1662).
Problem jest otwarty. Wszystkie znane częściowe wyniki dotyczą prawdziwości rozważanej
nierówności. Nie ma kontrprzykładów.

Niech x1 , . . . , xn będą nieujemnymi liczbami rzeczywistymi, z których co najmniej jedna


jest różna od zera. Przy pomocy tych liczb definiujemy liczby y1 , . . . , yn :
x1 x2 xn
y1 = , y2 = , . . . , yn = .
x1 + · · · + xn x1 + · · · + xn a1 + · · · + xn
Liczby y1 , . . . , yn są nieujemne i ich suma jest równa 1. Wstawiając je do nierówności z
problemu Janousa, otrzymujemy następującą jednorodną wersję tego problemu.

4.2.2 (Janous). Niech n i k będą liczbami naturalnymi takimi, że 2 6 k < n i niech x1 , . . . , xn


będą nieujemnymi liczbami rzeczywistymi. Udowodnić lub podać kontrprzykład na to, że
 
1 1
x1 x2 · · · xk + x2 x3 · · · xk+1 + · · · + xn x1 x2 · · · xk−1 6 max k
, k−1 (x1 + · · · + xn )k .
k n
50 Nierówności 4. Jednorodne nierówności wielomianowe

Znanych jest dużo różnych dowodów dla k = 2. Pierwszy taki dowód, wymyślony przez
W. L. Rabinowicza, jest w [Kw] 11/1974 s.45. W postaci jednorodnej powyższa nierówność
Janousa (dla k = 2) ma następującą postać.

4.2.3.
(1) 4x1 x2 6 (x1 + x2 )2 , dla x1 , x2 ∈ R.
(2) 3(x1 x2 + x2 x3 + x3 x1 ) 6 (x1 + x2 + x3 )2 , dla x1 , x2 , x3 ∈ R.
(3) 4(x1 x2 + x2 x3 + x3 x4 + x4 x1 ) 6 (x1 + x2 + x3 + x4 )2 , dla x1 , x2 , x3 , x4 ∈ R.
(4) 4(x1 x2 + x2 x3 + · · · + xn−1 xn + xn x1 ) 6 (x1 + x2 + · · · + xn )2 , dla x1 , x2 , . . . xn > 0,
gdzie n > 4. ([Kw] 4/1974 37, [Kw] 11/1974 45).

Stąd mamy następujące nierówności.

1
4.2.4. xy + yz + zx 6 , dla x + y + z = 1, x, y, z > 0. ([Ko04] 108).
3
1
4.2.5. xy + yz + zt + tx 6 , gdy x, y, z, t > 0 i x + y + z + t = 1. ([OM] Indie 1993).
4
 
1 1
4.2.6. x1 x2 + x2 x3 + · · · + xn x1 6 max , , dla x1 + · · · + xn = 1, x1 , . . . , xn > 0.
4 n
([Ko04] 108).

1
4.2.7. x1 x2 + x2 x3 + · · · + xn x1 6 , dla n > 4, x1 + · · · + xn = 1, x1 , . . . , xn > 0.
4
([Kw] 4/1974, 11/1974).

4.2.8 (W. Pompe 1994, M. Kuczma 1995). Problem Janousa ma pozytywne rozwiązanie dla
k 6 4 i dowolnego n. ([Crux] 7(1994) 196-199, 7(1995) 236-238).

Stąd otrzymujemy:

1
4.2.9. xyz + yzt + ztx + txy 6 , dla x + y + z + t = 1, x, y, z, t > 0. ([Ko04] 112).
16
1
4.2.10. x1 x2 x3 +x2 x3 x4 +x3 x4 x5 +x4 x5 x1 +x5 x1 x2 6 , dla x1 +· · ·+x5 = 1, x1 , . . . , x5 >
25
0. ([Ko04] 113).

1
4.2.11. x1 x2 x3 x4 +x2 x3 x4 x5 +x3 x4 x5 x1 +x4 x5 x1 x2 +x5 x1 x2 x3 6 , dla x1 +· · ·+x5 = 1,
125
x1 , . . . , x5 > 0. ([Ko04] 116).
 
1 1
4.2.12. x1 x2 x3 +x2 x3 x4 +· · ·+xn x1 x2 6 max , , dla x1 +· · ·+xn = 1, x1 , . . . , xn > 0.
27 n2
([Ko04] 112).
Nierówności 4. Jednorodne nierówności wielomianowe 51

4.2.13. Problem Janousa ma pozytywne rozwiązanie w następujących przypadkach.


(1) k = 5, n dowolne;
(2) n = k + 1; n = k + 2;
(3) n = k + 2, (L. Kurlandczyk, Piotr Kumor);

(4) n = km, k > 2, m > 2;


(5) n = km + 1, k > 2, m > 2;
(6) n = 2k − 2, n = 2k − 1, k > 2;
(7) n > 2k > 4.
([Crux] 7(1994) 196-199, [Ko04]).

⋆ L. Kurlandczyk, Spróbujmy rozwiązać zadanie, [Kw] 2(1996)40-42.


L. Kurlandczyk, Nierówność Janousa, [Ko04], 107-163.
oooooooooooooooooooooooooooooooooooooooooooooooooooooooooooooooooooooo
4.3 Jednorodne nierówności wielomianowe dwóch zmiennych
oooooooooooooooooooooooooooooooooooooooooooooooooooooooooooooooooooooo

4.3.1. a3 + b3 > ab(a + b), dla a, b > 0.

D. Wynika z twierdzenia Muirheada 3.3.1 (dla α = (3, 0), β = (2, 1). ⊠

4.3.2. 4(a3 + b3 ) > (a + b)3 , dla a, b > 0. (4.1.6).

4.3.3. a4 + b4 > ab(a2 + b2 ), dla a, b > 0.

D. Wynika z twierdzenia Muirheada 3.3.1 dla α = (4, 0), β = (3, 1). ⊠

4.3.4. a4 + b4 > 2ab3 , dla a > b > 0. ([CieS] 1996).

D. Korzystamy z nierówności między średnią arytmetyczną i geometryczną: a4 + b4 > 2a2 b2 >


2ab . ⊠
3

4.3.5. a5 + b5 > ab(a3 + b3 ) > a2 b2 (a + b), dla a, b > 0.

D. Wynika z twierdzenia Muirheada 3.3.1, gdyż (5, 0) > (4, 1) > (3, 2). ⊠

     
4.3.6. a+b a4 + b4 > a2 + b2 a3 + b3 , a, b > 0. ([WyKM] 494-68).

4.3.7. 5(a5 + b5 ) > 2(ab4 + a2 b3 + a3 b2 + a4 b), dla a > 0, b > 0. ([MC] 1/1998 39).

4.3.8. a6 + b6 > ab(a4 + b4 ) > a2 b2 (a2 + b2 ) > 2a3 b3 , dla a, b > 0.

D. Wynika z twierdzenia Muirheada 3.3.1, gdyż 60 > 51 > 42 > 33. ⊠


52 Nierówności 4. Jednorodne nierówności wielomianowe

4.3.9. 2(a3 + b3 )2 > (a2 + b2 )3 , dla a, b > 0. ([Ko00]).

4.3.10. an + bn > ab(an−2 + bn−2 ), dla a, b > 0.

D. Wynika z twierdzenia Muirheada 3.3.1 (dla α = (n, 0), β = (n − 1, 1). ⊠

4.3.11. Jeśli a, b ∈ R, a > 0, b > 0, to


        
n−1 m−1 an+m + bn+m + n + m − 1 an bm + am bn > nm an+m−1 b + abn+m−1 ,

dla wszystkich n, m ∈ N. ([Balt] 1995).

oooooooooooooooooooooooooooooooooooooooooooooooooooooooooooooooooooooo
4.4 Jednorodne nierówności wielomianowe trzech zmiennych stopnia 2
oooooooooooooooooooooooooooooooooooooooooooooooooooooooooooooooooooooo

4.4.1.
(1) x2 + y 2 + z 2 > xy + yz + zx,
(2) (x − y)(x − z) + (y − z)(y − x) + (z − x)(z − y) > 0.
(3) 3(x2 + y 2 + z 2 ) > (x + y + z)2 .
 2
x+y+z xy + yz + zx
4.4.2. > , dla x, y, z > 0. ([Ko00]).
3 3
q
1 
4.4.3. (x − y)2 + (y − z)2 + (z − x)2 6 x2 + y 2 + z 2 − 3 3 x2 y 2 z 2
3
6 (x − y)2 + (y − z)2 + (z − x)2 ,
dlx x, y, z > 0. ([OM] Irlandia 2005).

4.4.4. Jeśli a, b, c są dowolnymi liczbami rzeczywistymi, to

(a(y + z) + b(z + x) + c(x + y))2 > 4(ab + bc + ca)(xy + yz + zx),

dla wszystkich nieujemnych liczb x, y, z. (V. N. Murty, [Crux] z.3076).

4.4.5. Jeśli 0 < a 6 b 6 c są liczbami rzeczywistymi, to


 
1 1 1 (a + c)2
(ax + by + cz) x+ y+ z 6 (x + y + z)2 ,
a b c 4ac
dla dodatnich x, y, z. ([IMO] Longlist 1971).
Nierówności 4. Jednorodne nierówności wielomianowe 53

oooooooooooooooooooooooooooooooooooooooooooooooooooooooooooooooooooooo
4.5 Jednorodne nierówności wielomianowe trzech zmiennych stopnia 3
oooooooooooooooooooooooooooooooooooooooooooooooooooooooooooooooooooooo
4.5.1. 2(a3 + b3 + c3 ) > ab(a + b) + bc(b + c) + ca(c + a) > 6abc, dla a, b, c > 0.
Dowód. Wynika z twierdzenia Muirheada 3.3.1, gdyż (3, 0, 0) > (2, 1, 0) > (1, 1, 1) ⊠.

4.5.2. a3 + b3 + c3 > a2 b + b2 c + c2 a > 3abc, dla a, b, c > 0. (3.6.9).

4.5.3. Dla dodatnich liczb a, b, c zachodzą następujące nierówności.


(1) 3(a3 + b3 + c3 ) > (a + b + c)(a2 + b2 + c2 ) > 9abc, ([Kw] 2/1997 37);
3 3 3
(2) 3(a + b + c ) > (a + b + c)(ab + bc + ca), ([Kw] 2/1997 37);

(3) 8(a3 + b3 + c3 ) > 3(a + b)(b + c)(c + a), ([Dlt] 1/1978);

(4) a + b + c + 3abc > a (b + c) + b (a + c) + c2 (a + b),


3 3 3 2 2
([WaJ] 212(75), [Br83] 54);
1
(5) a3 + b3 + c3 + 6abc > (a + b + c)3 , ([S-kg] 35, [Mild]).
4

4.5.4. a(a − b)(a − c) + b(b − c)(b − a) + c(c − a)(c − b) > 0, dla a, b, c > 0. ([Crux] 1998 s.347).

Powyższa nierówność jest szczególnym przypadkiem następującej nierówności Schura.


4.5.5 (Nierówność Schura). Jeśli a, b, c > 0 oraz r > 0, to

ar (a − b)(a − c) + br (b − c)(b − a) + cr (c − a)(c − b) > 0 .

Równość zachodzi wtedy i tylko wtedy, gdy a = b = c lub dwie z liczb a, b, c są równe i trzecia
jest równa zero. ([RiM] July 2001, [Mild], [LeH2]).
Ta z kolei nierówność jest szczególnym przypadkiem następującej, jeszcze ogólniejszej
nierówności.
4.5.6. Niech I ⊆ R będzie przedziałem i niech f : I → R będzie funkcją taką, że f (x) > 0
dla x ∈ I. Załóżmy, że funkcja f jest rosnąca lub wypukła. Wtedy

f (x)(x − y)(x − z) + f (y)(y − z)(y − x) + f (z)(z − x)(x − y) > 0


dla wszystkich x, y, z ∈ I z wyjątkiem przypadku x = y = z. ([MG] 40(333)(1956) 217).

D. (E. M. Wright, [MG] 1956). Niech x, y, z ∈ I i oznaczmy:


g = f (x)(x − y)(x − z) + f (y)(y − z)(y − x) + f (z)(z − x)(x − y).
Jeśli x = y 6= z, to g = f (z)(x − z)2 > 0. Możemy więc dalej założyć, że x < y < z. Mamy zatem
nierówność
0 < f (y) 6 max(f (x), f (z))
(wynikającą z monotoniczności lub wypukłości). Mamy ponadto:
0 < (z − y)(y − x) < (z − x)(y − x) = (x − y)(x − z),
0 < (z − y)(y − x) < (x − y)(z − x).
Zatem f (y)(z − y)(y − x) < f (x)(x − y)(x − z) + f (z)(z − x)(z − y) i stąd g > 0. ⊠
54 Nierówności 4. Jednorodne nierówności wielomianowe

4.5.7. Dla dodatnich liczb a, b, c zachodzą następujące nierówności.


(1) (a + b + c)(a2 + b2 + c2 ) > 3(a2 b + b2 c + c2 a), ([Ko00]);

(2) a(a − c)2 + b(b − c)2 > (a − c)(b − c)(a + b − c), ([OM] Kanada 92, [Pa97]);

(3) (a + b + c)3 − 4(a + b + c)(ab + bc + ca) + 9abc > 0, ([Kw] 2/1997 37);
 
(4) (a + b + c) 2(ab + bc + ca) − (a2 + b2 + c2 ) 6 9abc, ([Math] 2006).

4.5.8. Dla dodatnich liczb a, b, c zachodzą nierówności:


(1) 8abc 6 (a + b)(b + c)(c + a); ([Kw] 2/1997 37, [ME] 11(1)(2006)).

(2) 6abc 6 ab(a + b) + bc(b + c) + ca(c + a). ([Str] 61).

   
a+b b+c c+a √ √ √
D. (a + b)(b + c)(c + a) = 8 > 8 ab · bc · ca = 8abc.
2 2 2
Wynika to również z nierówności 1.5.5 zastosowanej dla wypukłej funkcji f : (0, ∞) → R określonej
wzorem f (x) = − ln x. Dokładniej, na mocy 1.5.7 mamy:

ln(2a) + ln(2b) + ln(2c) 6 ln(a + b) + ln(b + c) + ln(c + a),

czyli 8abc 6 (a + b)(b + c)(c + a). Stąd 8abc 6 abc + a2 b + ab2 + b2 c + bc2 + c2 a + ca2 + 2abc, czyli

ab(a + b) + bc(b + c) + ca(c + a) > 6abc.

Wykazaliśmy to też w 4.5.1. ⊠

4.5.9. Niech a, b, c > 0 i niech

x = b + c − a, y = a + c − b, z = a + b − c.

Zachodzą wówczas następujące nierówności.


(1) xyz 6 abc, ([Khr1], [LeH2]).

(2) (a + b)(b + c)(c + a) > 8xyz, ([Berk] 4a/93).

(3) ab(z − c) + bc(x − a) + ca(y − b) > 0, ([StaZ] 63).

(4) 3abc > a2 x + b2 y + c2 z, ([Str67] 50).

(5) x3 + y 3 + z 3 > 3abc, ([Mat] 3/1999 z.1472).

D. (1). Możemy założyć, że a > b > c. Wtedy z > y > x oraz z > a, z + y > a + b i z + y + x =
a + b + c. Oczywiście z > 0 i y > 0. Jeśli x 6 0, to rozpatrywana nierówność jest oczywista. Dalej
zakładamy, że x > 0.
Z nierówności 1.5.5 dla wypukłej funkcji f : (0, ∞) → R, f (x) = − ln x, mamy

ln z + ln y + ln x 6 ln a + ln b + ln c,

czyli xyz 6 abc. ⊠


Nierówności 4. Jednorodne nierówności wielomianowe 55

4.5.10. Niech a, b, c ∈ N, x, y, z ∈ R+ . Wtedy

(ax + by + cz)(bx + cy + az)(cx + ay + bz) > (a + b + c)3 xyz.

([OM] Rosja 1996).

D. Z nierówności Cauchy’ego otrzymujemy:


ax + by + cz p
a+b+c bx + cy + az p
a+b+c cx + ay + bz p
a+b+c
> xa y b z c , > xb y c z a , > xc y a z b .
a+b+c a+b+c a+b+c
Mnożąc te nierówności stronami otrzymujemy tezę. ⊠

4.5.11. 9(a3 + b3 + c3 ) > (a + b + c)3 , dla a, b, c > 0. (4.1.6, [OM] W.Brytania 1996, [Ko00]).

oooooooooooooooooooooooooooooooooooooooooooooooooooooooooooooooooooooo
4.6 Jednorodne nierówności wielomianowe trzech zmiennych stopnia 4
oooooooooooooooooooooooooooooooooooooooooooooooooooooooooooooooooooooo

4.6.1. 2(a4 + b4 + c4 ) > ab(a2 + b2 ) + bc(b2 + c2 ) + ca(c2 + a2 )


> 2(a2 b2 + b2 c2 + c2 a2 )
> 2abc(a + b + c),
dla a, b, c > 0.

D. Wynika to z twierdzenia Muirheada 3.3.1, gdyż (4, 0, 0) > (3, 1, 0) > (2, 2, 0) > (2, 1, 1). ⊠

4.6.2. Dla a, b, c > 0 zachodzą następujące nierówności.


(1) a4 + b4 + c4 > a3 b + b3 c + c3 a > a2 bc + b2 ca + c2 ab, (3.6.9);

(2) a4 + b4 + c4 > a2 b2 + b2 c2 + c2 a2 > a2 bc + b2 ca + c2 ab, (3.6.9);

(3) a4 + b4 + c4 + abc(a + b + c) > 2(a2 b2 + b2 c + c2 a2 ), ([Crux] 1998 s.347);

(4) a4 + b4 + c4 + 2(a2 b2 + b2 c2 + c2 a2 ) 6 3(a3 b + b3 c + c3 a)), (Len Bos, [Crux] z.1288);

(5) 4(a4 + b4 + c4 ) > ab(a + b)2 + bc(b + c)2 + ca(c + a)2 , ([OMm] 1997/1998);
2 2 2 2 3 3 3
(6) (a + b + c ) > 3(a b + b c + c a), (Vasile Cirtoaje, [Mild], [Pkh] s.188);
2
(7) (ab + bc + ca) > 3abc(a + b + c), ([Kw] 2/1997 37);

(8) a2 b2 + b2 c2 + c2 a2 > abc(a + b + c), dla a, b, c ∈ R. ([Kw] 2/2003 M1834);

(9) a2 (a − b)(a − c) + b2 (b − c)(b − a) + c2 (c − a)(c − b) > 0, (nierówność Schura, 4.5.5).


56 Nierówności 4. Jednorodne nierówności wielomianowe

oooooooooooooooooooooooooooooooooooooooooooooooooooooooooooooooooooooo
4.7 Jednorodne nierówności wielomianowe trzech zmiennych stopnia 5
oooooooooooooooooooooooooooooooooooooooooooooooooooooooooooooooooooooo

4.7.1. Dla dodatnich liczb a, b, c zachodzą nierówności:

2(a5 + b5 + c5 ) > ab(a3 + b3 ) + bc(b3 + c3 ) + ca(c3 + a3 )


> a2 b2 (a + b) + b2 c2 (b + c) + c2 a2 (c + a)
> 2abc(a2 + b2 + c2 )
> 2abc(ab + bc + ca).

Dowód. Wynika z twierdzenia Muirheada 3.3.1, gdyż 500 > 410 > 320 > 311 > 221 ⊠.

4.7.2. Dla a, b, c > 0 zachodzą następujące nierówności.


(1) a5 + b5 + c5 > a4 b + b4 c + c4 a > a3 bc + b3 ca + c3 ab > a2 b2 c + b2 c2 a + c2 a2 b, (3.6.9);

(2) a5 + b5 + c5 > a3 b2 + b3 c2 + c3 a2 > a2 b2 c + b2 c2 a + c2 a2 b, (3.6.9);

(3) (a2 + b2 + c2 )(a + b − c)(b + c − a)(c + a − b) 6 abc(ab + bc + ca), ([Math] 2006);

(4) a3 (a − b)(a − c) + b3 (b − c)(b − a) + c3 (c − a)(c − b) > 0, (nierówność Schura, 4.5.5).

oooooooooooooooooooooooooooooooooooooooooooooooooooooooooooooooooooooo
4.8 Jednorodne nierówności wielomianowe trzech zmiennych stopnia 6
oooooooooooooooooooooooooooooooooooooooooooooooooooooooooooooooooooooo

4.8.1. Dla dodatnich liczb a, b, c zachodzą nierówności:

2(a6 + b6 + c6 ) > ab(a4 + b4 ) + bc(b4 + c4 ) + ca(c4 + a4 )


> a2 b2 (a2 + b2 ) + b2 c2 (b2 + c2 ) + c2 a2 (c2 + a2 )
> 2abc(a3 + b3 + c3 )
oraz 2(a3 b3 + b3 c3 + c3 a3 )
> abc (ab(a + b) + bc(b + c) + ca(c + a))
> 6a2 b2 c2 .

Pomiędzy abc(a3 + b3 + c3 ) i a3 b3 + b3 c3 + c3 a3 nie ma żadnych ogólnych nierówności.


D. Wynika z twierdzenia Muirheada 3.3.1, gdyż 600 > 510 > 420 > 321 > 222 oraz 420 > 411 >
321 i 420 > 330 > 321. Pomiędzy 411 = (4, 1, 1) i 330 = (3, 3, 0) nie ma żadnej nierówności.
Niech A(a, b, c) = abc(a3 + b3 + c3 ) i B(a, b, c) = a3 b3 + b3 c3 + c3 a3 . Wtedy A(2, 1, 1) = 20 > 17 =
B(2, 1, 1), A(2, 2, 1) = 68 < 80 = B(2, 2, 1). ⊠

4.8.2. Dla dodatnich liczb a, b, c zachodzą następujące nierówności.


(1) a6 + b6 + c6 > a5 b + b5 c + c5 a > a4 bc + b4 ca + c4 ab > a3 b2 c + b3 c2 a + c3 a2 b > 3a2 b2 c2 ;
(2) a6 + b6 + c6 > a4 b2 + b4 c2 + c4 a2 > a3 b2 c + b3 c2 a + c3 a2 b > 3a2 b2 c2 ;
(3) a6 + b6 + c6 > a3 b3 + b3 c3 + c3 a3 > a3 b2 c + b3 c2 a + c3 a2 b > 3a2 b2 c2 . (3.6.9).
Nierówności 4. Jednorodne nierówności wielomianowe 57

4.8.3. Dla dodatnich liczb a, b, c zachodzą następujące nierówności.


(1) a6 + b6 + c6 + 3(abc)2 > 2(a3 b3 + b3 c3 + c3 a3 ), ([Kw] 3/2003 17-18);

(2) (a3 + b3 + c3 )2 + 3(abc)2 > 4(a3 b3 + b3 c3 + c3 a3 ), (M. S. Klamkin, [Crux] z.2839);

(3) (a+b)2 (b+c)2 (c+a)2 > abc(2a+b+c)(2b+c+a)(2c+a+b), ([Crux] 1998 s.526, [NoO] s.79);
(4) (a + b − c)2 (b + c − a)2 (c + a − b)2 > (a2 + b2 − c2 )(b2 + c2 − a2 )(c2 + a2 − b2 ),
([IMO] Longlist 1986, [Ko00] s.42);

(5) (a2 + ab + b2 )(b2 + bc + c2 )(c2 + ca + a2 ) > (ab + bc + ca)3 , (S. Arslanagić, [Crux] z.2976);

(6) (a2 b + b2 c + c2 a)(ab2 + bc2 + ca2 ) > 9a2 b2 c2 , ([Mild]);


(7) 3(a2 b + b2 c + c2 a)(ab2 + bc2 + ca2 ) > abc(a + b + c)3 , ([AnC]);

(8) (a + b + c)3 (a + b − c)(b + c − a)(c + a − b) 6 27(abc)2 , dla a, b, c > 0. ([Math] 2006);

(9) a4 (a − b)(a − c) + b4 (b − c)(b − a) + c4 (c − a)(c − b) > 0, (nierówność Schura, 4.5.5).



3 2
 
4.8.4. a3 + b3 + c 6 a2 + b2 + c 2 a4 + b4 + c , dla dowolnych a, b, c.
4 (Patrz 2.6.5).

oooooooooooooooooooooooooooooooooooooooooooooooooooooooooooooooooooooo
4.9 Jednorodne nierówności wielomianowe trzech zmiennych stopnia > 6
oooooooooooooooooooooooooooooooooooooooooooooooooooooooooooooooooooooo
4.9.1. Dla a, b, c > 0 zachodzą następujące nierówności, wynikające z twierdzenia 3.6.9.
(1) a7 + b7 + c7 > a6 b + b6 c + c6 a > a5 bc + b5 ca + c5 ab > a4 b2 c + b4 c2 a + c4 a2 b >
a3 b2 c2 + b3 c2 a2 + c3 a2 b2 ;
(2) a7 + b7 + c7 > a5 b2 + b5 c2 + c5 a2 > a3 b3 c + b3 c3 a + c3 a3 b > a3 b2 c2 + b3 c2 a2 + c3 a2 b2 ;
(3) a7 + b7 + c7 > a4 b3 + b4 c3 + c4 a3 > a3 b3 c + b3 c3 a + c3 a3 b > a3 b2 c2 + b3 c2 a2 + c3 a2 b2 ;
(4) a6 b + b6 c + c6 a > a3 b3 c + b3 c3 a + c3 a3 b > a3 b2 c2 + b3 c2 a2 + c3 a2 b2 ;
(5) a5 bc + b5 ca + c5 ab > a3 b3 c + b3 c3 a + c3 a3 b.

4.9.2. a3 (b2 + c2 )2 + b3 (c2 + a2 )2 + c3 (a2 + b2 )2 > abc ab(a + b)2 + bc(b + c)2 + ca(c + a)2 ,
dla a, b, c > 0. ([OM] USA 2009).

4.9.3. a8 + b8 + c8 > a2 b2 c2 (ab + bc + ca). ([Dlt] 12/1978).

4.9.4. a6 b6 + b6 c6 + c6 a6 + 3a2 b4 c4 > 2(a3 + b3 + c3 )a3 b3 c3 , dla a, b, c > 0. ([Kw] 3/2003 17-18).

oooooooooooooooooooooooooooooooooooooooooooooooooooooooooooooooooooooo
4.10 Jednorodne nierówności wielomianowe trzech zmiennych stopnia n
oooooooooooooooooooooooooooooooooooooooooooooooooooooooooooooooooooooo
4.10.1. (ab)2n+3 + (bc)2n+3 + (ca)an+3 > (abc)n+2 (an + bn + cn ), dla n > 0 i a, b, c > 0.
(M. S. Klamkin, [Crux] z.2734).

4.10.2. an + bn + cn + (a + b + c)n > (a + b)n + (b + c)n + (c + a)n , dla n ∈ N, a, b, c > 0.


([Mat] 4/2004 z.1601).
U. ([Mat] 4/2004). Nierówność taka zachodzi również w przypadku gdy n jest liczbą rzeczywistą
taką, że 0 < n 6 1 lub n > 2. Natomiast jeśli 1 < n < 2, to an + bn + cn + (a + b + c)n <
(a + b)n + (b + c)n + (c + a)n . ⊠
58 Nierówności 4. Jednorodne nierówności wielomianowe

oooooooooooooooooooooooooooooooooooooooooooooooooooooooooooooooooooooo
4.11 Jednorodne nierówności wielomianowe czterech zmiennych
oooooooooooooooooooooooooooooooooooooooooooooooooooooooooooooooooooooo

4.11.1. Dla dodatnich liczb rzeczywistych a, b, c, d zachodzą nierówności:


(1) (a + b + c + d)2 6 3(a2 + b2 + c2 + d2 ) + 6ab, ([OM] Polska 1998/1999);

(2) a3 + b3 + c3 + d3 > abc + abd + acd + bcd, ([Sup]);

(3) (a + b + c + d)3 6 4(a3 + b3 + c3 + d3 ) + 24(bcd + cda + dab + abc), ([OM] Polska 2002/2003);
3
(4) (a + b + c + d) > 16(bcd + cda + dab + abc), ([Pkh] s.19);

(5) a + b + c + d + 2abcd > a b + a c + a d + b2 c2 + b2 d2 + c2 d2 ,


4 4 4 4 2 2 2 2 2 2

([IMO] Longlist 1976, [Djmp] s.111, [Kw] 3/1979 35);

(6) a4 b + b4 c + c4 d + d4 a > abcd(a + b + c + d), ([MM] 78(4)(2005) 324-325, [Mild]);


5 5 5 5 5
(7) (ac + bd) + (ad + bc) 6 (a + b) (c + d ), ([OM] St Petersburg 1995);

3 2
 
(8) a3 + b3 + c3 + d 6 a2 + b2 + c2 + d 2 a4 + b4 + c4 + d4 , (Patrz 2.6.5);
 2   
ab + ac + ad + bc + bd + cd a+b+c+d abc + bcd + cda + dab
(9) > ,
6 4 4
([Zw] 1999);
 3  2
ab + ac + ad + bc + bd + cd abc + abd + acd + bcd
(10) > , ([OM] Polska 1989).
6 4

4.11.2. (a + b + c + d)2 > 8(ac + bd), gdy a < b < c < d. ([OM] Norwegia 1993).

4.11.3. −1 6 xy + yz + zt + tx 6 0, gdy x + y + z + t = 0 oraz x2 + y 2 + z 2 + t2 = 1.


([AuP] 1996).

4.11.4. 25(ab + cd)2 > 16(a2 + d2 )(b2 + c2 ), dla 2 6 a, b, c, d 6 4. ([OM] St Petersburg 2000).

4.11.5. Jeśli a, b, c, d > 0, a5 + b5 6 1 i c5 + d5 6 1, to a2 c3 + b2 d3 6 1. ([MOc] 2001).


⊡⊡⊡⊡⊡⊡⊡⊡⊡⊡⊡⊡⊡⊡⊡⊡⊡⊡⊡⊡⊡⊡⊡⊡⊡⊡⊡⊡⊡⊡⊡⊡⊡⊡⊡⊡⊡⊡⊡⊡⊡⊡⊡⊡⊡⊡⊡⊡⊡⊡⊡⊡
5 Niejednorodne nierówności wielomianowe
⊡⊡⊡⊡⊡⊡⊡⊡⊡⊡⊡⊡⊡⊡⊡⊡⊡⊡⊡⊡⊡⊡⊡⊡⊡⊡⊡⊡⊡⊡⊡⊡⊡⊡⊡⊡⊡⊡⊡⊡⊡⊡⊡⊡⊡⊡⊡⊡⊡⊡⊡⊡

oooooooooooooooooooooooooooooooooooooooooooooooooooooooooooooooooooooo
5.1 Nierówności wielomianowe n zmiennych
oooooooooooooooooooooooooooooooooooooooooooooooooooooooooooooooooooooo

5.1.1. Niech x1 , . . . , xn > 0 i x1 + · · · + xn = 1. Wtedy:


1
(1) x21 + x22 + · · · + x2n > , ([Siw] 126);
n
1
(2) 1x21 + 2x22 + · · · + nx2n > √ , ([Mock] 1997(4));
2 n
X 1
(3) xi xj (xi + xj ) 6 , ([IMO] Shortlist 1991, [Djmp] s.257(555));
i<j
4
4
(4) x21 x2 + x2 x3 + · · · + x2n x1 6 , dla n > 3, ([MM] 4/ z.1292, [OM] Chiny 1994);
27
1
(5) x1 x2 · · · xn (x21 + x22 + · · · + x2n ) 6 n+1 , ([MM] 63(3)(1990) 192);
n
(6) xm m m
1 + x2 + · · · + xn > n
m−1
, dla m ∈ N. ([ME] 9(2)(2004) z.196).

5.1.2. Jeśli x1 , . . . , xn > 0 i x1 + · · · + xn = 10 oraz x21 + x22 + · · · + x2n > 20, to

x31 + x32 + · · · + x3n > 40. ([TT], [MC] 18(2)(2005) s.24).

5.1.3. (x1 + x2 + · · · + xn + 1)2 > 4(x21 + x22 + · · · + x2n ), dla x1 , . . . , xn ∈ [0, 1]. ([Kw] 2/1997 37).

5.1.4. Jeśli n > 3 oraz x1 , . . . , xn ∈ [0, 1], to

x1 + x2 + · · · + xn − (x1 x2 + x2 x3 + · · · + xn−1 xn + xn x1 ) 6 [n/2].

([Kw] 8/1979 44).

5.1.5. x21 + x22 + · · · + x2n 6 −nmM , dla x1 + · · · + xn = 0, gdzie m = min(x1 , . . . , xn ),


M = max(x1 , . . . , xn ). ([IMO] Shortlist 1972, [Djmp] s.89(384)).
D. ([Djmp] s.384). Ponieważ m 6 xi 6 M , więc (M − xi )(m − xi ) 6 0, dla i = 1, . . . , n. Zatem:
X X X X
06 (M − xi )(m − xi ) = nmM − (n + M ) xi + x2i = nmM + x2i . ⊠

n n
! n
!
X
2 4 X X
5.1.6. 1 + (xi + yi ) 6 1+ x2i 1+ yi2 , dla x1 , . . . , xn , y1 , . . . , yn ∈ R.
i=1
3 i=1 i=1
([IMO] Shortlist 1970, [Djmp] 73(374)).

59
60 Nierówności 5. Niejednorodne nierówności wielomianowe

5.1.7. Jeśli x1 , . . . , xn są liczbami rzeczywistymi takimi, że 0 < x1 6 x2 6 · · · 6 xn , to liczba

xk2 (x1 − x3 ) + xk3 (x2 − x4 ) + · · · + xk1 (xn − x2 )

jest nieujemna dla k > 1 i niedodatnia dla 0 < k < 1 ([Kw] 5/1995 M1485).

1
5.1.8. (1 + x1 )(1 + x2 ) · · · (1 + xn ) < 2, dla x1 , . . . , xn > 0, x1 + x2 + · · · + xn 6 .
2
([Kw] 1/2009 57,63).

n
Y n
Y
5.1.9. (x2i + 1) > (xi xi+1 + 1), dla x1 , . . . , xn > 0, xn+1 = x1 . ([MC] 12(1999) 32).
i=1 i=1

n
Y n
Y
5.1.10. (x3i + 1) > (xi x2i+1 + 1), dla x1 , . . . , xn > 0 przy czym xn+1 = x1 . ([Ko03] 113).
i=1 i=1

5.1.11. Niech x1 , . . . , xn , gdzie n > 4, będą parami różnymi liczbami rzeczywistymi takimi,
n
X n
X
że: xi = 0, x2i = 1. Dowieść, że spośród tych liczb można wybrać takie cztery różne
i=1 i=1
liczby a, b, c, d, aby zachodziły nierówności
n
X
a + b + c + nabc 6 x3i 6 a + b + d + nabd.
i=1

([OM] Polska 1993/1994).

oooooooooooooooooooooooooooooooooooooooooooooooooooooooooooooooooooooo
5.2 Nierówności wielomianowe jednej zmiennej
oooooooooooooooooooooooooooooooooooooooooooooooooooooooooooooooooooooo

5.2.1. x(x + 1)(x + 2)(x + 3) > −1, dla x ∈ R. ([Ko00]).

5.2.2. (x3 + x2 + 3)2 > 4x3 (x − 1)2 , dla x ∈ R. ([Berk] 2c/93, [OM] Mołdawia 2000).

1
5.2.3. x4 − x + > 0, dla x ∈ R. ([OM] ZSRR 1990).
2

5.2.4. Jeśli x5 − x3 + x = p > 0, to x6 > 2p − 1. ([Berk] 3b/92).

5.2.5. xn + n − 1 > nx, dla x > 0, n ∈ N. ([Mit2] s.20).

oooooooooooooooooooooooooooooooooooooooooooooooooooooooooooooooooooooo
5.3 Nierówności wielomianowe dwóch zmiennych
oooooooooooooooooooooooooooooooooooooooooooooooooooooooooooooooooooooo

5.3.1. x2 + y 2 > xy + x + y − 1, dla x, y ∈ R.

5.3.2. x2 − 3xy + 2y 2 + x − 2y + 3 > 0, dla x, y ∈ R. ([Kw] 2/1997 37).


Nierówności 5. Niejednorodne nierówności wielomianowe 61

5.3.3. x4 + y 4 + 8 > 8xy, dla x, y ∈ R.

5.3.4. x2 y 2 (x2 + y 2 ) 6 2, dla x, y > 0, x + y 6 2. ([OM] Irlandia 2000).

5.3.5. x3 y 3 (x3 + y 3 ) 6 2, dla x, y > 0, x + y = 2. ([OM] Indie 2002).

5.3.6. (1 − xm )n + (1 − y n )m > 1, dla x, y > 0, x + y = 1, n > 2, m > 2.


([OM] Polska 1993/1994, [MOc] z.491).

n
Y n
Y
5.3.7. (xi + y i−1 ) 6 (xi + y n−i ), dla x, y > 1. ([MC] 12(1999) 32).
i=1 i=1

5.3.8. Jeśli f (x) = ax2 + bx + c ∈ R[x], gdzie a, b, c > 0, to

f (xy)2 6 f (x)2 f (y)2

dla x, y ∈ R. ([OM] Rosja 1997).

5.3.9. Załóżmy, że wielomian f ∈ R[x] ma wszystkie współczynniki nieujemne. Wtedy:

f (xy)2 6 f (x2 )f (y 2 ),

dla wszystkich x, y ∈ R. ([Kw] 2/1998 M1609).

oooooooooooooooooooooooooooooooooooooooooooooooooooooooooooooooooooooo
5.4 Nierówności wielomianowe trzech zmiennych
oooooooooooooooooooooooooooooooooooooooooooooooooooooooooooooooooooooo

5.4.1. Jeśli a, b, c > 0, to:


(1) a2 + b2 + c2 + 2abc + 1 > 2(ab + bc + ca), (D.Grinberg, [MaOD] 32, [Pkh] s.164);

(2) a2 + b2 + c2 + 2abc + 3 > (1 + a)(1 + b)(1 + c), ([AnC]);

(3) (a2 + 2)(b2 + 2)(c2 + 2) > 9(ab + bc + ca), ([A-P] 2004, [LeH2]);

4
(4) (a2 + 3)(b2 + 3)(c2 + 3) > (3ab + 3bc + 3ca + abc)2 , ([Math] 2006);
27
(5) (a5 − a2 + 3)(b5 − b2 + 3)(c5 − c2 + 3) > (a + b + c)3 , ([OM] USA 2004);

(6) (b3 + a)(c3 + b)(a3 + c) > 125abc, gdy a, b, c > 2, ([OM] St Petersburg 1995);

(7) (abc)2 −2abc+2 > (z 2 −2a+2)(b2 −2b+2)(c2 −2c+2), gdy a, b, c > 1, ([OM] Chiny 2009).


5.4.2. a4 + b4 + c2 > 2 2abc, dla a, b, c > 0. ([OM] ZSRR 1992).

D. Wykorzystujemy tylko nierówność x2 + y 2 > 2xy :


√ √
(a4 + b4 ) + c2 > 2a2 b2 + c2 = ( 2ab)2 + c2 > 2 2abc. ⊠
62 Nierówności 5. Niejednorodne nierówności wielomianowe

5.4.3. Dla a, b, c ∈ R :.
3 4
(1) (a + b4 + c4 ) + 24 > 4a2 b + 4b2 c + 4c2 a,
2
3 4
(2) (a + b4 + c4 + 1) > ab2 + bc2 + ca2 . ([OM] St Petersburg 1996).
4

5.4.4. Niech a, b, c, A, B, C > 0. Jeśli a + A = b + B = c + C = k, to aB + bC + cA < k 2 .


([OM] ZSRR 1987).

5.4.5. a2 b2 + b2 c2 + c2 a2 + 3 > 6abc, gdy a + b + c = 0. ([OM] St Petersburg 1995).

5.4.6. ab + bc + ca 6 0, gdy a + b + c = 0. ([OM] Szwecja 1988).

5.4.7. Jeśli a + b + c = 0, to a3 + b3 + c3 > 0 ⇐⇒ a5 + b5 + c5 > 0. ([OM] W.Brytania 2004).

5.4.8. Jeśli a, b, c > 0 i a + b + c = 1, to:


(1) ab + bc + ca > 3abc;
7
(2) 0 6 ab + bc + ca − 2abc 6 , ([IMO] 1984, [Crux] 1998 s.164);
27
(3) 7(ab + bc + ca) 6 2 + 9abc, ([OM] Anglia 1999);
(4) 4(ab + bc + ca) 6 1 + 9abc, ([Crux] 1998 s.164);

(5) ab + bc + ca > 4(a2 b2 + b2 c2 + c2 a2 ) + 5abc, ([OM] Serbia i Czarnogóra 2006);

(6) ab + bc + ca 6 a + b + c + 6abc 6 a + b2 + c2 6 2(a3 + b3 + c3 ) + 3abc,


3 3 3 2

(E. T. H. Wang, [Crux] z.1215);

(7) ab + bc + ca > 8(a2 + b2 + c2 )(a2 b2 + b2 c2 + c2 a2 ), ([Math] 2006);

(8) 5(a2 + b2 + c2 ) 6 6(a3 + b3 + c3 ) + 1, ([AnC]);

1
(9) a3 + b3 + c3 + 6abc > , ([Mild]);
4
2
(10) a3 + b3 + c3 + 3abc > , ([Ko03] 14);
9
9
(11) a3 + b3 + c3 + 4abc 6 , ([Mild]);
32
4
(12) a2 b + b2 c + c2 a 6 , ([OM] Chiny 1992, [OM] Kanada 1999);
27
(13) 2 6 (1 − a2 )2 + (1 − b2 )2 + (1 − c2 )2 6 (1 + a)(1 + b)(1 + c), ([AuP] 2000);

(14) ab(a + b)2 + bc(b + c)2 + ca(c + a)2 > 4abc, ([MG] 483(1994) 367);

(15) 10(a3 + b3 + c3 ) − 9(a5 + b5 + c5 ) > 1, ([OM] Chiny 2005, [BiYel]).

1
5.4.9. (1 − a)(1 − b)(1 − c) > , dla a, b, c > 0 i a + b + c 6 12 . ([OM] Moskwa 1994/1995).
2
Nierówności 5. Niejednorodne nierówności wielomianowe 63

5.4.10. Niech a, b, c > 0, a + b + c = 2. Wtedy:


1
(1) a2 b + b2 c + c2 a 6 a3 + b3 + c3 6 1 + (a4 + b4 + c4 ), ([Math] 2006);
2
2 2 2 2 2 2
(2) a b + b c + c a 6 2, ([Pkh] s.23).

5.4.11. Niech a, b, c > 0, a + b + c = 3. Wtedy:


27
(1) ab2 + bc2 + ca2 6 , ([OM] Hong Kong 2002);
8
(2) (a + b )(b + c )(c + a2 ) 6 13 + abc, ([Pkh] s.205);
2 2

(3) (a + b2 )(b + c2 )(c + a2 ) 6 13 + abc(1 − 2abc), ([Pkh] s.207);

(4) (a + b2 )(b + c2 )(c + a2 ) 6 13, ([Pkh] s.207);

(5) (1 + a + a2 )(1 + b + b2 )(1 + c + c2 ) > 9(ab + bc + ca), ([Pkh] s.125);

(6) (a2 − ab + b2 )(b2 − bc + c2 )(c2 − ca + a2 ) 6 12, (Pham Kim Hung [Pkh] s.35).

5.4.12. Jeśli a, b, c > 0, abc = 1, to:


(1) a2 + b2 + c2 > a + b + c, ([Mild]);

(2) 2(a2 + b2 + c2 ) + a + b + c > 6 + ab + bc + ca, ([OM] Białoruś 2006);

(3) (ab − b + 1)(bc − c + 1)(ca − a + 1) 6 1, ([IMO] 2000, [Khr1]);

(4) ab2 + bc2 + ca2 > ab + bc + ca, (P. E. Tsaoussoglou, [Crux] z.2886);

(5) (a + b)(b + c)(c + a) > 4(a + b + c − 1), ([AnC]).

5.4.13. x + y + z > xy + yz + zx, gdy x, y, z > 0 i xy + yz + zx + xyz = 4. ([OM] Indie 1998).


5.4.14. xy + yz + zx > 9(x + y + z), dla x, y, z > 0 oraz x + y + z = xyz.
([OM] Białoruś 1996,[Pa97]).

5.4.15. 10x2 + 10y 2 + z 2 > 4, gdy x, y, z > 0 i xy + yz + zx = 1. ([Pkh] s.97).

5.4.16. x2 + y 2 + z 2 + 3xyz > 6, gdy x, y, z > 0 i xy + yz + zx = 3. ([ME] 13(1)(2008)).

5.4.17. (1 + x2 )(1 + y 2 )(1 + z 2 ) > 8, gdy x, y, z > 0 i xy + yz + zx = 3. (M.Rozenberg, [Pkh] s.50).

5.4.18. x3 + y 3 + z 3 > x + y + z, gdy x, y, z > 0 i xy + yz + zx 6 3xyz. ([OM] Rumunia 1999).

5.4.19. x(1 − y) + y(1 − z) + z(1 − x) < 1, dla x, y, z ∈ (0, 1). ([Kw] 2/1997 37).

1
5.4.20. xyz(1 − x)(1 − y)(1 − z) 6 , dla x, y, z ∈ [0, 1]. ([Mat] 1/1985).
64
5.4.21. 2(x3 + y 3 + z 3 ) − (x2 y + y 2 z + z 2 x) 6 3, dla x, y, z ∈ [0, 1]. ([Kw] 2/1997 35).

5.4.22. x3 + y 3 + z 3 6 5xyz, dla x, y, z ∈ [1, 2]. ([Pkh] s.75).


64 Nierówności 5. Niejednorodne nierówności wielomianowe

5.4.23. (x − y)3 + (y − z)3 + (z − x)3 > 0, gdy x < y < z. ([MaOD] 34).

5.4.24. (a − b)2 + (b − c)2 + (c − a)2 6 3, gdy a2 + b2 + c2 = 1. ([OM] Ukraina 1993).

5.4.25. a + b + c 6 abc + 2, gdy a2 + b2 + c2 = 2, a, b, c ∈ R. ([IMO] Shortlist 1987).

5.4.26. Jeśli a2 + b2 + c2 = 3, a, b, c > 0, to:


(1) a4 + b4 + c4 + a + b + c > 6,[Ko03] 41;
(2) (a + ab + c)2 + (b + ca + a)2 + (c + ab + b)2 6 27, ([Zw] 2006;
3 2 3 2 3 2
(3) a + b + b c + c a 6 3, ([Pkh] s.236).

5.4.27. 2(a + b + c) − abc 6 10, gdy a2 + b2 + c2 = 9. ([OM] Wietnam 2002, [Mild]).

oooooooooooooooooooooooooooooooooooooooooooooooooooooooooooooooooooooo
5.5 Nierówności wielomianowe dla boków trójkąta
oooooooooooooooooooooooooooooooooooooooooooooooooooooooooooooooooooooo
5.5.1. Jeśli a, b, c są długościami boków trójkąta, którego obwód wynosi 2, to

a2 + b2 + c2 + 2abc < 2.

([OM] Węgry 90, [Pa97]).

5.5.2. Liczby a, b, c są długościami boków trójkąta.


(1) 2(ab + bc + ca) > a2 + b2 + c2 .
(2) 3(ab + bc + ca) 6 (a + b + c)2 < 4(ab + bc + ca). ([Kw] 2/1997 37).

5.5.3. Liczby a, b, c są długościami boków trójkąta.


(1) (a2 + b2 + c2 )(a + b + c) > 2(a3 + b3 + c3 ). ([Kw] 2/1997 37).

(2) a3 + b3 + c3 + 3abc > 2(a + b)c2 . ([Kw] 2/1997 37).

(3) a3 + b3 + 3abc > c3 . ([OM] Moskwa 2002, [TTjs] 2002).

(4) (a + b − c)(b + c − a)(c + a − b) 6 abc. ([Kw]).

(5) 8(a + b − c)(b + c − a)(c + a − b) 6 (a + b)(b + c)(c + a). ([IMO] Longlist 1970, [Djmp] s.70).

(6) a2 (b + c − a) + b2 (c + a − b) + c2 (a + b − c) 6 3abc. ([IMO] 6, [MoP] 34).

(7) a2 (2b + 2c − a) + b2 (2c + 2a − b) + c2 (2a + 2b − c) > 9abc. ([Ko03] 40).

(8) a(b − c)2 + b(c − a)2 + c(a − b)2 + 4abc > a3 + b3 + c3 . ([Kw] 2/1997 37).

5.5.4. Jeśli a, b, c są bokami trójkąta, to

a3 + b3 + c3 6 (a + b − c)3 + (b + c − a)3 + (c + a − b)3 .

D. Wynika z 1.5.5 dla wypukłej funkcji f : (0, ∞) → R, f (x) = x3 . Patrz 1.5.8. ⊠


Nierówności 5. Niejednorodne nierówności wielomianowe 65

5.5.5. Liczby a, b, c są długościami boków trójkąta.


(1) a3 b + b3 c + c3 a > a2 b2 + b2 c2 + c2 a2 . ([Kw]).
2 2 2
(2) a b(a − b) + b c(b − c) + c a(c − a) > 0. ([IMO] 1983, [LeH2])

5.5.6. Niech a, b, c > 0. Liczby te są długościami boków pewnego trójkąta wtedy i tylko wtedy,
gdy
(a2 + b2 + c2 )2 > 2(a4 + b4 + c4 ).
([OM] Chiny 1988, [KoM] 2000(10) B3394).

5.5.7. Niech n > 3 oraz a1 , . . . , an > 0. Jeśli

(a21 + · · · + a2n )2 > (n − 1)(a41 + · · · + a4n ),

to każde trzy liczby spośród a1 , . . . , an są długościami boków trójkąta. ([OM] Chiny 1988).

5.5.8. Jeśli 5abc > a3 + b3 + c3 , a, b, c > 0, to istnieje trójkąt o bokach a, b, c.


([OM] Czechy-Słowacja 2000).

5.5.9. Jeśli a, b, c są bokami trójkąta, to

an + bn + cn 6 (a + b − c)n + (b + c − a)n + (c + a − b)n ,

dla n ∈ N.

D. Wynika to z nierówności 1.5.5 zastosowanej dla wypukłej funkcji f : (0, ∞) → R, f (x) = xn .


Patrz 1.5.8. ⊠
⋆ L. Carlitz, Some inequalities for a triangle, [Mon] 71(8)(1964) 881-885.
M. S. Klamkin, Extensions of some geometric inequalities, [MM] 49(1)(1976) 28-30.
M. S. Klamkin, Simultaneous triangle inequalities, [MM] 60(4)(1987) 236-237.
Hojoo Lee, Geometric inequalities, [LeH2] 1-11.
oooooooooooooooooooooooooooooooooooooooooooooooooooooooooooooooooooooo
5.6 Nierówności wielomianowe czterech zmiennych
oooooooooooooooooooooooooooooooooooooooooooooooooooooooooooooooooooooo

5.6.1. 2(a3 + 1)(b3 + 1)(c3 + 1)(d3 + 1) > (1 + abcd)(1 + a2 )(1 + b2 )(1 + c2 )(1 + d2 ), dla
a, b, c, d > 0. (Vasile Cirtoaje, [Mild]).

5.6.2. Jeśli a, b, c, d są liczbami rzeczywistymi takimi, że a + b + c + d = 0,

a3 + b3 + c3 + d3 > 0 ⇐⇒ a5 + b5 + c5 + d5 > 0. ([OM] W.Brytania 2004).

5.6.3. Jeśli a, b, c, d > 0, a + b + c + d = 1, to:


1
(1) 6(a3 + b3 + c3 + d3 ) > (a2 + b2 + c2 + d2 ) + , ([OM] Hong Kong 2005);
8
1
(2) (a + c)(b + d) + 2(ac + bd) 6 , ([Math] 2006);
2
1 176
(3) abc + bcd + cda + dab 6 + abcd, ([IMO] Shortlist 1993, [Djmp] s.276(580)).
27 27
66 Nierówności 5. Niejednorodne nierówności wielomianowe

5.6.4. ab(b + c) + bc(c + d) + cd(d + a) + da(a + b) 6 4, dla a, b, c, d > 0, a + b + c + d = 3.


([Pkh] s.152).

5.6.5. Jeśli a, b, c, d > 0, a + b + c + d = 4, to:


(1) a2 + b2 + c2 + d2 − 4 > 4(a − 1)(b − 1)(c − 1)(d − 1); ([Pkh] s.151);

(2) (1+a4 )(1+b4 )(1+c4 )(1+d4 ) > (1+a3 )(1+b3 )(1+c3 )(1+d3 ); (Pham Kim Hung [Pkh] s.190).

5.6.6. 25(ab + cd)2 > 16(a2 + d2 )(b2 + c2 ), dla a, b, c, d ∈ [2, 4]. ([OM] St Petersburg 2000).

5.6.7. (1 + a2 )(1 + b2 )(1 + c2 )(1 + d2 ) > (a + b + c + d)2 , dla a, b, c, d > 0, abcd = 1.


([Pkh] s.156).

5.6.8. Niech a, b, c, d będą dodatnimi liczbami rzeczywistymi. Wykazać, że co najmniej jedna


z następujących trzech nierówności nie jest prawdziwa:

a + b < c + d,
(a + b)(c + d) < ab + cd, ([WaJ] 118(69)).

(a + b)cd < ab(c + d).


oooooooooooooooooooooooooooooooooooooooooooooooooooooooooooooooooooooo
5.7 Nierówności z liczbami an + bn
oooooooooooooooooooooooooooooooooooooooooooooooooooooooooooooooooooooo
a6 + b6
5.7.1. > 3a2 b2 − 4. ([StaZ] 64).
2
5.7.2. 8(a4 + b4 ) > (a + b)4 . ([Kw] 2/1997 37).

5.7.3. 2(a3 + b3 )2 > (a2 + b2 )3 , dla a, b > 0. ([Ko00]).


 n
an + bn a+b
5.7.4. > , dla a, b > 0, n ∈ N. (1.4.4, [Str72] 50).
2 2

a + b a2 + b2 a3 + b3 a6 + b6
5.7.5. · · 6 , dla a, b > 0. ([Str1] 35, [B-rs] 124).
2 2 2 2
an + bn am + bm an+m + bn+m
5.7.6. · 6 , gdy n + m jest parzyste i a, b > 0. ([Str1] s.77).
2 2 2
5.7.7. (am − bm )(an + bn ) > (an − bn )(am + bm ), dla a > b > 0, m > n. ([Bedn] 66).

5.7.8. (am+p + bm+p )(an + bn ) > (an+p + bn+p )(am + bm ), dla a > 0, b > 0, a 6= b, m > n,
p > 0. ([Bedn] 66).
n
Y
5.7.9. (ak + bk )2 > (an+1 + bn+1 )n , dla a, b > 0, n ∈ N. ([OM] Polska 1989).
k=1

mm+1 + nn+1
5.7.10. Niech a = , gdzie m, n ∈ N. Wtedy am +an ­ mm +nn . ([OM] USA 1991).
mm + nn
Nierówności 5. Niejednorodne nierówności wielomianowe 67

5.7.11. Niech a, b, c, d > 0, a + b = c + d. Jeśli istnieje m ∈ N takie, że am + bm > cm + dm ,


to
an + bn > cn + dn ,
dla każdego naturalnego n > 1. ([Dlt] 8/2001 z.420).

oooooooooooooooooooooooooooooooooooooooooooooooooooooooooooooooooooooo
5.8 Nierówności wielomianowe dla liczb całkowitych
oooooooooooooooooooooooooooooooooooooooooooooooooooooooooooooooooooooo
5.8.1. 2(3n − 1)n > (3n + 1)n , dla n ∈ N. ([Crux] 1999 s.442).

5.8.2. 1 · 3 · 5 · · · (2n − 1) < nn , dla n > 1. ([San2] 10).

5.8.3.
(1) (2n + 1)n > (2n)n + (2n − 1)n , dla n > 2. ([Mon] 76(1)(1964) E1624, [WaJ] 462(87)).

(2) (2n + 2)n < (2n)n + (2n + 1)n , dla n > 2N . ([Mon] 76(1)(1964) E1624).

5.8.4. Jeśli n, m są liczbami naturalnymi takimi, że n > m2 > 16, to 2n > nm .


([Crux] 1997 s.375).

5.8.5. Jeśli 0 < x1 6 x2 6 · · · 6 xn są liczbami naturalnymi, to


 2
n n
X 2(n2 − 1) X
 |xi − xj | 6 (xi − xj )2 .
i,j=1
3 i,j=1

Równość zachodzi wtedy i tylko wtedy, gdy x1 , . . . , xn jest ciągiem arytmetycznym.


([IMO] Shortlist 2003).

Następne nierówności będą dla parami różnych liczb całkowitych (głównie naturalnych).

5.8.6. Jeśli a1 , . . . , an (gdzie n > 2) są parami różnymi liczbami całkowitymi, to

|a1 − a2 | + |a2 − a3 | + · · · + |an−1 − an | + |an − a1 | > 2n − 2.

D. Bez straty ogólności możemy założyć, że an jest najmniejszą z liczb a1 , . . . , an i ponadto, że


an = 0. Niech bi = |ai − ai+1 |, dla i = 1, . . . , n, gdzie an+1 = a1 . Załóżmy, że ak jest największą
spośród wszystkich liczb a1 , . . . , an . Ponieważ te liczby są parami różne, więc ak > n − 1. Mamy więc:

n−1 6 ak = ak − an = |ak − an | = |(ak − ak+1 ) + (ak+1 − ak+2 ) + · · · + (an−1 − an )|


6 |ak − ak+1 | + |ak+1 − ak+2 | + · · · + |an−1 − an |
= bk + bk+1 + · · · + bn−1 oraz

n − 1 6 ak = ak − an = |an − ak | = |(an − an+1 ) + (a1 − a2 ) + (a2 − a3 ) · · · + (ak−1 − ak )|


6 |an − a1 | + |a1 − a2 | + |a2 − a3 | · · · + |ak−1 − ak |
= bn + b1 + b2 + · · · + bk−1 .

Zatem b1 + · · · + bn = (bn + b1 + b2 + · · · + bk−1 ) + (bk + bk+1 + · · · + bn−1 ) > 2n − 2. ⊠


68 Nierówności 5. Niejednorodne nierówności wielomianowe

5.8.7. Jeśli a, b, c, d, e, f są parami różnymi liczbami całkowitymi, to

(a − b)2 + (b − c)2 + (c − d)2 + (d − e)2 + (e − f )2 + (f − a)2 > 18. ([MOc] 2002 z.172).

Powyższe zadanie jest szczególnym przypadkiem następującego stwierdzenia.

5.8.8. Jeśli a1 , . . . , an (gdzie n > 2) są parami różnymi liczbami całkowitymi, to

(a1 − a2 )2 + (a2 − a3 )2 + · · · + (an−1 − an )2 + (an − a1 )2 > 4n − 6.

Ponadto, dla każdego n > 2 istnieją takie parami różne liczby naturalne a1 , . . . , an , dla których
ta nierówność jest równością.

D. (A. Critch, [MOc] 2002, rozwiązanie zadania 172). Oznaczmy: bi = |ai − ai+1 |, dla i = 1, . . . , n,
gdzie an+1 = a1 . Liczby b1 , . . . , bn są nieujemne i wiemy (patrz 5.8.6), że b1 + · · · + bn > 2n − 2.
Korzystamy z nierówności pomiędzy średnią kwadratową i średnią arytmetyczną (patrz 2.2.1) i mamy:
r
b21 + · · · + b2n b1 + · · · + bn 2n − 2
> > .
n n n
4n2 −8n+4 4
Stąd b21 + · · · + b2n > n = 4n − 8 + n > 4n − 8. Zauważmy, że

b21 + · · · + b2n = (a1 − a2 )2 + · · · + (an − a1 )2 ≡ (a1 − a2 ) + · · · + (an − a1 ) = 0 (mod 2),

a zatem suma b21 + · · · + b2n jest liczbą parzystą. Z nierówności b21 + · · · + b2n > 4n − 8 wynika więc, że
b21 + · · · + b2n > 4n − 6.
Niech n > 2 i niech (a1 , . . . , an ) będzie taką permutacją zbioru {1, 2, . . . , n}, w której najpierw
występują kolejne liczby nieparzyste 1, 3, 5 · · · , a następnie kolejne liczby parzyste, od największej do
najmniejszej. Dla przykładu, gdy n = 6 to (a1 , . . . , a6 ) = (1, 3, 5, 6, 4, 2), a jeśli n = 7, to (a1 , . . . , a7 ) =
(1, 3, 5, 7, 6, 4, 2). Dla takiego ciągu (a1 , . . . , an ) rozważana nierówność staje się równością. ⊠

5.8.9. Jeśli a1 , . . . , an są parami różnymi liczbami naturalnymi, to


2n + 1
a21 + a22 + · · · + a2n > (a1 + · · · + an ). ([OM] Rumunia 1999, [AnAC] 237).
3
5.8.10. Jeśli a1 , . . . , an są parami różnymi liczbami naturalnymi, to

a31 + a32 + · · · + a3n > (a1 + a2 + · · · + an )2 . ([OM] Rosja 1998)

5.8.11. Jeśli a1 , . . . , an są parami różnymi liczbami naturalnymi, to

(a71 + a72 + · · · + a7n ) + (a51 + a52 + · · · + a5n ) > 2(a31 + a32 + · · · + a3n )2 .

([OM] Hong Kong 1999).


Nierówności 5. Niejednorodne nierówności wielomianowe 69

Udowodnimy teraz następujące twierdzenie.

5.8.12. Niech n > 2 i niech a1 , . . . , an będą liczbami rzeczywistymi takimi, że

0 < a1 < a2 < · · · < an .

Niech h : (0, ∞) → (0, ∞) będzie ściśle malejącą funkcją. Ponadto, niech


      
H x1 , . . . , xn = a1 + h(x1 ) a2 + h(x2 ) · · · an + h(xn ) .

Wtedy dla parami różnych liczb naturalnych x1 , . . . , xn zachodzi nierówność


   
H x1 , x2 , . . . , xn 6 H 1, 2, 3, . . . , n .

D. Niech x1 , . . . , xn będą parami różnymi liczbami naturalnymi. Istnieje wtedy taka permutacja σ
zbioru {1, 2, . . . , n}, że
1 6 xσ(1) < xσ(2) < · · · < xσ(n) .
Mamy wtedy nierówności: 1 6 xσ(1) , 2 6 xσ(2) , . . . , n 6 xσ(n) , z których wynika, że

τ (1) 6 x1 , τ (2) 6 x2 , ..., τ (n) 6 xn ,

gdzie τ jest permutacją odwrotną do permutacji σ. Ponieważ funkcja h jest malejąca, więc
   
H x1 , x2 , . . . , xn 6 H τ (1), τ (2), . . . , τ (n) .

Wystarczy zatem udowodnić, że dla każdej permutacji τ zbioru {1, . . . , n} zachodzi nierówność
   
(∗) H τ (1), τ (2), . . . , τ (n) 6 H 1, 2, . . . , n .

Zanim to zrobimy rozpatrzmy najpierw dwie takie permutację, które różnią się tylko na dwóch miej-
scach. Dokładniej, niech p < q będą elementami zbioru {1, . . . , n} i niech α, β będą takimi permuta-
cjami zbioru {1, . . . , n}, że α(k) = β(k) dla wszystkich k różnych od p i różnych od q oraz

α(p) = β(q), α(q) = β(p) i przy tym α(p) > α(q).

Pokażemy, że wtedy
   
(∗∗) H α(1), α(2), . . . , α(n) < H β(1), β(2), . . . , β(n) .

W tym celu zauważmy, że H(β(1), . . . , β(n)) − H(α(1), . . . , α(n)) = A · B, gdzie


Y  
A= ak + h(α(k))
k∈{1,...,n}r{p,q}

oraz      
B = ap + h(β(p)) aq + h(β(q)) − ap + h(α(p)) aq + h(α(q))
Liczba A jest oczywiście większa od zera. Natomiast
B = ap h(β(q)) + aq h(β(p))) − ap h(α(q)) − aq h(α(p))
= ap h(α(p)) + aq h(α(q)) − ap h(h(α(q)) − aq h(α(p))
   
= aq h(α(q)) − h(α(p)) − ap h(α(q)) − h(α(p))
  
= aq − ap ) h(α(q)) − h(α(p)) ,
70 Nierówności 5. Niejednorodne nierówności wielomianowe

a więc B jest również większe od zera. Nierówność (∗∗) została więc wykazana.
Teraz możemy udowodnić nierówność (∗). Niech τ będzie dowolną permutacją zbioru {1, . . . , n}.
Jeśli permutacja τ jest tożsamościowa, to nie ma czego dowodzić. Załóżmy więc, że istnieją w zbiorze
{1, . . . , n} takie dwa elementy p, q, że p < q oraz τ (p) > τ (q). Wtedy zastępujemy permutację τ
nową permutacją τ ′ taką, że τ ′ (p) < τ ′ (q) oraz τ ′ (k) = τ (k) dla k 6∈ {p, q}. Z nową permutacją τ ′
postępujemy podobnie. Proces kontynuujemy tak długo aż dojdziemy do permutacji σ takiej, że σ(1) <
σ(2) < · · · < σ(n), czyli do permutacji tożsamościowej. Przy każdej takiej zamianie wykorzystujemy
nierówność (∗∗) i w ten sposób otrzymujemy nierówność (∗). ⊠

Oto kilka nierówności wynikających z twierdzenia 5.8.12.

5.8.13. Jeśli x, y, z są parami różnymi liczbami naturalnymi, to

(x + 1)(2y + 1)(3z + 1) < 17xyz.

1
 1

1

D. Niech H(x, y, z) = 1 + x 2+ y 3+ z , gdzie x, y, z są parami różnymi liczbami natural-
50
nymi. Wiemy (patrz twierdzenie 5.8.12), że H(x, y, z) 6 H(1, 2, 3) = 3 < 17. Po pomnożeniu przez
xyz otrzymujemy rozważaną nierówność. ⊠

5.8.14. W poniższych tabelkach podano przykłady takich trójek (n, m, c), że nierówność

(1x1 + 1)(2x2 + 1)(3x3 + 1) · · · (nxn + 1) < c · x1 x2 · · · xn

zachodzi dla parami różnych liczb naturalnych większych od m.

n m c n m c n m c n m c n m c
3 0 17 3 1 12 3 2 10 3 3 9 3 4 9
4 0 71 4 1 48 4 2 41 4 3 37 4 4 34
5 0 269 5 1 247 5 2 206 5 3 185 5 4 173
6 0 2272 6 1 1517 6 2 1261 6 3 1130 6 4 1051

5.8.15. Jeśli x, y, z są parami różnymi liczbami naturalnymi, to

(x + 1)(y + 2)(z + 3) < 11xyz.

1 1
 1 1

1 1

D. Niech H(x, y, z) = 3 + x 2 + y 1 + z , gdzie x, y, z są parami różnymi liczbami natu-
16
ralnymi. Wiemy (patrz twierdzenie 5.8.12), że H(x, y, z) 6 H(1, 2, 3) = 9 . Po pomnożeniu przez 6xyz
otrzymujemy: (x + 3)(y + 2)(z + 1) 6 32
3 xyz < 11xyz. ⊠

5.8.16. Jeśli x, y, z, t są parami różnymi liczbami naturalnymi większymi od 3, to

(x + 1)(y + 2)(z + 3)(t + 4) < 5xyzt.

D. Niech x, y, z, t będą parami różnymi liczbami naturalnymi większymi od 3. Niech x = 3 + x1 ,


y = 3 + x2 , z = 3 + x3 oraz t = 3 + x4 . i niech
    
1 1 1 1 1 1 1 1
H(x1 , x2 , x3 , x4 ) = + + + + .
4 3 + x1 3 3 + x2 2 3 + x3 1 3 + x4
Nierówności 5. Niejednorodne nierówności wielomianowe 71

Ponieważ x1 , x2 , x3 , x4 są parami różnymi liczbami naturalnymi, więc z twierdzenia 5.8.12 wynika, że


64
H(x1 , x2 , x3 , x4 ) 6 H(1, 2, 3, 4) = .
315
512
Po pomnożeniu przez 24xyzt otrzymujemy: (x + 4)(y + 3)(z + 2)(t + 1) 6 105 xyzt < 5xyzt. ⊠

5.8.17. W poniższych tabelkach podano przykłady takich trójek (n, m, c), że nierówność

(1 + x1 )(2 + x2 )(3 + x3 ) · · · (n + xn ) < c · x1 x2 · · · xn

zachodzi dla parami różnych liczb naturalnych większych od m.

n m c n m c n m c n m c n m c
3 0 11 3 1 6 3 2 4 3 3 3 3 4 3
4 0 27 4 1 11 4 2 7 4 3 5 4 4 4
5 0 65 5 1 24 5 2 14 5 3 9 5 4 7
6 0 164 6 1 53 6 2 27 6 3 17 6 4 12

oooooooooooooooooooooooooooooooooooooooooooooooooooooooooooooooooooooo
5.9 Różne nierówności wielomianowe
oooooooooooooooooooooooooooooooooooooooooooooooooooooooooooooooooooooo

5.9.1. Niech a 6= b będą liczbami rzeczywistymi. Istnieją wtedy liczby całkowite m, n takie, że
am + bn < 0 oraz an + bm > 0. ([OM] Kanada 1972).

5.9.2. Jeśli a, b, c > 0 i a2 + b2 − ab = c2 , to

(a − c)(b − c) 6 0.

([OM] Moskwa 1997).

5.9.3. Niech a, b ∈ R. Jeśli a + b > 0 i ab > 0, to a > 0, b > 0.

5.9.4. Niech a, b, c ∈ R. Jeśli a + b + c > 0, ab + bc + ca > 0, abc > 0, to a > 0, b > 0,


c > 0. ([Dlt] 8/1994, 4).

5.9.5. Niech a, b, c, d ∈ R. Jeśli a + b + c + d > 0, ab + ac + ad + bc + bd + cd > 0,


abc + abd + acd + bcd > 0 oraz abcd > 0, to a > 0, b > 0, c > 0, d > 0. ([Ko03] 10).
U. Powyższe trzy zadania łatwo uogólnić dla większej liczby danych liczb. ⊠

⋆ S. W. Dvorianinow, E. A. Yasinovyi, Obtaining symmetric inequalities, [Kw] 7/1985 33-36.


A. Jarski, Jak udowodnić nierówność ?, [Kw] 2(1997) 35-37.
L. Pinter, Y. Hegedüs, Ordered sequences of numbers and inequalities, [Kw] 12/1985 14-16.
Naoki Sato, Symmetric polynomial inequalities, [Crux] 2001 529-533.
72 Nierówności 5. Niejednorodne nierówności wielomianowe
⊡⊡⊡⊡⊡⊡⊡⊡⊡⊡⊡⊡⊡⊡⊡⊡⊡⊡⊡⊡⊡⊡⊡⊡⊡⊡⊡⊡⊡⊡⊡⊡⊡⊡⊡⊡⊡⊡⊡⊡⊡⊡⊡⊡⊡⊡⊡⊡⊡⊡⊡⊡
6 Jednorodne nierówności wymierne
⊡⊡⊡⊡⊡⊡⊡⊡⊡⊡⊡⊡⊡⊡⊡⊡⊡⊡⊡⊡⊡⊡⊡⊡⊡⊡⊡⊡⊡⊡⊡⊡⊡⊡⊡⊡⊡⊡⊡⊡⊡⊡⊡⊡⊡⊡⊡⊡⊡⊡⊡⊡

oooooooooooooooooooooooooooooooooooooooooooooooooooooooooooooooooooooo
6.1 Jednorodne nierówności wymierne n zmiennych
oooooooooooooooooooooooooooooooooooooooooooooooooooooooooooooooooooooo
x2 y 2 (x + y)2
6.1.1. + > , dla a, b > 0, x, y ∈ R.
a b a+b
D. ([MaOD] 34).Z oczywistej nierówności (ay − bx)2 > 0 otrzymujemy kolejno:
x2 b2 + y 2 a2 > 2abxy,
x2 ab + x2 b2 + y 2 a2 + y 2 ab > x2 ab + 2abxy + y 2 ab,
x2 b(a + b) + y 2 a(a + b) > ab(x + y)2 .

Wystarczy teraz ostatnią z tych nierówności podzielić stronami przez ab(a + b). ⊠

6.1.2. Jeśli λ1 , . . . , λn > 0 oraz x1 , . . . , xn ∈ R, to

x21 x2 (x1 + · · · + xn )2
+ ··· + n > .
λ1 λn λ 1 + · · · + λn
x1 xn
Równość zachodzi wtedy i tylko wtedy, gdy λ1 = ··· = λn . ([MaOD] 35).

D. ([MaOD] 34-35). Indukcja ze względu na n. Dla n = 1 jest to oczywiste. Niech n > 2 i załóżmy,
że rozważana nierówność jest prawdziwa dla n − 1. Mamy wtedy (na mocy 6.1.1):

x21 x2 x2 (x1 + · · · + xn−1 )2 x2 (x1 + · · · + xn−1 + xn )2


+ · · · + n−1 + n > + n >
λ1 λn−1 λn λ1 + · · · + λn−1 λn λ1 + · · · + λn−1 + λn
i to kończy dowód. ⊠

Podamy teraz kilka zastosowań nierówności 6.1.2.

x1 xn (x1 + · · · + xn )2
6.1.3. + ··· + > , dla dodatnich liczb x1 , . . . , xn , y1 , . . . , yn .
y1 yn x1 y1 + · · · + xn yn
([MaOD] 35).

x1 xn x2 x2 (x1 + · · · + xn )2
D. + ··· + = 1 + ··· + n > .⊠
y1 yn x1 y1 xn yn x1 y1 + · · · + xn yn

 2
x1 xn 1 x1 xn
6.1.4. 2 + · · · + 2 > + ··· + ,
y1 yn x1 + · · · + xn y1 yn
dla dodatnich liczb x1 , . . . , xn , y1 , . . . , yn . ([MaOD] 35).
x21 x2n  2
x x y2 y2 1 x1 xn
D. 21 + · · · + 2n = 1 + · · · + n > + ··· + .⊠
y1 yn x1 xn x1 + · · · + xn y1 yn

73
74 Nierówności 6. Jednorodne nierówności wymierne

1 1 1 n2
6.1.5. + + ··· + > , dla x1 , . . . , xn > 0. ([Mat] 1/1957 71).
x1 x2 xn x1 + · · · + xn
D. Nierówność 6.1.2 dla λ1 = · · · = λn = 1. ⊠

6.1.6. Jeśli x1 , . . . , xn , y1 , . . . , yn są dodatnimi liczbami rzeczywistymi takimi, że x1 + · · · +


xn = y1 + · · · + yn , to

x21 x2n 1
+ ··· + > (x1 + · · · + xn ). ([A-P] 1991).
x1 + y1 xn + yn 2
D. ([MaOD] 36).

x21 x2n (x1 + · · · + xn )2 1


+ ··· + > = (x1 + · · · + xn ). ⊠
x1 + y1 xn + yn (x1 + · · · + xn ) + (y1 + · · · + yn ) 2

Szczególnym przypadkiem nierówności 6.1.6 jest następna nierówność.

x21 x22 x2n 1


6.1.7. + + ··· + > (x1 + · · · + xn ), dla x1 , . . . , xn > 0.
x1 + x2 x2 + x3 xn + x1 2
([Crux] 1998 s. 162).

6.1.8. Jeśli x1 , . . . , xn > 0, to:


1 1 1 n2
(1) + + ··· + > ,
S − x1 S − x2 S − xn (n − 1)S
gdzie S = x1 + · · · + xn , ([Mat] 5/1959 296);
 
1 2 n 1 1
(2) + + ··· + <4 + ··· + ,
x1 x1 + x2 x1 + x2 + · · · + xn x1 xn
([WaJ] 427(86));
    
1 1 1 1 1 1 1 1 1
(3) n x1 + x2 + ··· + xn > 1+x1 + 1+x2 + ··· + 1+xn n+ x1 + x2 + ··· + xn .
([Nord] 1999).

 
1 1
6.1.9. (x1 + · · · + xn ) + ··· + > n2 , dla x1 , . . . , xn > 0. (2.6.1, [MM] 42(3)(1969) 161).
x1 xn
  r r r 3
1 1 1 1 x1 x2 xn
6.1.10. (x1 +x2 +· · ·+xn ) + + ··· + > 3
+ 3
+ ··· + 3
> n2 ,
x1 x2 xn n x2 x3 x1
dla x1 ,. . . ,xn > 0. (M. Bencze, [Crux] z.3086).
 
1 1 1 (a + b)2 2
6.1.11. (x1 + x2 + · · · + xn ) + + ··· + 6 n ,
x1 x2 xn 4ab
dla x1 , . . . , xn ∈ [a, b], gdzie 0 < a < b. ([Kw] 7/1979 25, [Khr2], [Khr1]).

x21 x22 x2n−1 x2n


6.1.12. + + · · · + + 6 n − 1,
x21 + x2 x3 x22 + x3 x4 x2n−1 + xn x1 x2n + x1 x2
dla x1 , . . . , xn > 0. ([IMO] Shortlist 1985, [Djmp] s.192(478), [OM] Polska 1990).
Nierówności 6. Jednorodne nierówności wymierne 75

 
1 1 1 n2
6.1.13. (x21 + x22 + · · · + x2n ) + 2 + ··· + 2 > ,
x1 + x1 x2 x2 + x2 x3
2 xn + xn x1 2
dla x1 , . . . , xn > 0. (T. Mitev, [Crux] z.2937).

x31 x32 x3n 1


6.1.14. + + · · · + > (x1 + x2 + · · · + xn ),
x1 + x1 x2 + x2 x2 + x2 x3 + x3
2 2 2 2 xn + xn x1 + x1
2 2 3
dla x1 , . . . , xn > 0. ([OM] Węgry-Izrael 2003, [KoM] 2004 B3700, [Mild]).

6.1.15. Jeśli xn > xn−1 > · · · > x1 > 0, to:


x1 x2 x2 x3 xn−1 xn xn x1
(1) + +· · ·+ + > x1 +x2 · · ·+xn , ([OM] St Petersburg 2000 91, [Ko04] 16);
x3 x4 x1 x2
x1 x2 x2 x3 xn−1 xn xn x1
(2) + + ··· + + > x1 + x2 · · · + xn , ([OM] St Petersburg 2000);
xn x1 xn−2 xn−1
x1 x3 x2 x4 xn−1 x1 xn x2
(3) + + ··· + + > x1 + x2 · · · + xn , ([OM] St Petersburg 2000);
x2 x3 xn x1
x2 x3 xn x1 x1 x2 xn−1 xn
(4) + + ··· + + > + + ··· + + . ([Uiuc] 2001, [Ko03] s.107).
x1 x2 xn−1 xn x2 x3 xn x1

xs+1
1 xs+1
2 xs+1
n−1 xs+1
n xs1 xs2 xsn−1 xsn
6.1.16. + + · · · + + > + + · · · + + ,
xs2 xs3 xsn xs1 xs−1
2 xs−1
3 xns−1 x1s−1
dla s > 1 oraz x1 , . . . , xn > 0. ([Cmj] 23(5)(1992) 439, [Mat] 4/1994 240).

D. Dla dowolnych liczb rzeczywistych a, b zachodzi nierówność


as
(a − b) > (a − b).
bs
Wstawiając a = xi , b = xi+1 , dla i = 1, . . . , n, otrzymujemy n nierówności. Rozpatrywaną nierówność
otrzymamy po dodaniu stronami wszystkich tych n nierówności. ⊠

x21 x22 x2 x2
6.1.17. + + · · · + n−1 + n > x1 + x2 + · · · + xn ,
x2 x3 xn x1
dla s > 1 oraz x1 , . . . , xn > 0. ([OM] Chiny 1984).

D. Jest to nierówność 6.1.16 dla s = 1. Wynika to również natychmiast z 6.1.2. Inne dowody
znajdziemy, na przykład, w [Liu1] s.84-85 (cztery różne dowody). ⊠

x31 x32 x3 x3 1
6.1.18. + + · · · + n−1 + n > ((x1 + 1)2 + . . . + (xn + 1)2 ) − n,
x2 x3 xn x1 2
dla x1 , . . . , xn > 0. ([OM] Kijów 1996).

x31 x32 x3 x3
6.1.19. + + · · · + n−1 + n > x21 + x22 + · · · + x2n ,
x2 x3 xn x1
dla x1 , . . . , xn > 0. ([Mat] 2/2005 z.1623).
76 Nierówności 6. Jednorodne nierówności wymierne

6.1.20. Dla dodatnich liczb x1 , . . . , xn , y1 , . . . , yn , zachodzi nierówność


     x y xn yn 
1 1
x1 + · · · + xn y1 + · · · + yn > (x1 + y1 ) + · · · + (xn + yn ) + ··· + .
x1 + y1 xn + yn
(M. Kuczma, [Crux] 1997 s.112 z.2113).

6.1.21. Dla dodatnich liczb x1 , . . . , xn zachodzi nierówność


!  2
n X 1 X 1 
>4+ . ([IMO] Longlist 1959-1966, [Djmp] s.37).
2 i<j xi xj x
i<j i
+ xj

oooooooooooooooooooooooooooooooooooooooooooooooooooooooooooooooooooooo
6.2 Nierówność Nesbitta i jej uogólnienia
oooooooooooooooooooooooooooooooooooooooooooooooooooooooooooooooooooooo
6.2.1 (Nesbitt 1903). Dla dodatnich liczb rzeczywistych a, b, c zachodzi nierówność

a b c 3
+ + > .
b+c c+a a+b 2
([Str1] 33, [BoW] s.81, [Siw] 16).

D. Niech x = b + c, y = c + a, z = a + b. Mamy wtedy:


   
a b c 1 x y y z z x  1 3
+ + = + + + + + − 3 > (2 + 2 + 2 − 3) = .
b+c a+c a+b 2 y x z y x z 2 2
Wykorzystaliśmy nierówność 2.1.8. ⊠
U. Dziesięć różnych dowodów tej nierówności znajdziemy w artykule Hojoo Lee [MC] 14(1)(2001)
30-36. W [LeH2] jest 13 dowodów. Dziesięć różnych dowodów znajdziemy również w [Ko04] (strony
19 − 24). ⊠

Nierówność 6.2.1 jest szczególnym przypadkiem następującej ogólniejszej nierówności.

6.2.2. Jeśli u > 0, v > 0, to


a b c 3
+ + >
ub + vc uc + va ua + vb u+v
dla dodatich liczb rzeczywistych a, b, c.
q q q p
a b c
D. ([Brad] 49, 171). Niech x1 = ub+vc , x2 = uc+va , x3 = ua+vb , y1 = a(ub + vc),
p p
y2 = b(uc + va), y3 = c(ua + vb). Wtedy:
2
(x1 y1 + x2 y2 + x3 y3 ) = (a + b + c)2 ,
a b c
x21 + x22 + x23 = ub+vc + uc+va + ua+vb,
u+v
y12 + y22 + y32 = (u + v)(ab + bc + ca) 6 3 (a + b + c)2
 
i teza wynika z nierówności Cauchy’ego: (x1 y1 + x2 y2 + x3 y3 )2 6 x21 + x22 + x23 y12 + y22 + y32 (patrz
2.6.1). ⊠
Nierówności 6. Jednorodne nierówności wymierne 77

as bs cs 3
6.2.3. + + > , dla a, b, c > 0, abc = 1, s > 1. ([AnC]).
b+c c+a a+b 2

a b c 3 3 abc
6.2.4. + + + > 2 dla a, b, c > 0. ([MM] 84(1)(2011) 69).
b + c a + c a + b 2(a + b + c)
 
a b c 1 1 1
6.2.5. + + > 4abc + + , dla a, b, c > 0.
b+c a+c a+b (a + b)3 (b + c)3 (c + a)3
a b c d
6.2.6. + + + > 2, dla a, b, c, d > 0. ([Kw] 7/1985 47, 2/1997 43).
b+c c+d d+a a+b
U. W książce [Ko04] na stronach 24 - 29 znajduje się 8 różnych dowodów tej nierówności. ⊠

a1 a2 a3 a4 a5 5
6.2.7. + + + + > , dla a1 , . . . , a5 > 0. ([Ko00]).
a2 + a3 a3 + a4 a4 + a5 a5 + a1 a1 + a2 2
U. W książce [Ko04] na stronach 29 - 34 znajduje się 5 różnych dowodów tej nierówności. ⊠
a1 a2 a3 a4 a5 a6
6.2.8. + + + + + > 3, dla a1 , . . . , a6 > 0.
a2 + a3 a3 + a4 a4 + a5 a5 + a6 a6 + a1 a1 + a2
([Ko04] 34).

a1 a2 an−1 an 1 (a1 + · · · + an )2
6.2.9. + +· · ·+ + > · 2 , dla a1 , . . . , an > 0.
a2 + a3 a3 + a4 an + a1 a1 + a2 2 a1 + · · · + a2n
([Khr3] 22).

Pewne uogólnienie nierówności Nesbitta 6.2.1 zaproponował w 1954 roku H. S. Shapiro w


[Mon] (Problem 4603, strona 571).

6.2.10 (Problem Shapiro). Rozważmy nierówność:


a1 a2 an−1 an n
+ + ··· + + > ,
a2 + a3 a3 + a4 an + a1 a1 + a2 2

dla a1 , . . . , an > 0.
(1) Z powyższych faktów wynika, że nierówność ta jest prawdziwa dla n = 3, 4, 5, 6.
(2) Jeśli jest fałszywa dla pewnego nieparzystego n, to jest również fałszywa dla n + 1.
(3) Jeśli jest fałszywa dla pewnego n, to jest również fałszywa dla n + 2.
(4) (D. Djekovicz 1961). Dla n = 8 jest prawdziwa.
(5) (P. Novosad 1967). Dla n = 10 jest prawdziwa.
(6) (V. Levin, E. Godunova 1974). Dla n = 12 jest prawdziwa.
(7) (M. Lighthill, A. Zalauf). Dla n = 14 jest fałszywa.
(8) Nierówność ta jest fałszywa dla wszystkich parzystych n > 14. Dla pozostałych liczb
parzystych n > 4 jest prawdziwa.
(9) (L. Deykin 1971). Dla n = 25 jest fałszywa.
(10) (K. Trosh 1989). Dla n = 23 jest prawdziwa.
(11) Nierówność ta jest fałszywa dla wszystkich nieparzystych n > 25. Dla pozostałych
liczb nieparzystych n > 3 jest prawdziwa.
([MiV] 132-138, [M-pf] 440-471, [Ko04]).
78 Nierówności 6. Jednorodne nierówności wymierne

a b c d e
6.2.11. 1 < + + + + < 4, dla a, b, c, d, e > 0. ([AuP] 2000).
a+b b+c c+d d+e e+a
a1 a2 an−1 an n
6.2.12. + + ··· + + > , dla a1 , . . . , an > 0.
a2 + a3 a3 + a4 an + a1 a1 + a2 4
([WaJ] 128(69), [Dlt] 10/1994, [Ko04] 56).

a1 a2 an−1 an
6.2.13. + + ··· + + > 2, dla n > 3, a1 , . . . , an > 3.
an + a2 a1 + a3 an−2 + an an−1 + a1
([TT] 1982).

a1 a2 an−1 an
6.2.14. + + ··· + + > 2, dla n > 4, a1 , . . . , an > 0.
a2 + an a3 + a1 an + an−2 a1 + an−1
([Kw] 2/1997 43).

6.2.15 (Diananda 1959-1961). Rozważmy nierówność:


a1 a2 an−1 an n
+ + ··· + + > ,
a2 + a3 + a4 a3 + a4 + a5 an + a1 + a2 a1 + a2 + a3 3

dla a1 , . . . , an > 0.
(1) Nierówność ta jest prawdziwa dla n = 4, 5, 6, 7, 8.
(2) Jeśli jest fałszywa dla pewnego n, to jest również fałszywa dla n + 3.
(3) Dla n = 15 jest fałszywa. ([Ko04]).

a1 a2 an n
6.2.16. + +· · ·+ > , dla a1 , . . . , an > 0, gdzie S = a1 +· · ·+an .
S − a1 S − a2 S − an n−1
([Str1] s.72, [Kw] 9/1973 32, [OM] Australia 1993, [Crux] 1997 s.324).
  k  
k  k 
a1 a2 an n
6.2.17. + + ··· + > ,
S − a1 S − a2 S − an (n − 1)k
dla a1 , . . . , an > 0, 0 < k 6 1, gdzie S = a1 + · · · + an . ([IMO] Longlist 1989).
S − a1 S − a2 S − an
6.2.18. + + ··· + > n(n − 1),
a1 a2 an
dla a1 , . . . , an > 0, gdzie S = a1 + · · · + an . ([OM] Australia 1993, [Crux] 1997 s.324).

⋆ P. H. Diananda, Extensions of an inequality of H. S. Shapiro, [Mon] 66(1959) 489-491.


P. H. Diananda, On a conjecture of L. J. Mordell regarding an inequality involving quadratic forms,
[Jlms] 36(1961) 185-192.
P. H. Diananda, Some cyclic and other inequalities, [Pcam] 58(1962) 425-427.
J. Górnicki, Nierówności cykliczne, [Gorn] 69-71.
L. Kurlandczyk, Problem Shapiro, [Ko04], 9-106.
Hojoo Lee, Ten different proofs of an inequality, [MC] 14(1)(2001) 30-36.
A. M. Nesbitt, Problem 15 114, Educational Times, 3(2)(1903), 37-38.
D. S. Mitrinović, O nierówności Shapiro, [Mitr], [Mit2] 299-300.
D. S. Mitrinović, J. E. Pećarić, A. M. Finc, Shapiro’s inequality, [M-pf], 440-471.
D. S. Mitrinović, P. M. Vasić, Cyclic inequalities, [MiV], 131-138.
F. H. Northover, An invalid inequality, [Mon] 63(3)(1956) 191-192.
Nierówności 6. Jednorodne nierówności wymierne 79

oooooooooooooooooooooooooooooooooooooooooooooooooooooooooooooooooooooo
6.3 Jednorodne nierówności wymierne dwóch zmiennych
oooooooooooooooooooooooooooooooooooooooooooooooooooooooooooooooooooooo
1 1 1 9
6.3.1. + 2+ 2 > , dla x, y > 0. ([OM] Rosja 1999).
(x + y)2 x y 4xy

6.3.2. Dla x, y > 0:


1 1 4
(1) + > , ([Kw] 2/1997 42);
x y x+y
1 4 9
(2) + > , ([Kw] 2/1997 42);
x y x+y
2x2 + 3y 2 2y 2 + 3x2 4
(3) + 6 , ([Math] 2006).
2x3 + 3y 3 2y 3 + 3x3 x+y

x6 y6
6.3.3. x4 + y 4 6 + , dla x, y > 0. (3.6.9).
y2 x2
oooooooooooooooooooooooooooooooooooooooooooooooooooooooooooooooooooooo
6.4 Jednorodne nierówności wymierne trzech zmiennych
oooooooooooooooooooooooooooooooooooooooooooooooooooooooooooooooooooooo

6.4.1. Jeśli a, b, c > 0, to:


a b c 1 1 1
(1) + + > 3 + 3 + 3 , (3.6.9, [Mat] 2/1994 116);
b4 c4 a4 a b c
a b c 1 1 1
(2) + + > + + , ([MaOD] 34);
bc ca ab a b c
b c a a b c
(3) + + > + + , gdy 0 < c < b < a, ([OM] Moskwa 1993/1994);
a b c b c a
a b c a+b+c
(4) + + > √ , (7.1.5, [Math] 2006);
b c a 3
abc
a b c a + kb b + kc c + ka
(5) + + > + + , dla dowolnego k > 0, (Nguyen Viet Anh, [Pkh] s.149;
b c a a + kc b + ka c + kb
a2 b2 c2 a b c
(6) 2
+ 2
+ 2
> + + , (3.6.9, [Nord] 1987, [Pa97]);
b c a b c a
a2 b2 c2 9(ab + bc + ca)
(7) + + + > 12, ([Pkh] s.193);
b 2 c2 a2 a2 + b2 + c2
ac ba cb a b c
(8) 2
+ 2 + 2 > + + , (7.1.5);
b c a b c a
a2 + b2 b2 + c2 c2 + a2 a3 b3 c3
(9) a+b+c6 + + 6 + + , ([Ko00]);
2c 2a 2b bc ca ab
a2 b2 c2 (a − b)2
(10) + + >a+b+c+4 , ([Balk] 2005);
b c a a+b+c
80 Nierówności 6. Jednorodne nierówności wymierne

a2 b3 c4
(11) + 2 + 3 > −a + 2b + 3c, ([OM] Ukraina 2005);
b c a
a 3 b 3 c3 a2 b2 c2
(12) + + > + + , (3.6.9, [OM] Kanada 2002, [Ko03] 49);
bc ca ab b c a
a 3 b 3 c3
(13) + 2 + 2 > a + b + c, (3.6.9, 6.1.16, [Zw] 2003);
b2 c a
a3 b3 c3 a2 b2 c2
(14) + + > + + , (3.6.9, 6.1.16, [Mat] 4/1994 239);
b2 c2 a2 b c a
2 2
a b b c c a 2
(15) + + > a2 + b2 + c2 , gdy a > b > c > 0, ([OM] Wietnam 1991);
c a b
a10 b10 c10
(16) + + > a8 b + b8 c + c8 a, (3.6.9, [Ko03] 50).
c a b
xy yz zx
6.4.2. + + > x + y + z, dla x, y, z > 0. ([MOc] z.556).
z x y
U. Analogiczna nierówność dla czterech liczb x, y, z, t nie zachodzi. Przykład: x = 1, y = 216,
z = 6, t = 18. ([Ko04] 14). ⊠

x3 a3 (x + a)3
6.4.3. + > , dla x, y, z, a, b, c > 0, to ([Ko03] 33).
yz bc (y + b)(z + c)

a3 b3 c3 (a + b + c)3
6.4.4. + + > , dla a, b, c, x, y, z, > 0. ([OM] Białoruś 2000).
x y z 3(x + y + z)

D. ([AF00] 10). Korzystamy z nierówności Höldera:


     3
x31 + x32 + x33 y13 + y23 + y33 z13 + z23 + z33 > x1 y1 z1 + x2 y2 z+ x3 y3 z3 ,
√ √
zachodzącej
√ dodatnich liczb rzeczywistych
√ (patrz 2.7.2).
√ Podstawiamy: x1 = a/ 3 x, x2 = b/ 3 y,

x3 = c/ 3 z, y1 = y2 = y3 = 1, z1 = 3 x, z2 = 3 y, z3 = 3 z i mamy:
 3 
a b3 c3
+ + (1 + 1 + 1)(x + y + z) > (a + b + c)3 .
x y z

Dzielimy przez 3(x + y + z) i otrzymujemy tezę. ⊠

a3 b3 c3
6.4.5. + + > a + b + c, dla a, b, c > 0. ([Rias]).
bc ca ab
D. ([Rias]).
r r r
a3 3 a
3 b3 3 b
3 c3 3 c
3
+b+c>3 bc = 3a, +c+a>3 ca = 3b, +a+b>3 ab = 3c.
bc bc ca ca ab ab
Po dodaniu do siebie tych trzech nierówności otrzymujemy tezę. ⊠

1 1 1 1
6.4.6. + + 6 dla x, y, z > 0.
a3 + b3 + abc b3 + c3 + abc c3 + a3 + abc abc
([OM] USA 1997, [RiM] July 2001).
Nierówności 6. Jednorodne nierówności wymierne 81

6.4.7. Jeśli x, y, z > 0, to:


1 1 1 10
(1) + 2 + 2 > , (V.Cirtoaje, Nguyen Viet Anh, [Pkh] s.154);
x +y
2 2 y +z 2 z +x 2 (x + y + z)2
1 1 1 1 6
(2) + 2 + 2 + 2 > , ([Pkh] s.167);
x2 +y 2 y +z 2 z +x 2 x +y +z2 2 xy + yz + zx
1 1 1 9
(3) + + > , ([RiM] July 2001);
(x + y)2 (y + z)2 (z + x)2 4(xy + yz + zx)
1 1 1 1
(4) + + > , ([Pkh] s.249);
(2x + y)2 (2y + z)2 (2z + x)2 xy + yz + zx
1 1 1 27
(5) + + > , ([MOc] 2003 z.236);
y(x + y) z(y + z) x(z + x) 2(x + y + z)2
1 1 1 9
(6) + 2 + 2 > ,
4x2 − xy + 4y 2 4y − yz + 4z 2 4z − zx + 4x2 7(x + y 2 + z 2 )
2
(Vasile Cirtoaje, [Mild]).

 
1 1 1 1 1 1 1
6.4.8. + + 6 + + , dla a, b, c > 0. ([OM] Irlandia 1998, [Khr2], [Khr1]).
a+b b+c c+a 2 a b c

D. Wynika to z faktu 1.5.7 zastosowanego do wypukłej funkcji f : (0, ∞) → R, f (x) = x1 . ⊠

6.4.9. Jeśli a, b, c > 0, to:


1 1 1 3
(1) + + > , ([Siw] 76);
a+b b+c c+a a+b+c
1 1 1 3
(2) + + > , ([Kw] 2/1997 42);
a+b b+c c+a 2(a + b + c)
1 1 1 9
(3) + + > , ([OM] Irlandia 1998);
a+b b+c c+a 2(a + b + c)
1 1 1 a8 + b8 + c8
(4) + + 6 , ([IMO] Longlist 1967, [Djmp] s.47(355), [Crux] z.413, [Siw] 18);
a b c a3 b3 c3
1 1 1 1 √
(5) + + 6 , gdzie x = 3 abc, ([OM] Mołdawia 1998);
a+b+x b+c+x c+a+x x
 
a b c 4 1 1 1
(6) + + > + + ,
b2 + c2 c2 + a2 b2 + a2 5 b+c c+a a+b
(Pham Kim Hung [Pkh] s.45);

a b c 9
(7) + + > , ([AnC]);
(b + c)2 (c + a)2 (b + a)2 4(a + b + c)
2a 2b 2c a b c
(8) + 2 + 2 6 + + , ([Balt] 2004);
a2 + bc b + ca c + ab bc ca ab
a+b b+c c+a 1 1 1
(9) + + 6 + + , (Hojoo Lee, [Crux] z.2580);
ab + c2 bc + a2 ca + b2 a b c
82 Nierówności 6. Jednorodne nierówności wymierne

a b c 9
(10) + + 6 ,
(a + b)(a + c) (b + c)(b + a) (c + a)(c + b) 4(a + b + c)
([RiM] July 2001);
!   
a2 b2 c2 1 1 1 1 1 1
(11) + + + 2+ 2 >3 + + .
b+c c+a a+b a2 b c a+b b+c c+a

6.4.10. Dla dodatnich liczb a, b, c zachodzą następujące nierówności.


a+b b+c c+a a b c 9
(1) + + − − − > , ([Ko04] 17);
c a b b+c a+c a+b 2
 
a+b b+c c+a a b c
(2) + + >4 + + , ([AnC], [MaOD] 32);
c a b b+c c+a a+b
a+b b+c c+a a b c
(3) + + 6 + + , ([OM] Indie 2002, [Pkh] s.93);
a+c b+a c+b b c a
a+b b+c c+a (a + b + c)2
(4) + + 6 , ([Pkh] s.93);
b+c c+a a+b ab + bc + ca
a + b − 2c b + c − 2a c + a − 2b
(5) + + > 0, ([MaS] 4/1993 z.3797 [Ko00]);
b+c c+a a+b
a b c
(6) + + 6 1, ([OM] Mołdawia 2002);
b + 2a c + 2b a + 2c
a b c
(7) + + > 1, (6.2.2, [OM] Czechy-Słowacja 1999);
b + 2c c + 2a a + 2b
a b c
(8) + + > 1, (6.2.2);
2b + c 2c + a 2a + b
a b c 3
(9) + + > , (6.2.2);
b + 3c c + 3a a + 3b 4
a + 2b b + 2c c + 2a
(10) + + > 3, (Pham Kim Hung, [Pkh] s.163);
c + 2b c + 2a c + 2b
 
1 1 1
(11) (a + b + c) + + > 9, (6.1.9, [BoW] 7 s.80).
a b c

a−b b−c c−a


6.4.11. + + > 0, dla a > b > c > 0. ([Pie2]).
a+b b+c c+a

6.4.12. Dla dodatnich liczb a, b, c zachodzą następujące nierówności.


a2 b2 c2 2abc
(1) + + + > 1, ([Pkh] s.242);
(a + b)2 (b + c)2 (c + a)2 (a + b)(b + c)(c + a)
a2 b2 c2
(2) + + > 1, ([OM] Mołdawia 1999);
a2 + 2bc b2 + 2ca c2 + 2ab
a2 b2 c2 ab bc ca
(3) + + >1> + + ,
a2 + 2bc b2 + 2ca c2 + 2ab ab + c2 bc + a2 ca + b2
([OM] Bośnia Hercegowina 2000);
Nierówności 6. Jednorodne nierówności wymierne 83

a2 b2 c2 1
(4) + + > ,
a + 2(a + b)
2 2 b + 2(b + c)
2 2 c + 2(c + a)
2 2 3
(Pham Kim Hung [Pkh] s.45);

a2 + bc b2 + ca c2 + ab
(5) + + > 3, ([OM] Rosja 1998);
a2 + ab b2 + bc c2 + ca
a2 − bc b2 − ca c2 − ab
(6) + + > 0,
2a2 + b2 + c2 a2 + 2b2 + c2 a2 + b2 + 2c2
(Pham Kim Hung [Pkh] s.35);

a2 − bc b2 − ca c2 − ab
(7) + + > 0, dla 0 6 λ 6 2,
λa2 + b2 + c2 a2 + λb2 + c2 a2 + b2 + λc2
(Pham Kim Hung [Pkh] s.59);

ab bc ca a b c
(8) + + > + + , ([OM] Mołdawia 1999);
c(c + a) a(a + b) b(b + c) c+a a+b b+c
a2 b2 c2 3
(9) + + > , ([OM] Moskwa 1999/2000);
(a + b)(a + c) (b + c)(b + a) (c + a)(c + b) 4
(a + b)2 (b + c)2 ((c + a)2
(10) + 2 + 2 > 6, (P.Scholze, D.Grinberg, [Pkh] s.207);
c2 + ab a + bc b + ac
(b + c − a)2 (c + a − b)2 (a + b − c)2 3
(11) + + > ,
(b + c)2 + a2 (c + a)2 + b2 (a + b)2 + c2 5
([OM] Japonia 1997, [RiM] July 2001, [Crux] 2003 s.225);

(2a + b + c)2 (2b + c + a)2 (2c + a + b)2


(12) + + 6 8, ([OM] USA 2003);
2a2 + (b + c)2 2b2 + (c + a)2 2c2 + (a + b)2
 2  2  2
a + 2b b + 2c c + 2a
(13) + + > 27, ([OM] St Petersburg 1995);
c a b
a(b + c) b(c + a) c(a + b)
(14) + 2 + 2 > 2, ((Pham Kim Hung, [Pkh] s.201);
a2 + bc b + ca c + ab
a(3a − b) b(3b − c) c(3c − a) a3 + b3 + c3
(15) + + 6 , (G. Perz, [Crux] z.1976);
c(a + b) a(b + c) b(c + a) abc
a(b + c − a) b(c + a − b) c(a + b − c)
(16) + + > 0, ([Pkh] s.201);
a2 + 2bc b2 + 2ca c2 + 2ab
 2  2  2
a b c 9(a + b + c)2
(17) 2+ + 2+ + 2+ > , ([Pkh] s.180);
b c a ab + bc + ca
 2    
2a 2b 2 2c 2 9(a + b + c)2
(18) 1+ + 1+ + 1+ > , ([Pkh] s.181);
b c a ab + bc + ca
 
1 1 1 9
(19) (ab + bc + ca) + + > ,
(a + b)2 (b + c)2 (c + a)2 4
([OM] Iran 1996, [Crux] 1997 s.367).

6.4.13. Dla parami różnych liczb dodatnich a, b, c zachodzą nierówności:


a2 b2 c2
(1) + + > 1,
(a − b)2 (b − c)2 (c − a)2
84 Nierówności 6. Jednorodne nierówności wymierne

(Le Huu Dien Khue, [Pkh] s.151);


 
1 1 1
(2) (ab + bc + ca) + + > 4,
(a − b) 2 (b − c)2 (c − a)2
([OM] Wietnam 2008, [ChKh] 46, 131-132).

6.4.14. Dla dodatnich liczb a, b, c zachodzą następujące nierówności.


a3 b3 c3 a2 b2 c2
(1) + + > + + , ([Pkh] s.108);
b3 + c3 c3 + a3 a3 + b3 b2 + c2 c2 + a2 a2 + b2
a3 b3 c3 3
(2) + + > , ([OM] Wietnam 2005);
(a + b)3 (b + c)3 (c + a)3 8
a3 b3 c3 5abc
(3) + + + > 1,
(a + b)3 (b + c)3 (c + a)3 (a + b)(b + c)(c + a)
(Pham Kim Hung, [Pkh] s.241);

a3 b3 c3
(4) + + > 1,
a3 + b3 + abc b3 + c3 + abc c3 + a3 + abc
(Nguyen Van Thach, [Pkh] s.36);

a3 b3 c3
(5) + + > 1, ([MaOD] 42);
a3 + 2b3 b3 + 2c3 c3 + 2a3
2(a3 + b3 + c3 ) 9(a + b + c)2
(6) + 2 > 33, (Hojoo Lee, [Crux] z.2645).
abc a + b2 + c2

 n  n  n
a+b b+c c+a
6.4.15. + + > 3 · 2n , dla a, b, c > 0, n ∈ N. ([MaS] 1/1998 z.4296).
c a b

am bm cm an bn cn
6.4.16. + + > + + ,
bm + cm cm + am am + bm bn + cn cn + an an + bn
dla m > n oraz a, b, c > 0. (T. Zvonaru, [Crux] z.2970).

a2 b2 c2 a+b+c
6.4.17. + + > , dla a, b, c > 0. ([Nord] 2005).
b+c c+a a+b 2

D. ([Stee] 13, 227).


 2
2 a √ b √ c √
(a + b + c) = √ b+c+ √ c+a+ √ a+b
b+c c+a a+b
 2  
a b2 c2
6 + + (b + c) + (c + a) + (a + b)
b+c c+a a+b
 2 
a b2 c2
= 2 + + (a + b + c).
b+c c+a a+b

Wykorzystaliśmy nierówność Cauchy’ego 2.6.1. ⊠


Nierówności 6. Jednorodne nierówności wymierne 85

6.4.18. Dla dodatnich liczb a, b, c zachodzą następujące nierówności:


a2 b2 c2 3 a3 + b3 + c3
(1) + + > · 2 , (Michael Rozenberg, [Mild]);
b+c c+a a+b 2 a + b2 + c2
a2 b2 c2 3(ab + bc + ca)
(2) + + > , ([OM] IMSA Intramural 2000);
b + 3c c + 3a a + 3b 4(a + b + c)
bc ca ab a+b+c
(3) + + 6 , ([IMO] Longlist 1970, [Djmp] s.65);
b+c c+a a+b 2
a2 + 3ab b2 + 3bc c2 + 3ca
(4) + + 6 2(a + b + c), ([Kw] 6/2008 14);
a+b b+c c+a
ab bc ca 1
(5) + + 6 (a + b + c), ([AnC]);
a + b + 2c b + c + 2a c + a + 2b 4
ab bc ca 1
(6) + + 6 (a + b + c), ([Pkh] s.197);
a + 4b + 4c b + 4c + 4a c + 4a + 4b 9
ab + c2 bc + a2 ca + b2
(7) + + > a + b + c, (Hojoo Lee, [Crux] z.2581);
a+b b+c c+a
a2 (b + c) b2 (c + a) c2 (a + b)
(8) + + > 2(a + b + c); ([Ko03] 55),
bc ca ab
a2 − b2 c2 − b2 a2 − c2
(9) + + > 3a − 4b + c, dla a > b > c > 0,
c a b
([OM] Ukraina 1997, [Crux] 1997 s.76);

a3 + b3 + c3 a+b+c
(10) > , dla a, b, c > 0, ([Ko00]);
a +b +c
2 2 2 3
a3 b3 c3 3(ab + bc + ca)
(11) + + > ,
b − bc + c
2 2 c − ca + a
2 2 a − ab + b
2 2 a+b+c
([AnC], [Crux] 2005 s.179);

a3 b3 c3
(12) + + > a + b + c, ([Pkh] s.121);
b2 − bc + c2 c2 − ca + a2 a2 − ab + b2
a3 b3 c3 1
(13) + + > (a + b + c),
2a − ab + 2b
2 2 2b − bc + 2c
2 2 2c − ca + 2a
2 2 3
(Nguyen Viet Anh, [Pkh] s.203);

2a2 (b + c) 2b2 (c + a) 2c2 (a + b)


(14) + + 6 a + b + c.
(a + b)(a + c) (b + c)(b + a) (c + a)(c + b)
(V. N. Murty, [Crux] z.570).

 
a b c xy + yz + zx
6.4.19. (y + z) + (z + x) + (x + y) > 3 ,
b+c c+a a+b x+y+z
dla x, y, z, a, b, c > 0. (W. Janous, [Crux] z.1672).

a3 b3 c3 a+b+c
6.4.20. + + > ,
a2 + ab + b2 b2 + bc + c2 c2 + ca + a2 3
dla a, b, c > 0. ([Kw] 6/1986 36, 2/1997 43, [Dlt] 9/1989 M192, [KoM] B3384 2000).
86 Nierówności 6. Jednorodne nierówności wymierne

D. Wynika to z nierówności
a3 2 1
> a − b,
+ ab + b2
a2 3 3
która jest konsekwencją nierówności (a + b)(a − b)2 > 0. ⊠
 
1 1 1
6.4.21. (a3 + b3 + c3 ) + + > (a + b + c)2 , a, b, c > 0. ([Mit2] s.108).
a b c
a3 b3 c3 a2 + b2 + c2
6.4.22. + + > , dla a, b, c > 0. ([OM] Ukraina 1996).
b + 2c c + 2a a + 2b 3
a3 − b3 b3 − c3 c3 − a3 (a − b)2 + (b − c)2 + (c − a)2
6.4.23. + + 6 , dla a, b, c > 0.
a+b b+c c+a 4
([OM] Mołdawia 2004).

 n−2  
an bn cn 2 a + b + c n−1
6.4.24. + + > , dla a, b, c > 0.
b+c c+a a+b 3 2
([OM] Grecja 1987, [Pa97]).

an bn cn an−1 + bn−1 + cn−1


6.4.25. + + > , dla a, b, c > 0. ([MaS] 4/1993, [Ko04] 17).
b+c c+a a+b 2
ar br cr 1  r−1 
6.4.26. + + > a + br−1 + cr−1 dla r > 23 .
a+b b+c c+a 2
(V.Cirtoaje, Pham Kim Hung [Pkh] s.168).

ak+n bk+n ck+n


6.4.27. + k + k > an +bn +cn , dla a, b, c > 0, n, k ∈ N. ([OM] Serbia-Czarnogóra 2002).
bk c a
oooooooooooooooooooooooooooooooooooooooooooooooooooooooooooooooooooooo
6.5 Jednorodne nierówności wymierne czterech zmiennych
oooooooooooooooooooooooooooooooooooooooooooooooooooooooooooooooooooooo
6.5.1. Jeśli a, b, c, d > 0, to:
1 1 1 1 a+b+c+d
(1) + + + > , ([OM] Austria 2005);
a3 b3 c3 d3 abcd
1 1 1 1 1 1 243
(2) + 3 + 3 + 3 + 3 + 3 > ,
a +b
3 3 a +c 3 a +d 3 b +c 3 b +d 3 c +d 3 (a + b + c + d)3
([Pkh] s.156).

6.5.2. Jeśli a, b, c, d > 0, to:


1 1 1 1 4
(1) + 2 + 2 + 2 > , ([Pkh] s.23);
a + ab b + bc c + cd d + da
2 ac + bd
 
1 1 1 1 2 1 4 9 16
(2) + + + > 2+ 2 + + ,
a b c d a a + b2 a2 + b2 + c2 a2 + b2 + c2 + d2
(Pham Kim Hung [Pkh] s.24);
 2
1 1 1 1 1 4 12 18
(3) + + + > + 2 + 2 + 2 ,
a b c d a2 a +b 2 a +b +c
2 2 a + b + c2 + d2
2

(Pham Kim Hung [Pkh] s.25).


Nierówności 6. Jednorodne nierówności wymierne 87

6.5.3. Jeśli a, b, c, d > 0, to:


 
12 1 1 1 1 1 1 3 1 1 1 1
(1) 6 + + + + + 6 · + + + ,
a+b+c+d a+b a+c a+d b+c b+d c+d 4 a b c d
([Kw] 2/1997 42);

1 1 1 1 16
(2) + + + > , ([Str1] s.73);
a b c d a+b+c+d
1 1 1 1 16
(3) + + + > , ([Str1] 34);
a+b+c a+b+d a+c+d b+c+d a+b+c+d
a b c d 4
(4) + 2 + 2 + 2 > ,
b +c +d
2 2 2 a +c +d2 2 a +b +d 2 2 a +b +c
2 2 a+b+c+d
(Pham Kim Hung [Pkh] s.37).

6.5.4. Jeśli a, b, c, d > 0, to:


a b c d a+b+c+d
(1) + + + > √ , ([Math] 2006);
b c d a 4
abcd
a2 b b2 c c2 d d2 a
(2) + + + > ab + bc + cd + da, (3.6.9, [Ko03]);
d a b c
a3 b2 b3 c2 c3 d2 d3 a2
(3) + + + > ab2 c + bc2 d + cd2 a + da2 b, (3.6.9, [Ko03] 50).
c d a b
6.5.5. Dla dodatnich liczb a, b, c, d zachodzą następujące nierówności.
a b c d 4
(1) + + + > ;
2b + c 2c + d 2d + a 2a + b 3
a b c d
(2) + + + > 2, ([Ko04] 17);
a + 2b + c b + 2c + d c + 2d + a d + 2a + b
a b c d 2
(3) + + + > ,
b + 2c + 3d c + 2d + 3a d + 2a + 3b d + 2b + 3c 3
([IMO] Shortlist 1993, [OM] USA 1993, [Pa97]);

a b c d
(4) 1< + + + < 2, ([IMO] 1974, [Br80] 95);
a+b+d a+b+c b+c+d a+c+d
a+c b+d c+a d+b
(5) + + + > 4, ([IMO] Shortlist 1971, [Balt] 1996);
a+b b+c c+d d+a
a−b b−c c−d d−a
(6) + + + > 0, ([AnC], [OM] Chorwacja 2009);
b+c c+d d+a a+b
b−a c−b d−c a−d
(7) + + + > 0. ([Dlt] 2/2003 z.448);
d+a a+b b+c c+d
    
2a 2b 2c 2d
(8) 2+ 2+ 2+ 2+ > 9,
b+c c+d d+a a+b
(V.Cirtoaje, [Pkh] s.166);
    
ka kb kc kd
(9) 2+ 2+ 2+ 2+ > (k + 1)2 ,
b+c c+d d+a a+b
dla dowolnego k > 0, ([Pkh] s.166).
88 Nierówności 6. Jednorodne nierówności wymierne

1 1 1
6.5.6. 1 1 + 1 1 6 1 1 , dla a, b, c, d > 0. ([OM] Wietnam 1962, [ChKh] 33, 85).
a + b c + d a+c + b+d
D. Standardowe przekształcenia sprowadzją tę nierówność do równoważnej oczywistej nierówności

(ad − bc)2
> 0. ⊠
(a + b + c + d)(a + b)(c + d)

a2 b2 c2 d2 a+b+c+d
6.5.7. + + + > , dla a, b, c, d > 0.
b+c+d c+d+a d+a+b a+b+c 3

D. (a + b + c + d)2
 √ √ √ √ 2
= √ a b+c+d+ √ b c+d+a+ √ c d+a+b+ √ d a+b+c
b+c+d c+d+a d+a+b a+b+c

  
a2 b2 c2 d2
6 b+c+d + c+d+a + d+a+b + a+b+c (b + c + d) + · · · + (a + b + c)
 
a2 b2 c2 d2
=3 b+c+d + c+d+a + d+a+b + a+b+c (a + b + c + d).

Wykorzystaliśmy nierówność Cauchy’ego 2.6.1. ⊠

6.5.8. Dla dodatnich liczb a, b, c, d zachodzą następujące nierówności.


a2 b2 c2 d2
(1) + + + > 2, ([Ko04] 17);
b(a + c) c(b + d) d(c + a) a(d + b)
 2  2  2  2
a b c d 4
(2) + + ++ > , ([Pkh] s.94);
a+b+c b+c+d c+d+a d+a+b 9
a2 + b2 + c2 a2 + b2 + d2 a2 + c2 + d2 b2 + c2 + d2
(3) + + + > a + b + c + d,
a+b+c a+b+d a+c+d b+c+d
([Cmj] 1978 s.238, [OMm] 1997/1998);

(a − b)(a − c) (b − c)(b − d) (c − d)(c − a) (d − a)(d − b)


(4) + + + > 0,
a+b+c b+c+d c+d+a d+a+b
([IMO] Shortlist 2008);

a3 b3 c3 d3 a+b+c+d
(5) + + + > , ([Pkh] s.28).
a +b
2 2 b +c
2 2 c +d
2 2 d +a
2 2 2
⊡⊡⊡⊡⊡⊡⊡⊡⊡⊡⊡⊡⊡⊡⊡⊡⊡⊡⊡⊡⊡⊡⊡⊡⊡⊡⊡⊡⊡⊡⊡⊡⊡⊡⊡⊡⊡⊡⊡⊡⊡⊡⊡⊡⊡⊡⊡⊡⊡⊡⊡⊡
7 Różne nierówności wymierne
⊡⊡⊡⊡⊡⊡⊡⊡⊡⊡⊡⊡⊡⊡⊡⊡⊡⊡⊡⊡⊡⊡⊡⊡⊡⊡⊡⊡⊡⊡⊡⊡⊡⊡⊡⊡⊡⊡⊡⊡⊡⊡⊡⊡⊡⊡⊡⊡⊡⊡⊡⊡

oooooooooooooooooooooooooooooooooooooooooooooooooooooooooooooooooooooo
7.1 Nierówności wymierne ze stałym iloczynem
oooooooooooooooooooooooooooooooooooooooooooooooooooooooooooooooooooooo
W tym podrozdziale często zakładać będziemy, że iloczyn rozważanych liczb rzeczywistych
jest równy 1. To założenie, że iloczyn danych liczb jest równy 1, można wysłowić przy pomocy
innych równoważnych założeń. Zanotujmy kilka tego typu równoważności.

7.1.1. Jeśli x, y są liczbami rzeczywistymi różnymi od −1, to


1 1
xy = 1 ⇐⇒ + = 1.
1+x 1+y
1 1
D. + = 1 ⇐⇒ (1 + y) + (1 + x) = (1 + x)(1 + y) = 1 + x + y + xy ⇐⇒ xy = 1. ⊠
1+x 1+y

7.1.2. Jeśli x, y, z są dodatnimi liczbami rzeczywistymi, to


1 1 1
xyz = 1 ⇐⇒ + + = 1. ([MaS] 4/1993, [Mild]).
1 + x + xy 1 + y + yz 1 + z + zx
D. Załóżmy, że xyz = 1. Mamy wtedy:
1 1 1 1 x xy
+ + = + +
1 + x + xy 1 + y + yz 1 + z + zx 1 + x + xy x + xy + xyz xy + xyz + xyzx
1 x xy
= + +
1 + x + xy 1 + x + xy 1 + x + xy
1 + x + xy
= = 1.
1 + x + xy
1 1 1
Niech A = + + = 1 oraz s = xyz. Załȯżmy, że A = 1. Pokażemy, że
1 + x + xy 1 + y + yz 1 + z + zx
s = 1.
Przypuśćmy, że s > 1. Wtedy mamy sprzeczność:
1 x xy
1=A = + +
1 + x + xy x + xy + xyz xy + xyz + xyzx
1 x xy
= + +
1 + x + xy s + x + xy s + xs + xy
1 x xy 1 + x + xy
> + + = = 1.
1 + x + xy 1 + x + xy 1 + x + xy 1 + x + xy
Jeśli s < 1, to w ten sam sposób otrzymujemy sprzeczność 1 = A > 1. Zatem s = 1. ⊠
Założenie o dodatniości liczb x, y, z jest tutaj istotne. Liczby x = −2, y = − 13 , z = 1
2
spełniają równość
1 1 1
+ + =1
1 + x + xy 1 + y + yz 1 + z + zx

89
90 Nierówności 7. Różne nierówności wymierne

i ich iloczyn jest różny od 1.


Dokładnie tak samo wykazujemy podobną równoważność dla czterech i więcej dodatnich
liczb rzeczywistych. Zanotujmy to dla czterech liczb.

7.1.3. Jeśli x, y, z, t są dodatnimi liczbami rzeczywistymi, to xyzt = 1 wtedy i tylko wtedy,


gdy
1 1 1 1
+ + + = 1.
1 + x + xy + xyz 1 + y + yz + yzt 1 + z + zt + ztx 1 + t + tx + txy

Zastępując w powyższych równoważnościach dodatnie liczby x, y, z . . . odpowiednio ich


odwrotnościami x−1 , y −1 , z −1 , . . . , otrzymujemy:

7.1.4. Dla dodatnich liczb rzeczywistych x, y, z, t zachodzą następujące równoważności.


x y
xy = 1 ⇐⇒ + = 1,
x+1 y+1
xy yz zx
xyz = 1 ⇐⇒ + + = 1,
xy + y + 1 yz + z + 1 zx + x + 1
xyz yzt ztx txy
xyzt = 1 ⇐⇒ xyz+yz+z+1 + yzt+zt+t+1 + ztx+tx+x+1 + txy+xy+y+1 = 1.

a
7.1.5. b + cb + c
a > a + b + c, dla a, b, c > 0, abc = 1. ([OM] Czechy-Słowacja 2003).

D. ([Rias]). Z nierówności pomiędzy średnią arytmetyczną i średnią geometryczną mamy:


r r
a a b 2 a3
3 a 3
+ + >3 =3 = 3a.
b b c bc abc
b b c c c a
Podobnie: + + > 3b oraz + + > 3c. Dodajemy te trzy nierówności i otrzymujemy tezę. ⊠
c c a a a b

7.1.6. Jeśli a, b, c > 0, abc = 1, to:


a b c 1 1 1
(1) + + + 3 > a + b + c + + + , ([ME] 12(4)(2007));
b c a a b c
a b c
(2) + + + 5 > (1 + a)(1 + b)(1 + c), (Aaron Pixton, [Mild]).
b c a

7.1.7. Jeśli a, b, c > 0, abc = 1, to:


1 1 1 3
(1) + + > , ([IMO] Shortlist 1995, [OMm] 1995, [ME] 11(1)(2006));
a3 (b + c) b3 (c + a) c3 (a + b) 2
ab bc ca
(2) + + 6 1, ([IMO] Shortlist 1997, [Crux] 1998 s.460);
a5 + b5 + ab b5 + c5 + bc c5 + a5 + ca
1 1 1
(3) + 3 + 3 6 1, ([OM] Rosja 1998);
a + b + 1 b + c + 1 c + a3 + 1
3 3 3

a b c
(4) + + > 1, ([Pkh] s.245);
b4 + 2 c4 + 2 a4 + 2
Nierówności 7. Różne nierówności wymierne 91

   
1 1 1
(5) a−1+ b−1+ c−1+ 6 1, ([IMO] 2000, [LeH2]);
b c a
1 1 1 3
(6) + + > , ([Ko00], [OM] Kazachstan 2008, [MaOD] 42);
a(1 + c) b(1 + a) c(1 + b) 2
1 1 1
(7) + 2 + 2 > 1, ([Pkh] s.40);
a2 +a+1 b +b+1 c +c+1
1 1 1 1
(8) + + > , ([Pkh] s.150);
(a + 1)(a + 2) (b + 1)(a + 2) (c + 1)(c + 2) 2
1 1 1
(9) + 2 + 2 > 1, ([Pkh] s.40);
a2 +a+1 b +b+1 c +c+1
6 3
(10) 61+ , ([OM] Rumunia 2003);
ab + bc + ca a+b+c
1 1 1 1
(11) + + 6 , ([Mild]);
(a + 1) + b + 1 (b + 1) + b + 1 (c + 1) + a + 1
2 2 2 2 2 2 2
1 1 1
(12) + + > 1, ([Kw] 6/1997 M1597a);
2a + 1 2b + 1 2c + 1
1 1 1
(13) + + 6 1, ([Kw] 6/1997 M1597a);
a+b+1 b+c+1 c+a+1
1 1 1 1 1 1
(14) + + 6 + + , ([OM] Bułgaria 1997);
a+b+1 b+c+1 c+a+1 2+a 2+b 2+c
a b c 3
(15) + + > , ([ME] 12(4)(2007));
ab + 1 bc + 1 ca + 1 2
a b c 3
(16) + + > ,
(a + 1)(b + 1) (b + 1)(c + 1) (c + 1)(a + 1) 4
([OM] Czechy-Słowacja 2005, [MaOD] 12);

a b c
(17) + 2 + 2 6 1, ([OM] Serbia i Czarnogóra 2005);
a2 +2 b +2 c +2
a2 b2 c2
(18) + + 6 1, ([Math] 2006);
b3 c + a + b c3 a + b + c a3 b + c + a
ab bc ca 3
(19) + + > , ([OMm] 1996);
a+1 b+1 c+1 2
ab + 1 bc + 1 ca + 1
(20) + + > 3,
a+1 b+1 c+1
([OM] Ukraina 1998);

a3 b3 c3 3
(21) + + > ,
(1 + b)(1 + c) (1 + a)(1 + c) (1 + a)(1 + b) 4
([IMO] Shortlist 1998, [Djmp] 299(636)).

1 1 1 3
7.1.8. + + > √ √ > 1, dla a, b, c > 0 i abc = 2.
a(1 + b) b(1 + c) c(1 + a) 3
2(1 + 3 2)
([Ssm] 103(5)(2003) z.4748).
92 Nierówności 7. Różne nierówności wymierne

a2 b2 c2
7.1.9. + + > 1, dla a, b, c ∈ R r {1}, abc = 1.
(a − 1)2 (b − 1)2 (c − 1)2
([IMO] 2008).

a−2 b−2 c−2


7.1.10. + + 6 0, dla a, b, c > 0, abc = 8.
a+1 b+1 c+1
([OM] Rumunia 2008, [MaOD] 43).

a5 − a2 b5 − b2 c5 − c2
7.1.11. + + > 0, dla a, b, c > 0, abc > 1.
a5 + b2 + c2 b5 + c2 + a2 c5 + a2 + b2
([IMO] 2005, [ME] 11(1)(2006)).

a2 a2 b2 b2 c2 c2
7.1.12. a4 + b4 + c4 + 3(a + b + c) > + + + + + ,
b c c a a b
dla a, b, c > 0 i abc = −1. ([OM] Iran 2003/2004, [Mild]).

1 1 1 1 1 1
7.1.13. Niech x, y, z > 0, xyz = 1. Jeśli + + > x+y+z, to n + n + n > xn +y n +z n ,
x y z x y z
dla n ∈ N. ([OM] Rosja 1999).

7.1.14. Niech a, b, c, d > 0, abcd = 1. Wtedy:


1 1 1 1
(1) + + + > 2, ([ME] 12(4)(2007));
a(1 + b) b(1 + c) c(1 + d) d(1 + a)
1 + ab 1 + bc 1 + cd 1 + da
(2) + + + > 4, ([IMO] 2002);
a+b b+c c+d d+a
1 + ab 1 + bc 1 + cd 1 + da
(3) + + + > 4, ([OM] Rosja 1998);
1+b 1+c 1+d 1+a
1 1 1 1
(4) + + + > 1, ([OM] Chiny 2004, [Pkh] s.161);
(1 + a)2 (1 + b)2 (1 + c)2 (1 + d)2
1 1 1 1
(5) + + + > 22−k , dla k > 2, (Mathlinks Lore, [Mild]).
(1 + a)k (1 + b)k (1 + c)k (1 + d)k

7.1.15. Niech x1 , . . . , xn > 0, x1 x2 · · · xn = 1. Wtedy:


2 2 2
(1) x1 + · · · + xn > + + ··· + , ([Pkh] s.146);
1 + x1 1 + x2 1 + xn
1 1 1
(2) + + ··· + 6 1, ([OM] Rumunia 1999, [Mild]);
n − 1 + x1 n − 1 + x2 n − 1 + xn
1 1 1
(3) + + ··· + 6 n − 1, dla n > 2, ([OM] Rosja 1995);
1 + x1 1 + x2 1 + xn
1 1 1 x1 + · · · + xn + n
(4) + + ··· + 6 , dla n > 2, ([Mild]);
1 + x1 1 + x2 1 + xn 4
1 1 1
(5) + + ··· + > 1, dla n > 2, ([Zw] 2006);
1 + x1 1 + x2 1 + xn
Nierówności 7. Różne nierówności wymierne 93

1 1 1
(6) + + ··· + > 1, dla n > 4,
1 + x1 + x1 x2 1 + x2 + x2 x3 1 + xn + xn x1
([OM] Rosja 2004, [ME] 12(4)(2007));

x1 + 3 x2 + 3 xn + 3
(7) + + ··· + > 3, ([OM] W.Brytania 2005, [Pkh] s.198).
(x1 + 1)2 (x2 + 1)2 (xn + 1)2

⋆ S. Malikic, Inequalities with product condition, [ME] 12(4)(2007).


oooooooooooooooooooooooooooooooooooooooooooooooooooooooooooooooooooooo
7.2 Nierówności wymierne n zmiennych
oooooooooooooooooooooooooooooooooooooooooooooooooooooooooooooooooooooo
1 1 1 1 1 1
7.2.1. + 2 + ··· + 2 > 2 + 2 + ··· + 2 ,
x21 + x1 x2 + x2 xn + xn x1 + x2 x2 + x3 xn + x1
dla x1 , . . . , xn > 0. ([Mat] 4/1997 z.1391).

1 1 1
7.2.2. + ··· + > √ , dla x1 , . . . , xn > 1.
1 + x1 1 + xn 1 + x1 · · · xn
n

([IMO] Shortlist 1998, [Djmp] s.299(636)).

 
1 1 1 nn+1
7.2.3. (1 + x1 )(1 + x2 ) · · · (1 + xn ) + + ··· + > .
x1 x2 xn (n − 1)n−1
([Cmj] 22(2)(1991) 171).

n
X ai n
7.2.4. > , dla a1 , . . . , an > 0, a1 + · · · + an = 1. ([Balk] 1984).
i=1
2 − ai 2n − 1

a1 a2 · · · an (1 − (a1 + · · · + an )) 1
7.2.5. 6 n+1 ,
(a1 + · · · + an )(1 − a1 )(1 − a2 ) · · · (1 − an ) n
dla a1 , . . . , an > 0 i a1 + · · · + an < 1. ([IMO] Shortlist 1998).

1 1 1
7.2.6. k
+ k + · · · + k > nk+1 , dla a1 , . . . , an > 0, n, k ∈ N i a1 + · · · + an = 1.
a1 a2 an
([IMO] Longlist 1974).

1 1 1 8(n − 1)(1 − a1 a2 · · · an
7.2.7. + + ··· + −n> , dla a1 , . . . , an > 0, n, k ∈ N
a1 a2 an n2
oraz a1 + · · · + an = n. (Pham Kim Hung [Pkh] s.209).

7.2.8. Jeśli a1 + · · · + an = 1, a1 , . . . , an > 0, to

a21 a22 a2n−1 a2n 1


+ + ··· + + > .
a1 + a2 a2 + a3 an−1 + an an + a1 2

Równość zachodzi tylko wtedy, gdy a1 = · · · = an = n1 . ([OM] ZSRR).


94 Nierówności 7. Różne nierówności wymierne

7.2.9. ([OM] Mołdawia 2002). Jeśli a1 + · · · + an = 1 oraz u, v, a1 , . . . , an > 0, to

a31 a32 a3n−1 a3n 1


+ + ··· + + > .
ua1 + va2 ua2 + va3 uan−1 + van uan + va1 n(u + v)

7.2.10. ([OM] Mołdawia 2009) Jeśli m, n ∈ N, n > 2, a1 , . . . , an > 0 oraz a1 + · · · + an = 1, to

a2−m + a2 + · · · + an−1 a22−m + a3 + · · · + an a2−m + a1 · · · + an−2 nm − 1


1
+ +···+ n > n+ .
1 − a1 1 − a2 1 − an n−1

7.2.11. Jeśli a1 , . . . , an , b1 , . . . , bn są dodatnimi liczbami rzeczywistymi, to


n
X ak bk AB
> ,
a + bk
k=1 k
A+B

n
X n
X
gdzie A = ak , B = bk . ([Fom] 62/93).
k=1 k=1

7.2.12. Niech a1 , . . . , an będą długościami boków n-kąta (n > 3) i niech p = a1 + · · · + an .


Wtedy:
a1 an n
(1) + ··· + > , ([Kw] 2/1997 44);
p − 2a1 p − 2an n−2
 
p − a1 p − an a1 an
(2) + ··· + > (n − 1)2 + ··· + , ([Ko03] 59);
a1 an p − a1 p − an
n a1 an
(3) 6 + ··· + 6 2, ([IMO] Longlist 1979).
n−1 p − a1 p − an

7.2.13. Jeśli n > 2 oraz x1 , . . . , xn > 0, x21 + · · · + x2n = 1, to

x51 x52 x5n 1


+ + ··· + > .
x2 + x3 + · · · + xn x3 + · · · + xn + x1 x1 + · · · + xn−1 n(n − 1)

Równość zachodzi wtedy i tylko wtedy, gdy x1 = · · · = xn = √1 . ([OM] Turcja, [Mild]).


n

7.2.14. Jeśli a1 , . . . , an > 0 i an+1 = a1 , to


n
! n
Y a2 Y
1+ i > (1 + ai ). ([MC] 12(1999) 28, [KoM] 1999(12) A224, [Ko03] 109).
i=1
ai+1 i=1

7.2.15. Jeśli a1 , . . . , an > 0 i an+1 = a1 , to


n
! n
Y a3 Y
1+ i > (1 + ai ai+1 ). ([Ko03] 113).
i=1
ai+1 i=1
Nierówności 7. Różne nierówności wymierne 95

7.2.16. Niech 0 < p < q i niech a1 , . . . , an ∈ [p, q]. Wtedy


 
1 1 1 kn (q − p)2
(a1 + a2 + · · · + an ) + + ··· + 6 n2 + ,
a1 a2 an 4pq

gdzie kn = n2 dla parzystych n oraz kn = n2 − 1 dla nieparzystych n. ([Pkh] s.77).

a1 an
7.2.17. Jeśli b1 , . . . , bn jest permutacją dodatnich liczb a1 , . . . , an , to b1 + ··· + bn > n.
([Kurs] 115(1935)).

⋆ Kin-Yin Li, Using tangent lines to prove inequalities, [ME] 10(5)(2005) 1-2.
N. Sato, Tips on inequalities, [Crux] 1998 161-167.
N. Siedrakjan, O przekształceniu pewnej nierówności, [Kw] 2(1997) 42-44.
oooooooooooooooooooooooooooooooooooooooooooooooooooooooooooooooooooooo
7.3 Nierówności wymierne jednej zmiennej
oooooooooooooooooooooooooooooooooooooooooooooooooooooooooooooooooooooo
1 1
7.3.1. x8 − x5 − + 4 > 0, dla x 6= 0. ([OM] Irlandia 1998).
x x
xn + x−n − 2
7.3.2. > n2 , dla x > 0, x 6= 1, n ∈ N. ([OM] Węgry-Izrael 1992).
x + x−1 − 2
oooooooooooooooooooooooooooooooooooooooooooooooooooooooooooooooooooooo
7.4 Nierówności wymierne dwóch zmiennych
oooooooooooooooooooooooooooooooooooooooooooooooooooooooooooooooooooooo
7.4.1. Niech x, y > 0 i xy = 1. Jeśli α 6 β, to

xα + y α 6 xβ + y β .

D. Niech β = α + γ. Mamy wtedy:


x2α + 1 x2β + 1
xα + y α 6 xβ + y β ⇐⇒ 6 ⇐⇒ xγ (x2α + 1) 6 x2α+2γ + 1
xα xβ
⇐⇒ x2α+γ + xγ 6 x2α+2γ + 1 ⇐⇒ x2α+2γ − x2α+γ − xγ + 1 > 0

⇐⇒ (xγ − 1) x2α+γ − 1 > 0.

Ostatnia nierówność jest oczywiście prawdziwa. ⊠

7.4.2. Niech x > 0. Jeśli α 6 β, to


1 1
xα + α
6 xβ + β .
x x
(Wynika z 7.4.1).

7.4.3. Niech x1 , . . . , xn > 0, x1 x2 · · · xn = 1 i α 6 β. Czy prawdą jest, że wtedy

xα1 + · · · + xαn 6 xβ1 + · · · + xβn ?

Z 7.4.1 wynika, że tak jest dla n = 2.


96 Nierówności 7. Różne nierówności wymierne

1 1 1
7.4.4. + > dla a, b > 0. ([Pkh] s.161).
(1 + a)2 (1 + b)2 1 + ab
 2  2
1 1 25
7.4.5. a+ + b+ > , dla a, b > 0 i a + b = 1. ([OM] Indie 1988).
a b 2
1 a2 b2 1
7.4.6. 6 + < , dla a, b > 0 i a + b = 1. ([OM] Indie 1997).
3 a+1 b+1 2
1
7.4.7. a b
6 ax + by, dla a, b, x, y > 0, a + b = 1. ([OM] Grecja 2001).
x + y

ab √
7.4.8. 6 2 − 1, gdy a2 + b2 = 4 i a, b > 0. ([OM] Austria 1989).
a+b+2

7.4.9. Jeśli 0 6 a 6 b 6 1, to:


b−a
(1) 0 6 6 1,
1 − ab
a b
(2) 0 6 + 6 1,
1+b 1+a
1
(3) 0 6 ab2 − ba2 6 . ([MaOD] 4).
4
a2 b2
7.4.10. + > 8, dla a > 1, b > 1. ([OM] ZSRR 1992).
b−1 a−1
 m  m
a b
7.4.11. 1+ + 1+ > 2m+1 , dla a, b > 0, m ∈ Z.
b a
([IMO] Shortlist 1968, [Djmp] s.53(263)).

a b
7.4.12. + > 1, dla a, b > 0, a + b 6 2. ([OM] St Petersburg 1995).
b + ab a + ab
a b 1
7.4.13. + 4 6 , dla a, b > 0. ([Kw] 5/1995).
a4 +b 2 b +a 2 ab
as+1 bs+1 (a + b)s+1
7.4.14. + > , dla a, b, x, y, s > 0. ([Bryn] 4.3).
xs ys (x + y)s
1 1 2
7.4.15. + > , gdy |a| < 1 i |b| < 1.
1−a 2 1−b 2 1 − ab
([OM] Rosja 1993, [MaS] 2/1993).

7.4.16. Jeśli f : R+ → R+ jest funkcją rosnącą, to


1 1 1 1
+ > + ,
f (a) + a f (b) + b f (a) + b f (b) + a
dla a, b ∈ R+ . ([Bedn] 62).
Nierówności 7. Różne nierówności wymierne 97

oooooooooooooooooooooooooooooooooooooooooooooooooooooooooooooooooooooo
7.5 Nierówności wymierne trzech zmiennych
oooooooooooooooooooooooooooooooooooooooooooooooooooooooooooooooooooooo

7.5.1. Jeśli a, b, c > 0, to:


a b c 9
(1) + + > , ([OM] Indie 1997);
1 + bc 1 + ca 1 + ab 10
a b c 9
(2) + 2 + 2 > , ([OM] Serbia i Czarnogóra);
b +c c +a a +b
2 4
3 6
(3) 1 + > , ([MaOD] 32);
ab + bc + ca a+b+c
1 1 1 3
(4) + + > ,
ab + a bc + b ca + c 1 + abc
(M. Aassila, [Crux] z.2362, [OM] Indie 1997, [KoM] 2003 A324);
  
1 1 1 1 1 1 9
(5) + + + + > , (W. Janous, [Pkh] s.145);
a b 1+a 1+b 1+c
c 1 + abc
c+1 a+1 b+1
(6) a+b+c>a +b +c , ([OM] Rosja 1998);
a+1 b+1 c+1
1 1 1 a b c 3(a + 1)(b + 1)(c + 1)
(7) 3+a+b+c+ + + + + + > , ([Kw] 1988);
a b c b c a 1 + abc
1 + a2 1 + b2 1 + c2
(8) + + > 2, (IBMO 2002);
a2 + b + 1 b2 + c + 1 c2 + a + 1
a2 − b2 b2 − c2 c2 − a2
(9) + + 6 1, ([OM] Grecja 2005);
2a2 + 1 2b2 + 1 2c2 + 1
1 + bc 1 + ac 1 + ab 1 1 1
(10) + + > + + , dla różnych a, b, c,
a(a − b)(a − c) b(b − a)b − c) c(c − a)(c − b) a b c
([Ssm] 103(1)(2003));
   
a b c 2(a + b + c)
(11) 1+ 1+ 1+ >2+2 √ , ([A-P] 1998, [Pkh] s.18).
b c a 3
abc

a4 b4 c4
7.5.2. + + > 48, dla a, b, c > 1. ([ME] 13(5)(2009)).
(b − 1)2 (c − 1)2 (a − 1)2

a b c
7.5.3. < + , gdy a, b, c > 0 i a < b + c.
1+a 1+b 1+c
(Stellenbosch, Léo Sauvé, [Crux] z.54).

1 1 1 1 1 1
7.5.4. + 2 + 2 > 2 + 2 + 2 , dla a, b, c > 0.
a2 +a b +b c +c a +b b +c c +a
([Mat] 4/1997 z.1391).

a2 + 3ab b2 + 3bc c2 + 3ca


7.5.5. + + 6 2, dla a, b, c > 0, a + b + c = 1. ([Kw] 6/2008 14).
a+b b+c c+a
98 Nierówności 7. Różne nierówności wymierne

D. ([Kw]). Dla dowolnych dodatnich liczb rzeczywistych x, y zachodzi nierówność


2xy x+y
6 .
x+y 2

Mamy bowiem: 4xy 6 (x + y)2 ⇐⇒ 0 6 x2 − 2xy + y 2 = (x − y)2 . Z tej nierówności wynika, że

x2 + 3xy (x2 + xy) + 2xy 2xy 3 1


= =x+ = x + y.
x+y x+y x+y 2 2
a2 + 3ab b2 + 3bc c2 + 3ca 3 1
Mamy zatem: + + 6 (a + b + c) + (a + b + c) = 2. ⊠
a+b b+c c+a 2 2

7.5.6. Jeśli a, b, c > 0 oraz a + b + c = 1, to:


1 1 1
(1) + + > 2, ([Kw] 2/2007 24);
1 + 4a 2 1 + 4b 2 1 + 4c2
1 1 1 25
(2) + + > , ([Kw] 6/2006 19);
a b c 1 + 48abc
1 1 1 2 2 2
(3) + + > + + , ([OM] Rosja 2003, [Kw] 2/2004 M1881);
a+b b+c c+a 1+a 1+b 1+c
a b c 3
(4) + + 6 , ([OM] Polska 1987);
a+1 b+1 c+1 4
1+a 1+b 1+c 2a 2b 2c
(5) + + 6 + + , ([OM] Japonia 2004, [Pkh] s.42);
1−a 1−b 1−c b c a
a b c 3
(6) + + > , ([OM] Rosja 1998);
b+c+1 c+a+1 a+b+1 5
ab bc ca 1
(7) + + 6 , ([MOc] 2000, [OMm] 1998/1999);
c+1 a+1 b+1 4
a2 + b b2 + c c2 + a
(8) + + > 2, ([AnC]);
b+c c+a a+b
a − bc b − ca c − ab 3
(9) + + 6 , ([OM] Kanada 2008);
a + bc b + ca c + ab 2
a2 b2 c2 3
(10) + + > , ([Cmj] 19(3)(1988) 291-292);
a + bc b + ca c + ab 4
a3 b3 c3 1
(11) + + > , ([OM] Chorwacja 1999, [MOc] 2002 z.161);
a2 + b2 b2 + c2 c2 + a2 2
 
a b c 3
(12) (ab + bc + ca) 2 + 2 + 2 > , (G. Dospinescu, [Crux] z.3062);
b +b c +c a +a 4
 2  2  2
1 1 1 100
(13) a+ + b+ + c+ > , ([MaOD] 30);
a b c 3
a7 + b7 b7 + c7 c7 + a7 1
(14) + 5 + 5 > , ([OM] Kazachstan 2000).
a +b
5 5 b +c 5 c +a 5 3

   
1 1 1
7.5.7. 1+ 1+ 1+ > 64, dla a, b, c > 0, a + b + c = 1. ([Crux] 1998 s.164).
a b c
Nierówności 7. Różne nierówności wymierne 99

   
1 1 1
7.5.8. −1 −1 − 1 > 8, dla a, b, c > 0, a + b + c = 1. ([Crux] 1998 s.167).
a b c
   
1 1
7.5.9. − 1 ··· − 1 > 1024, dla a1 , . . . , a5 > 0, a1 + · · · + a5 = 1. ([WyKM] 447-65).
a1 a5

7.5.10. Jeśli a, b, c > 0 oraz a + b + c = 3, to:


1 1 1
(1) + + > 1, (Pham Kim Hung, [Pkh] s.211);
2 + 2a b
2 2 2+b c
2 2 2 + c2 a2
ab + bc + ca a3 + b3 + c3
(2) > , (([Pkh] s.160);
a3 b3 + b3 c3 + c3 a3 36
1 1 1
(3) + + > 2, ([Pkh] s.30);
1 + 2b2 c 1 + 2c2 a 1 + 2a2 b
1 1 1
(4) 2
+ 2 + 2 > a2 + b2 + c2 , (V. Cirtoaje, [Pkh] s.56);
a b c
1 1 1
(5) + + 6 1, ([Pkh] s.59);
a2 + b + c a + b2 + c a + b + c2
1 1 1 3
(6) + + 6 , ([Pkh] s.62);
9 − ab 9 − bc 9 − ca 8
a b c 3
(7) + + > , ([Pkh] s.237);
1+b 3 1+c 3 1+a 3 2
2+a 2+b 2+c
(8) a2 + b2 + c2 > + + , ([Pkh] s.147);
2+b 2+c 2+a
a b c 3
(9) + 2 + 2 > , ([OM] Bułgaria 2003, [Crux] z.2994, [Pkh] s.27);
b +1 c +1 a +1
2 2
a b c 3
(10) + 2 + 2 > , ((Pham Kim Hung, [Pkh] s.247);
b +c c +a a +b
2 2
a b c 3
(11) + + > , (([Pkh] s.35);
1 + ab 1 + bc 1 + ca 2
a+1 b+1 c+1
(12) + 2 + 2 > 3, ([Pkh] s.30);
b +1 c +1 a +1
2

a2 b2 c2
(13) + + > 1, ([Pkh] s.29);
a + 2b3 b + 2c3 c + 2a3
a b c 3
(14) + + > , ([Crux] z.2994);
b+1 c+1 a+1 2
a2 b2 c2 3
(15) + + > , ([Crux] z.2994);
b +1 c +1 a +1
2 2 2 2
a2 b2 c2
(16) + + > 1, ([Pkh] s.29);
a + 2b2 b + 2c2 c + 2a2
a2 b2 c2 3
(17) + + > . ([Crux] z.2994).
b+1 c+1 a+1 2
100 Nierówności 7. Różne nierówności wymierne

1 1 1 1
7.5.11. + + 6 ,
2ab + ac + bc 2bc + ba + ca 2ca + cb + ab abc
dla a, b, c > 0, a + b + c = 4. ([MOc] z.556).

a b c
7.5.12. + + 6 2, dla a, b, c ∈ [0, 1]. ([Fom] 25/90).
1 + bc 1 + ac 1 + ab

a b c
7.5.13. + + + (1 − a)(1 − b)(1 − c) 6 1, dla a, b, c ∈ [0, 1].
b+c+1 c+a+1 a+b+1
([Mild]).

a b c 3
7.5.14. + + 6 , dla a, b, c ∈ (0, 1].
1 + b + ca 1 + c + ab 1 + a + bc a+b+c
([OM] Ukraina 1998, [Crux] 2003 s.445).

a b c 3x √
7.5.15. + + > , dla a, b, c ∈ (0, 1), gdzie x = 3 abc. ([OM] Irlandia 2002).
1−a 1−b 1−c 1−x

a+b b+c c+a


7.5.16. 2 6 + + 6 3, dla a, b, c ∈ [ 12 , 1]. ([OM] Rumunia 2006).
1+c 1+a 1+b
 
1 1 1
7.5.17. (a + b + c) + + 6 10, dla a, b, c ∈ [1, 2]. (Patrz 7.2.16, [Pkh] s.76).
a b c
   
1 1 1 a b c
7.5.18. (a + b + c) + + >6 + + , dla a, b, c ∈ [1, 2].
a b c b+c c+a a+b
([OM] Wietnam 2006, [Pkh] s.146).

7.5.19. Liczby a, b, c są długościami boków trójkąta.


1 1 1 1 1 1
(1) + + 6 + + . ([OM] Polska 1993/1994);
a b c b+c−a c+a−b a+b−c
 
a b c a b c
(2) 2 + + > + + + 3, ([Kw]);
b c a c a b
   
a b c a b c
(3) 3 + + −1 >2 + + , (Vasile Cirtoaje, [Mild]);
b c a c a b
3 a b c
(4) 6 + + < 2, ([OM] Indie 89, [OM] Norwegia 1993, [Pa97]);
2 b+c c+a a+b
a b c ab + bc + ca 5
(5) + + + 6 , ([Pkh] s.202);
b + c c + a a + b a2 + b2 + c2 2
a b c
(6) + + > 3, ([Kw]);
b+c−a c+a−b a+b−c
a b c b+c−a c+a−b a+b−c
(7) + + > + + , ([Zw] z.40);
b+c−a c+a−b a+b−c a b c
a b c
(8) + + > 1, (S.Riasa,, [Pkh] s.41);
3a − b + c 3b − c + a 3c − a + b
Nierówności 7. Różne nierówności wymierne 101

!  
a2 b2 c2 2 2 1
2 1 1
(9) 3 2 + 2+ 2 > (a + b + c ) 2 + 2 + 2 , ([MM] 44(3)(1971) 172);
b c a a b c
a2 + 2bc b2 + 2ac c2 + 2ab
(10) + 2 + 2 > 3, ([OM] Moskwa 1999);
b2 + c2 c + a2 a + b2
a2 b2 c2 √
(11) + + > 3 3R, ([OM] Bośnia Hercegowina 1999).
b+c−a c+a−b a+b−c

7.5.20. Jeśli a, b, c są długościami boków trójkąta, to

a−b b−c c−a 1


+ + < . ([Ibe] 1989, [OMm] 1999/2000).
a+b b+c c+a 16

7.5.21. Znaleźć największą liczbę naturalną n taką, że dla każdego trójkąta o bokach a, b, c
zachodzi nierówność
a−b b−c c−a 1
+ + < .
a+b b+c c+a n
Odp. n = 23. (P. Kumor, [Mat] 4/2004 z.1629, rozw. [Mat] 3/2005).

7.5.22. Jeśli a2 + b2 + c2 = 1 i a, b, c > 0, to:



a b c 3 3+3
(1) + + > ([Ko03] 47);
1−a 1−b 1−c 2
1 1 1 9
(2) + + 6 , (V. Cirtoaje, [Crux] z.3032);
1 − ab 1 − bc 1 − ca 2

a b c 3 3
(3) + + > ([Crux] 2003 s.243, [OM] Brazylia 2004, [Ko03] 47);
1 − a2 1 − b2 1 − c2 2
a b c (n + 1)(n+1)/n
(4) + + > (T. Zvonaru, [Crux] z.2935);
1 − an 1 − bn 1 − cn n
1 1 1 2(a3 + b3 + c3 )
(5) + + > 3 + (Hojoo Lee, [Crux] z.2532);
a2 b2 c2 abc
a b c 3 √ √ √
(6) 2 + 2 + 2 > (a a + b b + c c), (Mediterranean Math. Comp. 2002);
b +1 c +1 a +1 4
a b c
(7) 3 + 3 + 3 > 3, ([Pkh] s.171);
a + bc b + ca c + ab
a2 b2 c2 3
(8) + + > ([OM] Bośnia Hercegowina 2002);
1 + 2bc 1 + 2ac 1 + 2ab 5
  √
1 1 1
(9) + + − (a + b + c) > 2 3 (P. E. Tsaoussoglou, [Crux] z.2946);
a b c
  √
1 1 1
(10) + + + (a + b + c) > 4 3 (P. E. Tsaoussoglou, [Crux] z.2946);
a b c
1 √
(11) a + b + c + > 4 3. ([OM] Kazachstan 2000).
abc
102 Nierówności 7. Różne nierówności wymierne

7.5.23. Jeśli a2 + b2 + c2 = 3 i a, b, c > 0, to:


1 1 1
(1) + + > 3, ([Pkh] s.165);
2−a 2−b 2−c
a b c
(2) + + 6 1, (V.Cirtoaje, [Pkh] s.200);
a+2 b+2 c+2
a b c 1
(3) + 2 + 2 6 , (Pham Kim Hung, [Pkh] s.145));
a + 2b + 3 b + 2c + 3 c + 2a + 3
2 2
1 1 1 3
(4) + + > , ([OM] Białoruś 1999);
1 + ab 1 + bc 1 + ca 2
1 1 1
(5) + + > 1, ([OM] Estonia 2004);
1 + 2ab 1 + 2bc 1 + 2ca
1 1 1 1 1 1
(6) + + + + + > 3, ([Pkh] s.181));
3 − ab 3 − bc 3 − ca 3 − a 2 3−b 2 3 − c2
1 1 1
(7) + 3 + 3 > 1, (Pham Kim Hung [Pkh] s.31).
a +2 b +2 c +2
3

1 1 1
7.5.24. + + 6 1, dla a4 + b4 + c4 = 3 i a, b, c > 0.
4 − ab 4 − bc 4 − ca
([OM] Mołdawia 2005, [Pkh] s.64).


a3 b3 c3 943
7.5.25. + + > , dla a4 + b4 + c4 = 1 i a, b, c > 0. ([Ko03] 47).
1 − a8 1 − b8 1 − c8 8

7.5.26. Jeśli ab + bc + ca = 1 i a, b, c > 0, to:


1 1 1 5
(1) + + > , (Berkely Math. Circle, [Pkh] s.159);
a+b b+c c+a 2
1 1 1 1
(2) + + + > 3, ([Pkh] s.159).
a+b b+c c+a a+b+c

⋆ P. Aleksiejew, L. Kurlandczyk, Boki trójkąta (po rosyjsku), Matematiczeskij Krużok 3/99, 42-45.
oooooooooooooooooooooooooooooooooooooooooooooooooooooooooooooooooooooo
7.6 Nierówności wymierne czterech zmiennych
oooooooooooooooooooooooooooooooooooooooooooooooooooooooooooooooooooooo
a b c d 1
7.6.1. + + + 6 , dla a, b, c, d > 0, a + b + c + d < 1. ([OM] Indie 1995).
b a d c 64abcd

a2 b2 c2 d2 1
7.6.2. + + + > , dla a, b, c, d > 0, a + b + c + d = 1.
a+b b+c c+d d+a 2
Równość zachodzi dokładnie wtedy, gdy a = b = c = d = 14 . ([OM] Irlandia 1999).

7.6.3. Jeśli a, b, c, d > 0 oraz a + b + c + d = 4, to:


1 1 1 1
(1) + + + 6 1, (V.Cirtoaje, [Pkh] s.27);
5 − abc 5 − bcd 5 − cda 5 − dab
Nierówności 7. Różne nierówności wymierne 103

1 1 1 1
(2) + 2 + 2 + 2 > 2, ([Pkh] s.30);
a2 +1 b +1 c +1 d +1
a b c d
(3) + + + > 2, ([Pkh] s.27);
b2 + 1 c2 + 1 d2 + 1 a2 + 1
1 1 1 1 1
(4) + 2 + 2 + 2 6 , ([Pkh] s.55);
a + 11 b + 11 c + 11 d + 11
2 3
a+1 b+1 c+1 d+1
(5) + 2 + 2 + 2 > 4, ([Pkh] s.30);
b +1 c +1 d +1 a +1
2

a b c d
(6) + 2 + 2 + 2 > 2, ([Pkh] s.28);
b2 c +1 c d+1 d a+1 a b+1
1 + ab 1 + bc 1+c 1+d
(7) + + + > 4, ([Pkh] s.30).
1+b c2 2 1+c d2 2 1+d a
2 2 1 + a2 b2

a+b c+d a+c


7.6.4. + 64 , dla a, b, c, d ∈ [1, 2]. ([OM] Ukraina 1992).
b+c d+a b+d

7.6.5. Jeśli a, b, c, d > 0 oraz a2 + b2 + c2 + d2 = 4, to:


1 1 1 1
(1) + + + 6 1, (Pham Kim Hung [Pkh] s.61);
5−a 5−b 5−c 5−d
a2 b2 c2 d2 4
(2) + + + > , ([Pkh] s.54);
b+c+d c+d+a d+a+b a+b+c 3
1 1 1 1
(3) + + + 6 2, (Pham Kim Hung [Pkh] s.195).
3 − abc 3 − bcd 3 − cda 3 − dab

a3 b3 c3 d3 1
7.6.6. + + + > ,
b+c+d c+d+a d+a+b a+b+c 3
dla a, b, c, d > 0, ab + bc + cd + da = 1. ([IMO] Shortlist 1990, [Djmp] s.253(540)).

oooooooooooooooooooooooooooooooooooooooooooooooooooooooooooooooooooooo
7.7 Nierówności wymierne dla liczb całkowitych
oooooooooooooooooooooooooooooooooooooooooooooooooooooooooooooooooooooo
1 1 3 1997 1
7.7.1. < · · ··· · < . ([OM] Kanada 1997).
1999 2 4 1998 44
n
7.7.2. nn > (n + 1)n−1 + , dla 2 6 n ∈ N. ([Miss] z.153).
n+1
 n
1
7.7.3. 1+ < 2, dla n ∈ N, n > 1. ([Bryn] 4.1).
n2
  !
1 1 1
7.7.4. 1 − 2 · · · 1 − 2 > , gdzie k > 2 oraz n1 , . . . , nk są parami różnymi liczbami
n1 nk 2
naturalnymi większymi od 1. ([WyKM] z.553).
104 Nierówności 7. Różne nierówności wymierne

D. ([WyKM] s.161). Niech m będzie największą z liczb n1 , . . . , nk . Wtedy m > 3 oraz


         
1 1 1 1 1 1
1− 2 1 − 2 ··· 1 − 2 > 1 − 2 1 − 2 ··· 1 − 2 .
n1 n2 nk 2 3 m
Prawa strona tej nierówności jest równa
(1 · 3)(2 · 4)(3 · 5) · · · ((m − 1)(m + 1)) m+1 1 1
= = +
22 · 32 · ṁ2 2m 2 2m
1 1
i oczywiście 2 + 2m > 21 . ⊠
 n  n  n
3n + 1 1 2 n
7.7.5. < + + ··· + < 2, dla n > 1. ([Putn] 1962).
2n + 2 n n n
2 22 23 2n+1
7.7.6. + 2 + 4 + · · · + 2n < 1, dla n > 1. ([OM] Mołdawia 2009).
3+1 3 +1 3 +1 3 +1
n  2  2
X 1 n s
7.7.7. ai + >n + , dla a1 , . . . , an ∈ N, gdzie s = a1 + · · · + an .
i=1
ai s n
([OM] Wietnam 1980).

7.7.8. Jeśli x1 , . . . , xn są parami różnymi liczbami naturalnymi, to


x31 + x32 + · · · + x3n n(n + 1)
> . ([Mon] 2/2005 z.11012).
x1 + x2 + · · · + xn 2
a21 a22 a2 n(n + 1)
7.7.9. + +···+ n > , gdy {a1 , . . . , an } jest permutacją zbioru {1, 2, . . . , n}.
1 2 n 2
([Bedn] 182).

   
1 1 1 50
7.7.10. 1+ 2+ 3+ 6 , dla parami różnych liczb naturalnych x, y, z.
x y c 3
1
D. (Sposób I). Niech a1 = 1, a2 = 2, a3 = 3, h(x) = oraz
x
   
1 1 1
H(x, y, z) = 1+ 2+ 3+ .
x y z
50
Zauważmy, że 3 = H(1, 2, 3). Teza wynika więc z twierdzenia 5.8.12.
1
 1

1

(Sposób II). Powtarzamy dowód twierdzenia 5.8.12. Niech H(x, y, z) = 1 + x 2+ y 3+ z .
50
Zauważmy, że = f (1, 2, 3). Niech x, y, z będą parami różnymi liczbami naturalnymi. Rozpatrzmy 6
3
przypadków.
Przypadek 1: x < y < z. W tym przypadku x > 1, y > 2 oraz z > 3. Mamy więc:
     
1 + x1 2 + y1 3 + z1 6 1 + 11 2 + 12 3 + 31 = f (1, 2, 3) = 50 3 .

Przypadek 2: x < z < y. W tym przypadku x > 1, y > 3 oraz z > 2. Mamy więc:
     
1 + x1 2 + y1 3 + z1 6 1 + 11 2 + 13 3 + 12 = f (1, 3, 2) = 49 50
3 < 3 .

Przypadek 3: y < x < z. W tym przypadku x > 2, y > 1 oraz z > 3. Mamy więc:
     
1 + x1 2 + y1 3 + z1 6 1 + 12 2 + 11 3 + 13 = f (2, 1, 3) = 15 < 503 .

W ten sam sposób postępujemy w następnych trzech przypadkach. ⊠


Nierówności 7. Różne nierówności wymierne 105

   
1 1 1 91
7.7.11. 1 + 2+ 3+ 6 , dla parami różnych liczb naturalnych x, y, z wię-
x y z 8
kszych od 1. (D. Marghidanu, [Crux] z.3040).
   
1 1 1 91
D. Niech H(x, y, z) = 1 + 1+x 2+ 1+y 3+ 1+z . Zauważmy, że H(1, 2, 3) = 8 . Teza wy-
nika więc z twierdzenia 5.8.12. ⊠

Następne dwie nierówności również są konsekwencjami twierdzenia 5.8.12.


    
1 1 1 1 425
7.7.12. 1+ 2+ 3+ 4+ 6 , dla parami różnych liczb naturalnych
a b c d 6
a, b, c, d.
    
1 1 1 1
7.7.13. 1+ 2+ 3+ 4+ 6 40, dla parami różnych liczb naturalnych
a b c d
a, b, c, d, większych od 2.

Zanotujmy podobnego typu nierówności dla pięciu i sześciu zmiennych.

7.7.14. Jeśli x1 , x2 , . . . , x5 są parami różnymi liczbami naturalnymi, to


     
1 1 1 1 1 1105
1+ 2+ 3+ 4+ 5+ 6 .
x1 x2 x3 x4 x5 3
    
1 1 1 1105
D. Niech H(x1 , . . . , x5 ) = 1 + x1 2+ x2 ··· 5 + x5 . Zauważmy, że H(1, 2, 3, 4, 5) = 3 .
Teza wynika więc z twierdzenia 5.8.12. ⊠

7.7.15. Jeśli x1 , x2 , . . . , x6 są parami różnymi liczbami naturalnymi większymi od 2, to


      
1 1 1 1 1 1
1+ 2+ 3+ 4+ 5+ 6+ 6 1260.
x1 x2 x3 x4 x5 x6
    
1 1 1
D. Niech H(x1 , . . . , x6 ) = 1 + 2+x1 2+ 2+x2 ··· 6 + 2+x6 . Wtedy H(1, 2, 3, 4, 5, 6) = 1260
i teza wynika więc z twierdzenia 5.8.12. ⊠

7.7.16. W poniższych tabelkach podano przykłady takich trójek (n, m, c), że nierówność
     
1 1 1 1
1+ 2+ 3+ ··· n + <c
x1 x2 x3 xn

zachodzi dla parami różnych liczb naturalnych większych od m.

n m c n m c n m c n m c n m c
3 0 17 3 1 12 3 2 10 3 3 9 3 4 9
4 0 71 4 1 48 4 2 41 4 3 37 4 4 34
5 0 269 5 1 247 5 2 206 5 3 185 5 4 173
6 0 2272 6 1 1517 6 2 1261 6 3 1130 6 4 1051
106 Nierówności 7. Różne nierówności wymierne

7.7.17. W poniższych tabelkach podano przykłady takich trójek (n, m, c), że nierówność
     
1 1 1 1 1 1 1 1
+ + + ··· + 6c
1 x1 2 x2 3 x3 n xn
zachodzi dla parami różnych liczb naturalnych większych od m.

n m c n m c n m c
3 0 16/9 3 1 125/144 3 2 3/5
4 0 625/576 4 1 9/20 4 2 2401/8640
5 0 27/50 5 1 16807/86400 5 2 512/4725
6 0 117649/518400 6 1 1024/14175 6 2 6561/179200

1
Każdą liczbę wymierną postaci, gdzie n ∈ N, nazywamy ułamkiem prostym. Przedsta-
n
wiamy nierówności z ułamkami prostymi, które pojawiły się w [N-1].

7.7.18. Jeśli (an ) jest ciągiem takim, że a1 = 2 i an+1 = a1 a2 · · · an + 1, dla n ∈ N, to


1 1 1
+ + ··· + < 1.
a1 a2 an
([Fom] 38/86, [N-1]).

1 1 1 1 5
7.7.19. Jeśli x, y są liczbami naturalnymi takimi, że + < 1, to + 6 .
x y x y 6
1 1 1 1 1 1 41
7.7.20. Jeśli x, y, z są liczbami naturalnymi takimi, że + + < 1, to + + 6 .
x y z x y z 42
([Fom] 15/86).

7.7.21. Niech x1 , x2 , . . . , xn będą takimi liczbami naturalnymi, że żadna z nich nie jest po-
czątkowym fragmentem żadnej innej (na przykład 12 jest początkowym fragmentem liczb 12,
125 lub 12405). Zachodzi wtedy nierówność:
1 1 1
+ + ··· + < 3. ([Balt] 2000).
x1 x2 xn
s 
1 1 1 1 n k + 1
7.7.22. + + + ··· + > n − 1, dla n, k ∈ N.
kn kn + 1 kn + 2 kn + n − 1 k
([OM] Izrael 1995).

1 1 1 13
7.7.23. + + ··· + > , dla n > 2. ([BaL] 463, [Szn] 1.70, [G-if] 102).
n+1 n+2 2n 24
1 1 1 3
7.7.24. + + ··· + > , dla n > 3. ([G-if] 204, [N-1]).
n+1 n+2 2n 5
1 1 1
7.7.25. + + · · · + 2 > 1. ([Kurs] 125(1938)).
n n+1 n
Nierówności 7. Różne nierówności wymierne 107

1 1 1
7.7.26. 1 < + + ··· + < 2. ([BoL] 51 s.55, [Szn] 1.70).
n+1 n+2 3n + 1
7.7.27. Dla dowolnej liczby naturalnej n oznaczmy:
1 1 1
h(n) = 1 + + + ··· + .
2 3 n
Przyjmujemy dodatkowo, że h(0) = 0.
1
(1) n(n + 1) n < n + h(n), dla n > 2. ([Putn] 1975).
1
− n−1
(2) (n − 1)n < n − h(n), dla n > 3. ([Putn] 1975).
√   
1
(3) n n
n + 1 − 1 < h(n) < n 1 − √ + 1. ([Kw] 8/77 45).
n
n
n
(4) < h (2n − 1) < n, dla n > 2. ([BoL] 40 s.55).
2
(5) Jeśli h(n) > m, to h(3n) > m + 1. ([OM] Litwa 1993).
 
2 1 1 1
(6) h(n) > 2 h(2) + h(3) + · · · + h(n) , dla n > 2. ([OM] Mołdawia 1998).
2 3 n
7.7.28. Dla każdej liczby naturalnej n zachodzi nierówność
X1 n
ϕ(m) X 1
> ,
k m k=1 k

gdzie sumowanie po lewej stronie przebiega wszystkie liczby naturalne k takie, że 1 6 k 6 n


i nwd(k, m) = 1. Przez ϕ(m) oznaczamy liczbę wszystkich liczb naturalnych mniejszych lub
równych m i względnie pierwszych z m. ([IMO] 1978, [KoM] 2000(5) A240).

1 1 1 2
7.7.29. + 2 + ··· + 2 < . ([Kw] 8/78 47).
22 3 n 3
1 1 1 1 1
7.7.30. + 2 + 2 + ··· + 2 < 2 − . ([Crux] 1998 s.170).
12 2 3 n n
1 1 1 1 3n
7.7.31. + 2 + 2 + ··· + 2 > . ([Crux] z.108).
12 2 3 n 2n + 1
m 1 1 1 m
7.7.32. < + +···+ < . ([Siw] 75).
(n + 1)(n + m + 1) (n + 1)2 (n + 2)2 (n + m)2 n(n + m)

1 1 1 1
7.7.33. + 2 + ··· + < . ([BoL] 41 s.55, [Dlt] 1/77 6).
32 5 (2n + 1)2 4

7.7.34. Jeśli (an ) jest ciągiem arytmetycznym o wyrazach dodatnich i różnicy r > 0, to
1 1 1 1 1
2 + 2 + ··· + 2 < 2 +
a1 a2 an a1 ra1
dla n ∈ N. ([Bedn] 103).
108 Nierówności 7. Różne nierówności wymierne

1 1 1
7.7.35. Jeśli x, y, z są liczbami naturalnymi takimi, że + = 2 , to nwd(x, y) > 2.
x3 y 3 z

1 1 1 5
7.7.36. + 3 + ··· + 3 < . ([IMO] Longlist 1969, [OM] Grecja 2005).
1 3 2 n 4
1 1 1 1
7.7.37. + + ··· + 3 < . ([OM] Irlandia 1990).
33 43 n 12
     
1 1 1 1 1
7.7.38. 1− 1− 1− ··· 1 − 3 > . ([IMO] Longlist 1971).
23 33 4 3 n 2

7.7.39. Oznaczmy przez a(n) sumę 11 + 22 + · · · + nn . Dla n > 3 zachodzą następujące


nierówności.
(1) 3a(n) > (n + 1)n .
(2) 2a(n) < (n + 1)n .
1 1 1 1
(3) > + + + ···. ([Kw] 6/95 M1493).
n n a(n) a(n + 1) a(n + 2)

7.7.40. Niech (an ) będzie ciągiem parami różnych liczb naturalnych, których rozwinięcia
dziesiętne nie zawierają cyfry 0. Wtedy

X 1
< 29.
n=1
an

([Br80] 89, [B-rs] 236).

7.7.41. Niech (an ) będzie ciągiem parami różnych liczb naturalnych, których rozwinięcia dzie-

X 1
siętne nie mają na początku cyfry 9. Wtedy szereg jest zbieżny. ([MM] 21(2)(1947) s.112).
a
n=1 n

7.7.42. Niech (an ) będzie ciągiem parami różnych liczb naturalnych, których rozwinięcia
dziesiętne nie zawierają cyfry 9. Wtedy

X 1
< 28.
n=1
an

([Br83] s.77).
⊡⊡⊡⊡⊡⊡⊡⊡⊡⊡⊡⊡⊡⊡⊡⊡⊡⊡⊡⊡⊡⊡⊡⊡⊡⊡⊡⊡⊡⊡⊡⊡⊡⊡⊡⊡⊡⊡⊡⊡⊡⊡⊡⊡⊡⊡⊡⊡⊡⊡⊡⊡
8 Nierówności z pierwiastkami
⊡⊡⊡⊡⊡⊡⊡⊡⊡⊡⊡⊡⊡⊡⊡⊡⊡⊡⊡⊡⊡⊡⊡⊡⊡⊡⊡⊡⊡⊡⊡⊡⊡⊡⊡⊡⊡⊡⊡⊡⊡⊡⊡⊡⊡⊡⊡⊡⊡⊡⊡⊡

oooooooooooooooooooooooooooooooooooooooooooooooooooooooooooooooooooooo
8.1 Nierówności z pierwiastkami n zmiennych
oooooooooooooooooooooooooooooooooooooooooooooooooooooooooooooooooooooo

8.1.1. Dla dodatnich liczb x1 , . . . , xn zachodzą następujące nierówności:


√ √ √ √
(1) x1 + · · · + xn 6 ( 2 + 1) x1 + · · · + xn , ([IMO] 1987);
x1 x2 xn n
(2) q +q + ··· + q > √ , dla x1 , . . . , xn > 0,
x1 x2 + x22 x2 x3 + x23 xn x1 + x21 2
([Crux] 2003 s.403).

8.1.2. Niech x1 , . . . , xn > 0, x1 + · · · + xn = 1. Wtedy:


√ √ √ √ √ √ √
(1) n − 1 ( x1 + x2 + · · · + xn ) 6 1 − x1 + 1 − x2 + · · · + 1 − xn , ([Khr3] 22);
s s s
1 − x1 1 − x2 1 − xn 2
(2) n−16 + + ··· + 6n−2+ √ , ([Crux] z.2053);
1 + x1 1 + x2 1 + xn 3
√ √ √
x1 + x2 + · · · + xn x1 x2 xn
(3) √ 6√ +√ + ··· + √ ,
n−1 1 − x1 1 − x2 1 − xn
([OM] Chiny 1988, [Liu1] 123-124);
n
X xi π
(4) 16 √ √ < , gdzie x0 = 0.
i=1
1 + x0 + x1 + · · · + xi−1 · xi + xi+1 + · · · + xn 2
[OM] Chiny 1995/1996, [Liu2] 83-84).
q q q
8.1.3. x21 + 1 + · · · + x2n + 1 > 2n(x1 + · · · + xn ), dla x1 , . . . , xn > 0, x1 · · · xn = 1.
(G.Dospinescu, [Crux] 1996 s.135, [Pkh] s.149).

8.1.4. Niech x1 , . . . , xn > 0, x1 x2 · · · xn = 1. Wtedy:


√ √
(1) x1 + · · · + xn 6 x1 + · · · + xn , ([OM] Mołdawia 1998);
r
x x2 xn n
(2) √ 1 +√ + ··· + √ > , ([OM] Indie 1995);
1 − x1 1 − x2 1 − xn n−1
s s
x21 + 1 x2n + 1
(3) + ··· + 6 x1 + · · · + xn , (Gabriel Dospinescu, [Mild]).
2 2
√ √ √
8.1.5. x1 + · · · + xn 6 x1 x2 + 1 + x2 x3 + 1 + · · · + xn x1 + 1 < n + x1 + · · · + xn ,
dla x1 , . . . , xn ∈ [0, 2]. ([Kw] 2/2005 15).

109
110 Nierówności 8. Nierówności z pierwiastkami

oooooooooooooooooooooooooooooooooooooooooooooooooooooooooooooooooooooo
8.2 Nierówności z pierwiastkami dwóch zmiennych
oooooooooooooooooooooooooooooooooooooooooooooooooooooooooooooooooooooo

8.2.1. Jeśli a, b > 0, to:


√ 1 √  a+b
(1) ab 6 a + ab + b 6 , ([MM] 78(5)(2005) 397);
3 2
s
(a − b)2 a2 + b2 √ (a − b)2
(2) 6 − ab 6 √ , ([KoM] 1998(1) Gy3175);
2(a + b) 2 2(a + b)
√ √ a b
(3) a + b 6 √ + √ , ([MaOD] 4);
b a
√ √ √
a + a b + ab2 + b a + ab + b
3 2 3

(4) 6 , ([OM] Chorwacja 1998);


4 3
(a − b)2 a+b √ (a − b)2
(5) < − ab < , gdy a > b, ([OM] Czechosłowacja 1953/1954);
8a 2 8b
s s
a+b an+1 − bn+1 n an + bn
(6) 6 n
6 , gdy a > b, ([MOc] 1997/1998 z.3).
2 (a − b)(n + 1) 2
√ √
3

5
8.2.2. 2 2 a + 3 b > 5 ab, dla a, b > 0. ([Kw] 10/1986 34).
q p
8.2.3. x2 + y 2 + 1 > x y 2 + 1 + y x2 + 1, dla x, y ∈ R.
([OM] Estonia 2000, [Crux] 2003 s.32, patrz 8.3.13).

1 1 2
8.2.4. √ +p 6√ , dla x, y ∈ (0, 1). ([OM] Rosja 2000, [MaOD] 31).
x2 + 1 y2 + 1 xy + 1

x y 2
8.2.5. p 2 +√ 6 √ , dla x, y ∈ [0, 1]. ([OM] St Petersburg 1995).
y +2 x +2
2 3
x y 2
8.2.6. p 2 +√ 6 √ , dla x, y ∈ [0, 1]. ([OM] St Petersburg 1995).
2y + 3 2x2 + 3 5

x y 2
8.2.7. p 2 +√ 6 , dla x, y ∈ [0, 1/2]. ([OM] St Petersburg 1995).
4y + 1 4x2 + 1 2
oooooooooooooooooooooooooooooooooooooooooooooooooooooooooooooooooooooo
8.3 Jednorodne nierówności z pierwiastkami trzech zmiennych
oooooooooooooooooooooooooooooooooooooooooooooooooooooooooooooooooooooo
1 1 1 3
8.3.1. √ + √ + √ >√ dla x, y, z > 0.
x x+y y y+z z z+x 2xyz
(Phan Thanh Nam, [OM] Wietnam 2005, [Pkh] s.182).

1 1 1 1 1 1
8.3.2. + + > √ + √ + √ , dla x, y, z > 0. ([Ko00], [MaOD] 8).
x y z xy yz zx
Nierówności 8. Nierówności z pierwiastkami 111

D. Z nierówności pomiędzy średnimi mamy:


1 1 2 1 1 2 1 1 2
+ >√ , + >√ , + >√ .
x y xy y z yz z x zx

Po dodaniu stronami i podzieleniu przez 2 otrzymujemy rozpatrywaną nierówność. ⊠

8.3.3. Jeśli x, y, z > 0, to:


1 1 1 6
(1) p +p 2 +p 2 > , (Pham Kim Hung, [Pkh] s.202);
+ yz
x2 y + zx z + xy x+y+z

1 1 1 2 2
(2) p +p 2 +p 2 > , (Pham Kim Hung, [Pkh] s.223);
x2 + yz y + zx z + xy xy + yz + zx
1 1 1 4
(3) p +p 2 +p 2 > , (Pham Kim Hung, [Pkh] s.239).
4x2 + yz 4y + zx 4z + xy x+y+z

8.3.4. Jeśli x, y, z > 0, to:


√ √ √
y+z z+x x+y 4(x + y + z)
(1) + + >p .
x y z (x + y)(y + z)(z + x)
(Darij Grinberg, [Mild]);
√ √ √ √ √ √ √
x+y+z+ x x+y+z+ y x+y+z+ z 9+3 3
(2) + + > √ .
y+z z+x x+y 2 x+y+z
([Crux] z.2743);
p p √
x3 + y 3 y3 + z3 z 3 + x3 6(xy + yz + zx)
(3) + + > p .
x +y
2 2 y +z
2 2 z +x
2 2 (x + y + z) (x + y)(y + z)(z + x)
([ME] 3/2009, [Mon] 9/2011 850).

x y z
8.3.5. p +p 2 +p 2 > 1, dla x, y, z > 0. ([IMO] 2001).
x2 + 8yz y + 8zx z + 8xy

Powyższa nierówność jest szczególnym przypadkiem następującej nierówności.

x y z 3
8.3.6. p 2 + p 2 + p 2 > √ , dla λ > 8 oraz x, y, z > 0.
x + λyz y + λzx z + λxy 1+λ
([Crux] 2002 s.308).

8.3.7. Jeśli x, y, z > 0, to:


s
x y z x2 + y 2 + z 2
(1) + + > , (Vo Quoc Ba Can, [Pkh] s.192);
y z x xy + yz + zx
r r r
x y z
(2) + + > 2, ([Kw] 6/2002 18);
y+z z+x x+y
r r r s
x y z xyz
(3) + + >2 1+ , ([ME] 12(5)(2008));
y+z z+x x+y (x + y)(y + z)(z + x)
112 Nierówności 8. Nierówności z pierwiastkami

r r r
x y z 3
(4) + + 6 √ , (V. Cirtoaje, S. Riasat, [Rias]);
x+y y+z z+x 2
r r r
x y z
(5) + + 6 1, ([Pkh] s.73);
4x + 4y + x 4y + 4z + x 4z + 4x + y
x y z
(6) p + √ + p > 2, ([OM] Rumunia 1993, [Crux] 1997 s.325);
y2 + z2 z 2 + x2 x2 + y 2

x y z 3 2
(7) 1< p 2 +p 2 +√ 6 , ([OM] Chiny 2004, [BiYel]);
x + y2 y + z2 z 2 + x2 2
x+y y+z z+x
(8) p +p 2 +p 2 > 4, (Pham Kim Hung, [Pkh] s.172);
z + xy
2 x + yz y + zx
x y z
(9) p 2 +√ +p 2 > 1, ([Pkh] s.72);
4y + yz + 4z 2 4z + zx + 4x
2 2 4x + xy + 4y 2
√ √ √
xy yz zx xy yz zx
(10) + + 6 2 + 2 + 2 , ([OM] Rosja 1996);
z x y z x y
x y z
(11) p 2 +p 2 +p 2 > 1, ([IMO] 2001, [Kw] 4/2002 18);
x + 8yz y + 8zx z + 8xy
x y z
(12) p + p + p 6 1,
x + (x + y)(x + z) y + (y + z)(y + x) z + (z + x)(z + y)
([MOc] 1997/1998 z.47);
s s s
x2 + 2yz y 2 + 2zx z 2 + 2xy
(13) + + > 3, ([Pkh] s.173);
y2 + z2 z 2 + x2 x2 + y 2
r r r
xy yz zx
(14) + + 6 1,
4x2 + y 2 + 4z 2 4y 2 + z 2 + 4x2 4z 2 + x2 + 4y 2
((Pham Kim Hung, [Pkh] s.230).

√ √ √ √ √
8.3.8. x+y+ y+z+ z+x6 6· x + y + z, dla x, y, z > 0.

D. ([Stee] 13, 227).


√ √ √ √ √ √
x+y+ y+z+ z+x x+y y+z z+x
√ = √ ·1+ √ ·1+ √ ·1
x+y+z x+y+z x+y+z x+y+z
s 
x+y y+z z+x
6 + + (12 + 12 + 12 )
x+y+z x+y+z x+y+z
√ √
= 3·2= 6.

Wykorzystaliśmy nierówność Cauchy’ego 2.6.1. ⊠

8.3.9. Jeśli x, y, z > 0, to:


x y z 5√
(1) √ +√ +√ 6 x + y + z, ([Crux] 1/1991 z.1490);
x+y y+z z+x 4
Nierówności 8. Nierówności z pierwiastkami 113

√ √ √
x y z x+ y+ z
(2) √ +√ +√ > √ , ([Pkh] s.185);
x+y y+z z+x 2
r
x y z 3
(3) √ +√ +√ > (x + y + z), ([OM] Serbia i Czarnogóra 2005);
y+z z+x x+y 2
s s s √ √ √
x3 y3 z3 x+ y+ z
(4) + + > √ ,
x + xy + y 2
2 y + yz + z 2
2 z + zx + x2
2
3
(Le Trung Kien, [Pkh] s.217).

√ p √ √ √ √
8.3.10. 2x + 2y + 2z 6 x+y+ y+z+ z + x, dla x, y, z > 0.
√ p √ √ √ √
2x + 2y + 2z 6 x+y+ y+z+ z + x, dla x, y, z > 0.
n n n n n n
8.3.11.

D. Wynika z nierówności 1.5.5 zastosowanej dla wypukłej funkcji f : (0, ∞) → R, f (x) = − n x.
Patrz 1.5.7. ⊠

8.3.12. Jeśli x, y, z > 0, to:


q p q
(1) x2 + xy + y 2 + x2 + xz + z 2 > y 2 + yz + z 2 , ([MaS] 4/1985 z.2769);
q q p √
(2) x2 + xy + y 2 + y 2 + yz + z 2 + z 2 + zx + x2 > 3 xy + yz + zx, ([Crux] 1990 s.46);
q q q q
(3) 2(x2 + y 2 ) + 2(y 2 + z 2 ) + 2(z 2 + x2 ) > 2(x + y)2 + 3(y + z)2 + 3(z + x)2 ,
([OM] Polska 2003/2004);
q q q
(4) x2 + 8yz + y 2 + 8zx + z 2 + 8xy 6 3(x + y + z), ([Pkh] s.60);
q q q
(5) 3(x + y + z) > x2 + yz + y 2 + zx + z 2 + xy, ((Pham Kim Hung, [Pkh] s.153);
s
 
1 yz zx xy x2 + y 2 + z 2 x+y+z
(6) + + > > , ([OM] Irlandia 2007, [MaOD] 43);
3 x y z 3 3
s s s
x3 y3 z3 x+y+z
(7) + + > √ , ([Zw] 2002);
x+y y+z z+x 2
xy + yz + zx x+y+z
(8) p p √ 6 √ ,
x + xy + y + y + yz + z + z + zx + x
2 2 2 2 2 2 3 3
(M. S. Klamkin, [Crux] z.805).

q p
8.3.13. x2 + y 2 + z 2 > x y 2 + z 2 + y x2 + z 2 , dla x, y, z ∈ R. ([Crux] 2003 s.33).

D. Z nierówności pomiędzy średnimi mamy:


x2 + (y 2 + z 2 ) p y 2 + (x2 + z 2 ) p
> x y2 + z2 , > y x2 + z 2 .
2 2
Po dodaniu stronami otrzymujemy rozpatrywaną nierówność. ⊠
114 Nierówności 8. Nierówności z pierwiastkami

√ √ √
8.3.14. x(x − y)(x − z) + y(y − z)(y − x) + z(z − x)(z − y) > 0, dla x, y, z > 0.
(Nierówność Schura, 4.5.5).

√ √ √
8.3.15. xy + yz + zx > x yz + y xz + z xy, dla x, y, z > 0. ([Ko00], [MaOD] 8).

D. Z nierówności pomiędzy średnimi mamy:


√ √ √
xy + zx > 2x yz, yz + xy > 2y zx, zx + yz > 2z xy.

Dodajemy te nierówności do siebie stronami i po podzieleniu przez 2 otrzymujemy rozpatrywaną


nierówność. ⊠

p p p
8.3.16. Niech a, b, c > 0, x = b2 − bc + c2 , y = c2 − ca + a2 , z = a2 − ab + b2 .
Wtedy xy + yz + zx > a2 + b2 + c2 . (Nguyen Anh Tuan, [Pkh] s.37).
p p p
8.3.17. Niech a, b, c > 0, x = b2 + bc + c2 , y = c2 + ca + a2 , z = a2 + ab + b2 .
Wtedy xy + yz + zx > (a + b + c)2 . ([Pkh] s.37).

8.3.18. Jeśli x, y, z > 0, to:


q
(1) x2 + y 2 + z 2 + 3 3 x2 y 2 z 2 > 2(xy + yz + zx), ([MaOD] 32);

1 √ √ √
(2) (x + y + z)2 > x yz + y xz + z xy, ([OM] ZSRR 1991);
3
q
(3) (x + y)(x + z) > 2 xyz(x + y + z), ([MaOD] 41);

√ √ √ √  q
(4) xyz x+ y+ z + (x + y + z)2 > 4 3xyz(x + y + z), ([ME] 11(3)(2006));
s
x2 + y 2 + z 2 + xy + yz + zx x+y+z x2 + y 2 + z 2
(5) 6 · , ([OM] Węgry-Izrael 2009);
6 3 3
x3 y3 z3
(6) p + √ + p > x2 + y 2 + z 2 , ([Pkh] s.121).
y 2 − yz + z 2 z 2 − zx + x2 x2 − xy + y 2

q
8.3.19. x2 (y + z) + y 2 (z + x) + z 2 (x + y) > (xy + yz + zx) 3 (x + y)(y + z)(z + x), dla
x, y, z > 0 ([Pkh] s.50).
q q p 2x2 2y 2 2z 2
4x3 + 4y 3 + 4y 3 + 4z 3 + 4z 3 + 4x3 6 + + , dla x, y, z > 0.
3 3 3
8.3.20.
x+y y+z z+x
(Thomas J. Mildorf, [Mild]).
q q q
8.3.21. 3
2x2 − yz + 3
2y 2 − zx + 3
2z 2 − xy > 0 dla x, y, z > 0. ([Pkh] s.185).

s 2 s 2 s 2
2x 2y 2z
+ + > 3, dla x, y, z > 0. ([Mild]).
3 3 3
8.3.22.
y+z z+x x+6
q q q
8.3.23. 3
x3 + 7xyz + 3
y 3 + 7xyz + 3
z 3 + 7xyz 6 2(x + y + z), dla x, y, z > 0.
Nierówności 8. Nierówności z pierwiastkami 115

Powyższa nierówność jest szczególnym przypadkiem następującej nierówności.

8.3.24. Jeśli λ > 2 jest liczbą rzeczywistą, to


s s s
x3 + λxyz y 3 + λxyz z 3 + λxyz
+ + 6 x + y + z,
3 3 3

1+λ 1+λ 1+λ

dla x, y, z > 0. (Vasile Cirtoaje, [Mild]).

s
x2 y 2 z 2 3 4 x4 + y 4 + z 4
8.3.25. + + > dla x, y, z > 0. ([Pkh] s.204).
y x x 2 3
s
x2 y2 z2 4 x + y + z
4 4 4
8.3.26. + + >3 dla x, y, z > 0. ([Pkh] s.205).
y+z z+x x+y 3

⋆ O. Mushkarov, N. Nikolov, Some generalizations of an inequality from IMO 2001, [Crux] 2002
308-312.
oooooooooooooooooooooooooooooooooooooooooooooooooooooooooooooooooooooo
8.4 Niejednorodne nierówności z pierwiastkami trzech zmiennych
oooooooooooooooooooooooooooooooooooooooooooooooooooooooooooooooooooooo
p p p √
3 2
8.4.1. x2 + (1 − y)2 + y 2 + (1 − z)2 + z 2 + (1 − x)2 > 2 , x, y, z ∈ R. ([KoM] 1998(9)).

√ p √ p
8.4.2. x2 + 1 + y2 + 1 + z2 + 1 > 6(x + y + z), dla x, y, z > 0. (8.1.3, [Crux] 1996 s.135).

r q
2 √ √
x+ y+ xyz, dla x, y, z ∈ [0, ∞). ([Fom] 26/93).
3 4
8.4.3. z> 32

8.4.4. Jeśli a, b, c > 0 i a + b + c = 1, to:


√ √ √ √
(1) a + b + b + c + c + a 6 6, (patrz 8.3.8);
p p p
(2) a + b2 + b + c2 +
c + a2 > 2, (Phan Thanh Nam, [Pkh] s.72);
√ √ √
(3) 4a + 1 + 4b + 1 + 4c + 1 < 5, ([MaOD] 20);
√ √ √ √
(4) 4a + 1 + 4b + 1 + 4c + 1 6 21, ([MaOD] 20);
√ √ √ 1
(5) a b + b c + c a 6 √ , ([OM] Bośnia Hercegowina 2005);
3
√ √ √
(6) ab + c + bc + a + ca + b 6 2; ([OM] Meksyk 2007, [MaOD] 39);

(7) a2 + b2 + c2 + 2 3abc 6 1, ([OM] Polska 1999/2000);

√ √ √ 2 2
(8) a 1 − bc + b 1 − ca + c 1 − ab > , ([ME] 12(5)(2008));
3
q q q √
(9) a + (b − c)2 + b + (c − a)2 + c + (a − b)2 > 3, ([ME] 12(5)(2008));
116 Nierówności 8. Nierówności z pierwiastkami

r s r r
a b c 3
(10) + + 63 , (V. N. Murty, [Crux] z.3051);
a + bc b + ca c + ab 2
ab bc ca 1
(11) √ +√ +√ 6√ , ([OM] Chiny 2006, [Pkh] s.156):
ab + bc bc + ca ca + ab 2
s s r
ab bc ca 3
(12) + + 6 , ([Crux] z.2883);
c + ab a + bc b + ca 2
√ √
abc 3 3
(13) 6 , (V. N. Murty, [Crux] z.3051);
(1 − a)(1 − b)(1 − c) 8
a b c
(14) √ +√ +√ > 1, ([Pkh] s.50).
3
a + 2b 3
b + 2c 3
c + 2a
q
8.4.5. ax + by + cz + 2 (ab + bc + ca)(xy + yz + zx) 6 a + b + c, dla a, b, c, x, y, z > 0,
x + y + z = 1. ([OM] Ukraina 2001).

8.4.6. Jeśli a, b, c > 0 i a + b + c = 2, to:


√ √ √
(1) a + b − 2ab + b + c − 2bc + c + a − 2ca > 2, (Pham Kim Hung [Pkh] s.35);
s s r
a+b b+c c+a √
(2) − ab + − bc + − ca > 2, ([ME] 12(5)(2008));
2 2 2
a b c
(3) √ +√ +√ > 1, ([Pkh] s.199).
3+b +c
2 2 3+c +a
2 2 3 + a2 + b2
8.4.7. Niech a, b, c > 0, a + b + c = 3. Wtedy:
√ √ √
(1) a + b + c > ab + bc + ca, ([OM] Rosja 2002, [Pkh] s.17);
p p p
(2) a 1 + b3 + b 1 + c3 + c 1 + a3 6 5, ([Pkh] s.169);
r s r
a b c √
(3) + + > 3, (Phan Thanh Viet, [Pkh] s.148);
1 + 2bc 1 + 2ca 1 + 2ab
q p q p q p q√
(4) a+ b2 + c2 + b+ c2 + a2 + c+ a2 + b2 >3 2 + 1,
(Phan Hong Son, [Pkh] s.39).

8.4.8. Niech x, y, z > 0, xyz = 1. Wtedy:


x+y y+z z+x √ √ √
(1) √ + √ + √ > x + y + z, ([MG]);
z x y
x y z
(2) √ +√ +√ > 1, ([Pkh] s.48);
7+y+z 7+z+x 7+x+y
x y z
(3) p +√ +p > 1, ([Pkh] s.48);
7+y +z
2 2 7+z +x
2 2 7 + x2 + y 2
s s s
x+y y+z z+x
(4) + + > 3, ([Pkh] s.22).
x+1 y+1 z+1
Nierówności 8. Nierówności z pierwiastkami 117

√ √ √
8.4.9. x3 + y 3 + z 3 > x y + z + y z + x + z x + y, gdy x, y, z > 0 oraz xyz = 2.

x2 y2 z2 4
8.4.10. p + p + p > , dla x, y, z > 0,
(1 + x )(1 + y )
3 3 (1 + y )(1 + z )
3 3 (1 + z )(1 + x )
3 3 3
xyz = 8. ([A-P] 2005).

√ q
8.4.11. xyz + (1 − x)(1 − y)(1 − z) < 1, dla x, y, z ∈ (0, 1). ([OM] Rumunia 2002).

x y z q
8.4.12. √ +√ +√ 6 2(x + y + z), dla x, y, z ∈ [0, 1]. ([OM] Rosja 1998).
1 + yz 1 + zx 1 + xy

r   r   q
x+y 2 y+z 2 
z+x 2

8.4.13. 1− 2 + 1− 2 + 1− 2 > 6, gdy x, y, z > 0 i x2 +y 2 +z 2 = 1.
([ME] 12(5)(2008)).
q q q √
x y z
8.4.14. x2 +y 2 +1 + y 2 +z 2 +1 + z 2 +x2 +1 6 3, gdy x, y, z > 0 i x2 + y 2 + z 2 = 3.
(Pham Kim Hung [Pkh] s.35).

8.4.15. Jeśli x, y, z > 0 i xy + yz + zx = 1, to:



1 1 1 2
(1) √ +√ +√ >2+ , (Le Trung Kien, [Pkh] s.179);
x+y y+z z+x 2
s √
1 3
3
(2) 3 3
+ 6(x + y + z) 6 , ([OM] Słowenia 2005).
xyz xyz

8.4.16. Jeśli x, y, z są bokami trójkąta, to:


√ √ √ √ √ √
(1) x + y − z + y + z − x + z + x − y 6 x + y + z, ([A-P] 1996, [Khr2], [Khr1]);
√ √ √ √ √ √
(2) n x + y − z + n y + z − x + n z + x − y 6 n x + n y + n z, ([Khr2], [Khr1]).

D. Wynika z nierówności 1.5.5 zastosowanej dla wypukłej funkcji f : (0, ∞) → R, f (x) = − n x.
Patrz 1.5.8. ⊠

8.4.17 ([Mild]). Jeśli x, y, z są bokami trójkąta, to:


x y z √
p + p + p > 3.
2y 2 + 2z 2 − x2 2z 2 + 2x2 − y 2 2x2 + 2y 2 − z 2

8.4.18 ([Pkh] s.19). Jeśli x, y, z są bokami trójkąta o obwodzie równym 3, to:


1 1 1 9
√ +√ +√ > .
x+y−z y+z−x z+x−y xy + yz + zx
118 Nierówności 8. Nierówności z pierwiastkami

oooooooooooooooooooooooooooooooooooooooooooooooooooooooooooooooooooooo
8.5 Nierówności z pierwiastkami czterech zmiennych
oooooooooooooooooooooooooooooooooooooooooooooooooooooooooooooooooooooo
√ √ √ √ √ √
8.5.1. x + y + z + y + z + t + z + t + x + t + x + y 6 12 · x + y + z + t, dla
x, y, z, t > 0.
√ √ √ √ √ √ √ √
x+y+z+ y+z+t+ z+t+x+ t+x+y √ x+y+z √ y+z+t √ z+t+x t+x+y
D. √
x+y+z+t
= x+y+z+t
·1+ x+y+z+t
·1+ x+y+z+t
· 1 √x+y+z+t ·1
r 
x+y+z y+z+t z+t+x t+x+y
√ √
6 x+y+z+t + x+y+z+t + x+y+z+t + x+y+z+t (12 + 12 + 12 + 12 ) = 3·4= 12.
Wykorzystaliśmy nierówność Cauchy’ego 2.6.1. ⊠

r r r s
x y z t
8.5.2. + + + > 2, dla x, y, z, t > 0. ([MOc] z.462).
y+z z+t t+x x+y
q q
1+ y+z
D. ([MOc]). Z nierówności pomiędzy średnimi mamy: y+z x 6 2
x
= x+y+z
2x . Stąd
x
y+z >
q q q q
2x 2x x y z t 2(x+y+z+t)
x+y+z > x+y+z+t . Zatem: y+z + z+t + t+x + x+y > x+y+z+t = 2. ⊠

8.5.3. Dla dodatnich liczb rzeczywistych x, y, z, t zachodzą następujące nierówności.


(xy + zt)(xt + yz) √
(1) > xyzt, ([OM] St Petersburg 2004);
(x + z)(y + t)
s s s s
x3 y3 z3 t3 x+y+z+t
(2) + + + > √ , ([Zw] 2003);
y+z z+t t+x x+y 2
r r
xy + xz + xt + yz + yt + zt xyz + yzt + ztx + txy
(3) > 3 , ([Pkh] s.108);
6 4
x2 y 2 z 2 t2 √ q
(4) + + + > 2 2 4 x4 + y 4 + z 4 + t4 , ([Pkh] s.205).
y z t x
s s s s
x+1 y+1 z+1 t+1
8.5.4. + + + > 4, dla x, y, z, t > 0, x + y + z + t = 4.
xy + 1 yz + 1 zt + 1 tx + 1
(Pham Kim Hung, [Pkh] s.184).

oooooooooooooooooooooooooooooooooooooooooooooooooooooooooooooooooooooo
8.6 Nierówności z pierwiastkami i liczbami naturalnymi
oooooooooooooooooooooooooooooooooooooooooooooooooooooooooooooooooooooo

1 1 · 3 · · · · · (2n − 1) 3 1
8.6.1. √ < < · √ . ([Kw] 2/1971 27 M23).
2n 2 · 4 · · · · · 2n 2 2n
1 1 3 5 2n − 1 1
8.6.2. √ < · · ··· 6√ . ([Kw] 8/1978 47, [G-if] 102).
4n + 1 2 4 6 2n 2n + 1
1 3 5 2n − 1 1
8.6.3. · · ··· 6√ . ([Crux] 1998 s.167).
2 4 6 2n 3n + 1
Nierówności 8. Nierówności z pierwiastkami 119

√ √ 1 √ √
8.6.4. n+1− n < √ < n − n − 1, dla n ∈ N. ([OM] Słowenia 2001).
2 n
2 √ √ √ √ 4n + 3 √
8.6.5. n n < 1 + 2 + ··· + n < n, dla n ∈ N. ([Putn] 1948).
3 6

√ 1 1 1
8.6.6. n < 1 + √ + √ + ··· + √ . ([KoMe] z.21).
2 3 n
√ √ 1
D. ([KoMe]). Wynika to z oczywistej nierównośći: k+1− k<√ .⊠
k+1

√ 1 1 1 √
8.6.7. 2( n + 1 − 1) < 1 + √ + √ + · · · + √ < 2 n. ([G-if] 205, [Str67] 42).
2 3 n
√ √ 1 1 1 √
8.6.8. 2 n + 1 − 1 2 + 1 < 1 + √ + √ + · · · + √ < 2 n − 1, dla n > 2. ([Siw] 52).
2 3 n

1 1 1 1
8.6.9. + √ + √ + ··· + √ < 2. ([Kw] 10/1983 47).
2 3 2 4 3 (n + 1) n

X 1
8.6.10. √ < p, dla p > 1. ([Bryn] 4.11).
k=1 (k + 1) k
p

8.6.11. Jeśli x1 < x2 < · · · < xn są liczbami naturalnymi, to


√ √ √
x2 − x1 x3 − x2 xn − xn−1 1 1 1 1
+ + ··· + < + + + ··· + 2. ([Kw] 3/1972 40).
x2 x3 xn 1 2 3 n

√ √
7 + 3 < 4. ([Mock] 2/2001).
3 3
8.6.12.
√ √
8.6.13. 11 10 > 10 + 10. ([OM] St Petersburg 1995).
1993 1000


8.6.14. 2n > 1 + n 2n−1 , dla n = 2, 3, · · · . ([G-if] 102).
m √ √ m + 2n
8.6.15. < 2 ⇐⇒ 2 < , dla m, n ∈ N. ([MaOD] 4).
n m+n
1 1 1
8.6.16. √ √ +√ √ + ··· + √ √ > 24. ([Fom] 56/86).
1+ 3 5+ 7 9997 + 9999

1 1
8.6.17. √ + √ > 1, dla m, n ∈ N. ([Fom] 23/72, [Br80] 56).
m
n+1 n
m+1
√ m  m n √ n
D. n
m+161+ , gdyż m + 1 6 1 + . Analogicznie m n + 1 6 1 + . Zatem:
n n m
1 1 1 1 n m
√ + √ > + = + = 1. ⊠
m
n+1 n
m+1 1 + m/n 1 + n/m) n+m n+m
120 Nierówności 8. Nierówności z pierwiastkami

√ √ √
8.6.18. min( n m, m n) < 3, gdy m, n ∈ N, m 6= n. ([Mon] 77(7)(1970) E2190, [B-zm] 44).
3

√ √
8.6.19. n+1
n+1< n
n, dla n > 2. ([Str67] 41, [B-rs] 154).
√  √ √ √ n
n+1
8.6.20. n > n+1 , dla n > 8. ([Putn] 1940, [Mock] 3/2004).
r
n2 + n n n2 + 2n
8.6.21. 2 < n+1
< 2 . ([Cmj] 19(1)(1988) 82-84).
n +n+1 n+1 n + 2n + 1
n
n X √
X 1
8.6.22. n 6 p 6 n ln(1 + 2), dla n > 3. ([Bryn] 4.15, s.113).
i=1 j=1 i2 + j 2
r q √
√ 1+ 5
8.6.23. 1+ 1 + ··· + 1< . ([Pie2]).
2
r q √ 5
8.6.24. 2+ 2 + ··· + 2< . (Wynika z 8.6.26).
2
r q √
8.6.25. 3+ 3 + ··· + 3 < 3. (Wynika z 8.6.26).
r q √ √
8.6.26. a+ a + ··· + a< a + 1, dla a > 0. ([Fich], [Bedn] 124).
r q √
8.6.27. 1+ 2 + ··· + n < 2. ([Mat] 5-6/1967 273, [Pie1], [Pie2]).
s r q √
8.6.28. 5+ 3+ 5+ 3 + . . . < 3. ([Pie1], [Pie2]).

oooooooooooooooooooooooooooooooooooooooooooooooooooooooooooooooooooooo
8.7 Różne nierówności z pierwiastkami
oooooooooooooooooooooooooooooooooooooooooooooooooooooooooooooooooooooo
8.7.1. Wśród czterech różnych liczb rzeczywistych istnieją dwie takie liczby a i b, że
1 + ab 1
√ √ > . ([KoM] 2000(3) B3358).
1+a · 1+b
2 2 2

8.7.2. Jeśli (an ) jest ciągiem arytmetycznym o wyrazach dodatnich i różnicy r > 0, to dla
wszystkich n ∈ N zachodzą następujące nierówności.
2 √ √ 1 1 1
(1) ( an+1 − a1 ) < √ + √ + · · · + √ ,
r a1 a2 an
√ √ √ 2 √ √
(2) a1 + a2 + · · · + an < (an+1 an+1 − a1 a1 ). ([Bedn] 103).
3r
8.7.3. Ciąg (an ) określamy następująco:
p
1 + a2n − 1
a0 = 1, an+1 = , n ∈ N.
an
π
Wykazać, że an > . ([OM] Kanada-USA 2000).
2n+2
⊡⊡⊡⊡⊡⊡⊡⊡⊡⊡⊡⊡⊡⊡⊡⊡⊡⊡⊡⊡⊡⊡⊡⊡⊡⊡⊡⊡⊡⊡⊡⊡⊡⊡⊡⊡⊡⊡⊡⊡⊡⊡⊡⊡⊡⊡⊡⊡⊡⊡⊡⊡

9 Różne nierówności
⊡⊡⊡⊡⊡⊡⊡⊡⊡⊡⊡⊡⊡⊡⊡⊡⊡⊡⊡⊡⊡⊡⊡⊡⊡⊡⊡⊡⊡⊡⊡⊡⊡⊡⊡⊡⊡⊡⊡⊡⊡⊡⊡⊡⊡⊡⊡⊡⊡⊡⊡⊡

oooooooooooooooooooooooooooooooooooooooooooooooooooooooooooooooooooooo
9.1 Pochodna i nierówności
oooooooooooooooooooooooooooooooooooooooooooooooooooooooooooooooooooooo
Przypomnijmy, że przez R+ oznaczamy zbiór wszystkich nieujemnych liczb rzeczywistych.
Jeśli f jest różniczkowalną funkcją n zmiennych x1 , . . . , xn , to przez f¯ oznaczamy sumę
wszystkich pochodnych cząstkowych funkcji f , tzn.

∂f ∂f ∂f
f¯ = + + ··· + .
∂x1 ∂x2 ∂xn

9.1.1. Niech f będzie różniczkowalną funkcją taką, że f (0) > 0 oraz f ′ (x) > 0 dla x ∈ R+ .
Wtedy f (x) > 0 dla x ∈ R+ .

9.1.2. Niech f będzie różniczkowalną funkcją n zmiennych. Załóżmy, że funkcja ta spełnia


następujące dwa warunki.
(1) Dla każdego i ∈ {1, 2, . . . , n} zachodzi nierówność

f (x1 , . . . , xi−1 , 0, xi+1 , . . . , xn ) > 0,

dla wszystkich x1 , . . . , xi−1 , xi+1 , . . . , xn ∈ R+ .


(2) f¯(x1 , . . . , xn ) > 0, dla wszystkich x1 , . . . , xn ∈ R+ .
Wtedy dla wszystkich x1 , . . . , xn ∈ R+ , zachodzi nierówność
 
f x1 , . . . , xn > 0.

([Dlt] 10(2004)).

D. ([Dlt] 10(2004)). Niech x1 , . . . , xn ∈ R+ . Oznaczmy przez a najmniejszą z liczb x1 , . . . , xn i


niech y1 = x1 − a, y2 = x2 − a, . . . , yn = xn − a. Wtedy liczby y1 , . . . , yn są nieujemne i co najmniej
jedna z nich jest równa zero. Zatem, na mocy warunku (1), mamy nierówność f (y1 , . . . , yn ) > 0.
Rozważmy funkcję jednej zmiennej t określoną wzorem
 
g(t) = f y1 + t, y2 + t, . . . , yn + t .

Jest to funkcja różniczkowalna i g(0) = f (y1 , . . . , yn ) > 0. Z warunku (2) wynika, że jeśli t ∈ R+ , to

g ′ (t) = f¯(y1 + t, y2 + t, . . . , yn + t) > 0.

Funkcja g spełnia więc założenia stwierdzenia 9.1.1. Zatem g(t) > 0, dla wszystkich t ∈ R+ i wobec
tego f (x1 , . . . , xn ) = f (y1 + a, . . . , yn + a) = g(a) > 0. ⊠

121
122 Nierówności 9. Różne nierówności

9.1.3. Przykłady nierówności, zachodzących dla nieujemnych liczb rzeczywistych, które łatwo
można wykazać przy pomocy twierdzenia 9.1.2.
(1) a3 + b3 + c3 > a2 b + b2 c + c2 a.
(2) a3 + b3 + c3 + 3abc > ab(a + b) + bc(b + c) + ca(c + a). ([Dlt] 10(2004)).

D. (1). Niech f (a, b, c) = a3 + b3 + c3 − a2 b − b2 c − c2 a. Wtedy f (0, b, c) > 0, f (a, 0, c) > 0,


f (a, b, 0) > 0 oraz
f¯(a, b, c) = (a − b)2 + (b − c)2 + (c − a)2 > 0.
Teza wynika więc z 9.1.2. ⊠

⋆ L. Kurlandczyk, Suma pochodnych, [Dlt] 10(2004), 8-10.


Pham Kim Hung, Derivative and applications, [Pkh] 107-116.
oooooooooooooooooooooooooooooooooooooooooooooooooooooooooooooooooooooo
9.2 Nierówności z max i min
oooooooooooooooooooooooooooooooooooooooooooooooooooooooooooooooooooooo
9.2.1. min(a, b) + max(a, b) = a + b.
   
9.2.2. min max(a, b), c = max min(a, c), min(b, c) .
   
9.2.3. max min(a, b), c = min max(a, c), max(b, c) .
   
9.2.4. min max(a, c), max(b, d) > max min(a, b), min (c, d) . ([Mat] 4/1949 59).

   
9.2.5. max a2 −b, b2 −c, c2 −a > max a2 −a, b2 −b, c2 −c , dla a, b, c > 0. ([Fom] 21/91).
   
9.2.6. max a2 − b, b2 − c, c2 − d, d2 − a > max a2 − a, b2 − b, c2 − c, d2 − d , dla
a, b, c, d > 0. ([Fom] 28/91).

 
(a − 1)2 + b(2a − b) a b
9.2.7. > min , , dla a, b > 0, a + b < 1. ([ME] 12(5)(2008)).
(b − 1) + a(2b − a)
2 b a

9.2.8. Niech a, b, c, d > 0. Jeśli c + d 6 min(a, b), to

ad + bc 6 ab.

([Mon] 63(6)(1958) E1291).

D. ad + bc 6 (c + d) max(a, b) 6 min(a, b) max(a, b) = ab. ⊠

max(a, b, c) min(a, b, c) a+b+c


9.2.9. + > √ − 1. (S. Riasat, [Rias]).
min(a, b, c) max(a, b, c) 3
abc
   
a b c b c a 1 1 1
9.2.10. 3 max + + , + + > (a + b + c) + + , dla a, b, c > 0.
b c a a b c a b c
(M. S. Klamkin, [Crux] z.2064).
Nierówności 9. Różne nierówności 123

s
3 a3 + b3 + c3 + 3abc
9.2.11. > max(a, b, c), dla boków trójkąta. ([MaOD] 34).
2

9.2.12.
  1
(1) min (a − b)2 , (b − c)2 , (c − a)2 6 (a2 + b2 + c2 ), dla a, b, c ∈ R.
2
([Mon] 76(6)(1969) E2032, [OM] Polska 1985).

(2) Jeśli a1 , . . . , an ∈ R, n > 2, to

min (ai − aj )2 = A · (a21 + · · · + a2n ),


ai 6=aj

12
gdzie A = . ([Mon] 76(6)(1969) E2032).
n(n2 − 1)

9.2.13. 3 min(a, b, c) 6 1 + abc dla a, b, c ∈ R, a2 + b2 + c2 = 9. (V.Nicula, [Pkh] s.189).

9.2.14. 5 min(a, b, c) 6 1 + 4abc dla a, b, c > 0, a2 + b2 + c2 = 3. ([Pkh] s.190).

 
3 3 3 3 1 1 1 1
9.2.15. a +b +c +d > max a + b + c + d, + + + , gdy a, b, c, d > 0 i abcd = 1.
a b c d
([OM] Iran 1997).
!
b3 c3 c3 d3 d3 a3 a3 b3 bc cd da ab a2 b2 c2 d2
9.2.16. + + + > max + + + , + + + ,
a6 b6 c6 d6 a2 b2 c2 d2 bc cd da ab
dla a, b, c, d > 0. ([Cmj] 19(5)(1988) 450).

1
9.2.17. max(a1 + a2 + a3 , a2 + a3 + a4 , a3 + a4 + a5 , a4 + a5 + a6 , a5 + a6 + a7 ) > , dla
3
a1 , a2 , . . . , a7 > 0, a1 + · · · + a7 = 1. ([IMO] Shortlist 1981, [Djmp] s.144(444)).

9.2.18. Niech k, n ∈ N, k 6 n, a1 , . . . , an 6 0, a1 + · · · + an = 1. Wtedy


1
max (ai + ai+1 + · · · + ai+k−1 ) > ,
i=1,...,n−k+1 r

gdzie r jest taką liczbą naturalną, że k(r − 1) < n 6 kr. ([Djmp] s.444).

  |x1 | + · · · + |xn |
9.2.19. max x1 , . . . , xn , −x1 , −x2 , . . . , −xn > , dla x1 , . . . , xn ∈ R.
2n − 1
([Kw] 4/2005).

9.2.20. Jeśli a1 , . . . , an (n > 3) są liczbami rzeczywistymi takimi, że a1 + · · · + an > n oraz


a21 + · · · + a2n > n2 , to
max(a1 , . . . , an ) > 2.
([OM] USA 1999).
124 Nierówności 9. Różne nierówności

oooooooooooooooooooooooooooooooooooooooooooooooooooooooooooooooooooooo
9.3 Nierówności z bezwzględną wartością
oooooooooooooooooooooooooooooooooooooooooooooooooooooooooooooooooooooo

9.3.1. |a| − |b| 6 |a − b|, dla a, b ∈ R.

9.3.2. Dla dowolnych liczb rzeczywistych a, b, c zachodzą następujące nierówności.


(1) |a + b − c| + |a − b + c| + | − a + b + c| > |a| + |b| + |c|, ([OM] Moskwa 1995, [Mat] 1/1996 s.45).

(2) |a| + |b| + |c| + |a + b + c| > |a + b| + |b + c| + |c + a|, ([MaOD] 3).

9.3.3. Niech a, b, c ∈ R. Jeśli |a| > |b + c|, |b| > |c + a| oraz |c| > |a + b|, to

a + b + c = 0.

([Balt] 1996).

9.3.4. Niech a, b, c ∈ R. Jeśli |a − b| > |c|, |b − c| > |a| oraz |c − a| > |b|, to jedna z liczb
a, b, c jest sumą dwóch pozostałych. ([OM] Moskwa 1996).

x3 + y 3 + z 3 3
9.3.5. > xyz + (x − y)(y − z)(z − x) , dla x, y, z > 0.
3 4
([OM] Rumunia 2007, [MaOD] 34).

9.3.6. Jeśli a1 , . . . , an , b1 , . . . , bn są liczbami rzeczywistymi, to


X X
(|ai − aj | + |bi − bj |) 6 |ai − bj |. ([OM] Polska 1998/1999).
i<j i,j

9.3.7. Jeśli a, b, c są liczbami zespolonymi takimi, że |a| = |b| = |c| oraz liczby a2 , b2 , c2 są
parami różne, to
ab bc ca √
+ + > 3. ([Crux] 2004 s.319 z.2863).
a2 − b2 b2 − c2 c2 − a2
oooooooooooooooooooooooooooooooooooooooooooooooooooooooooooooooooooooo
9.4 Nierówności z częścią całkowitą
oooooooooooooooooooooooooooooooooooooooooooooooooooooooooooooooooooooo
Przedstawiamy nierówności z częścią całkowitą liczby rzeczywistej, które pojawiły się w
[N10]. We wszystkich poniższych nierównościach zakładamy, że: x, y, z ∈ R, a, b, c ∈ Z oraz
n, m ∈ N.

9.4.1. [x] + [y] 6 [x + y] 6 [x] + [y] + 1.

9.4.2. [x][y] 6 [xy], dla x > 0, y > 0.

9.4.3. [2x + 2y] > [x + y] + [x] + [y]. ([S50] 135, [B-zm] 36).
Nierówności 9. Różne nierówności 125

9.4.4. [2x] + [2y] > [x + y] + [x] + [y]. ([S50] 135, [B-zm] 36, [N10]).

9.4.5. [3x] + [3y] > 2[x + y] + [x] + [y]. ([KoMe] 413b).

9.4.6. [3x] + [3y] > 2[x] + 2[y] + [x + y].

9.4.7. [(n + 1)x] + [(n + 1)y] > n[x] + n[y] + [x + y]. ([N10]).

9.4.8. [3x] + [3y] + [3z] > 2[x] + 2[y] + 2[z] + [x + y + z]. ([Pa94] 45, [N10]).

9.4.9. [4x] + [4y] > 3[x + y] + [x] + [y]. ([KoMe] 413c).

9.4.10. [4x] + [4y] > 2[x + y] + 2[x] + 2[y]. ([KoMe] 413d).

9.4.11. [4x] + [4y] > [x + y] + 3[x] + 3[y]. ([KoMe] 413e).

9.4.12. [5x] + [5y] > [3x + y] + [x + 3y] dla x, y > 0. ([OM] USA 1975, [Mat] 1/1979 53).

9.4.13. n[x] 6 [nx] 6 n[x] + n − 1. ([DoC] 287).

[x] [2x] [nx]


9.4.14. [nx] > + + ··· + , dla 0 < x ∈ R. ([OM] USA 1981, [KoM] 2003, A311).
1 2 n
 
1
9.4.15. (k1 + k2 + · · · + kn ) + n − 1 6 k1 + k2 + · · · + kn dla k1 , . . . , kn ∈ Z. ([DoC] 288).
n
n
X √
9.4.16. (−1)[k 2]
6 1 + 2 log2 n. ([Dlt] 8/1995).
k=1

9.4.17. Jeśli x, y, z > 0 i x + y + z = 11, to:


(1) [x]4 + [y]4 + [z]4 > 243,
(2) x[x] + y [y] + z [z] > 81. ([OM] Rosja 2003).

9.4.18. Niech 0 < x ∈ R.


(1) Jeśli [x]2 − x[x] + 3 6 0, to x > 4.75.
(2) Jeśli [x]2 − x[x] + 4 6 0, to x > 5.8. ([OM] St Petersburg 1998).

9.4.19. [x2 ] + [x8 ] > 2[x5 ], dla x ∈ R. ([NoO] s.46).

h√ i h√ i h√ i h√ i h√ i
9.4.20. a + b + a+b > 2a + 2b , dla a > 1, b > 1. ([MM] 69(3)(1996) 226).

q 
9.4.21. Niech an = (n + 1)2 + n2 .

(1) Istnieje nieskończenie wiele liczb naturalnych n takich, że an+1 − an > 1.


(2) Istnieje nieskończenie wiele liczb naturalnych n takich, że an+1 − an = 1.
([IMO] Longlist 1988).
126 Nierówności 9. Różne nierówności

oooooooooooooooooooooooooooooooooooooooooooooooooooooooooooooooooooooo
9.5 Nierówności z częścią ułamkową
oooooooooooooooooooooooooooooooooooooooooooooooooooooooooooooooooooooo
Przedstawiamy nierówności z częścią ułamkową liczby rzeczywistej, które pojawiły się
w książce [N10].

9.5.1. Dla dowolnej liczby naturalnej n zachodzą następujące nierówności.


√ 1
(1) {n 2} > √ , dla n ∈ N. Dla każdego ε > 0 istnieje liczba naturalna n taka, że
2n 2
√ 1
{n 2} < √ + ε.
2n 2
([IMO] Longlist 1979).
√ 1
(2) {n 3} > √ , dla n ∈ N. Nie istnieje żadna stała c > 1 taka, że
n 3
√ c
{n 3} > √
n 3

dla wszystkich n ∈ N. ([OM] Białoruś 2000).


√ 3
(3) {n 7} > √ , dla n ∈ N, ([OM] Bośnia Hercegowina 2006).
n 7
1 √ 1
(4) < {n 7} < 1 − , dla n ∈ N. ([OM] Białoruś 2005).
2n 6n
n√ o n√ o n√ o n2 − 1
(5) 1 + 2 + ··· + n2 6 , ([OM] Rosja 1999, [Kw] 2/2000 M1699).
2
n2 − n n√ o n√ o n√ o n2 − 1
(6) 6 1 + 2 + ··· + n2 6 , ([KoM] 2002, B3545).
2 2
        √
n n n n n
(7) − + − · · · − (−1) < 2n, ([Kw] 6/2002 M1821).
1 2 3 n

9.5.2. Dla dowolnych różnych liczb naturalnych n i m istnieje liczba rzeczywista a taka, że

1 2 1 2
6 {an} 6 oraz 6 {am} 6 .
3 3 3 3
([OM] Węgry-Izrael 1994, [Crux] 1998 s.206).

9.5.3. 4[x]{x} 6 x2 . ([OM] Moskwa 1995/1996).

9.5.4. Niech x > 0.


(1) Jeśli log[x] + log{x} > 3, to x > 1001.998.
(2) Jeśli log[x] + log{x} > 2, to x > 101.98. ([OM] St Petersburg 1998).
Nierówności 9. Różne nierówności 127

9.5.5. Niech p będzie liczbą pierwszą i s liczbą całkowitą taką, że 0 < s < p. Następujące dwa
warunki są równoważne.
(1) s ∤ p − 1.
(2) Istnieją dwie liczby naturalne m, n takie, że 0 < m < n < p oraz
   
sm sn s
+ < .
p p p
([OM] USA 2006).


9.5.6. Ciąg xn = 4{n 2} jest ograniczony i dla dowolnych n 6= m zachodzi nierówność
1
|xn − xm | > .
|n − m|
([WaJ] 257(78), [N10]).

oooooooooooooooooooooooooooooooooooooooooooooooooooooooooooooooooooooo
9.6 Nierówności potęgowe
oooooooooooooooooooooooooooooooooooooooooooooooooooooooooooooooooooooo
 b+1  b
a+1 a
9.6.1. > , dla a, b > 0. ([OM] Szwecja 1984).
b+1 b

9.6.2. aa bb cc + ab bc ca + ac ba cb 6 1, gdy a, b, c > 0 i a + b + c = 1. ([Crux] 1998 s.226 z.A198).

9.6.3. aa bb cc > (abc)(a+b+c)/3 , dla a, b, c > 0. ([OM] USA 1974, [Mon] 82(3)(1975) 218-221).

9.6.4. (a + b − c)a (b + c − a)b (c + a − b)c 6 aa bb cc dla boków trójkąta. ([Pkh] s.22).

 a+b+c
b c a a+b+c
9.6.5. a b c 6 , dla a, b, c > 0. ([Crux] 2002 s.57).
3
ab
a+b
9.6.6. > 1, dla a, b ∈ Z. ([OM] St Petersburg 1997).
a−b

9.6.7. xx1 2 + xx2 3 + · · · + xxn−1


n
+ xxn1 > 1, dla x1 , . . . , xn > 0, n > 2. ([Mon] 4/1980 S6).

9.6.8. Jeśli f : D → R+ jest funkcją rosnącą, to

f (a)2a f (b)2b f (c)2c > f (a)b+c f (b)a+c f (c)a+b ,

dla dowolnych a, b, c ∈ D. ([Bedn] 185).


128 Nierówności 9. Różne nierówności

oooooooooooooooooooooooooooooooooooooooooooooooooooooooooooooooooooooo
9.7 Nierówności z logarytmami
oooooooooooooooooooooooooooooooooooooooooooooooooooooooooooooooooooooo
√ a−b a+b
9.7.1. ab < < , dla a, b > 0, a 6= b. ([OM] ZSRR 1986, [MM] 78(5)(2005) 397).
ln a − ln b 2
Średnią logarytmiczną dodatnich liczb a < b nazywamy liczbę

a−b
L(a, b) = .
ln a − ln b
Z powyższego faktu wynika, że

G(a, b) < L(a, b) < A(a, b),


√ a+b
gdzie G(a, b) = ab jest średnią geometryczną i A(a, b) = jest srednią arytmetyczną
2
liczb 0 < a < b. Rozpatrzmy jeszcze liczby
2ab
H(a, b) = (średnia harmoniczna),
a+b
1 √
N (a, b) = (a + ab + b),
3
2(a2 + ab + b2 )
T (a, b) = .
3(a + b)
Można udowodnić:

9.7.2. H(a, b) 6 G(a, b) 6 L(a, b) 6 N (a, b) 6 A(a, b) 6 T (a, b), dla 0 < a < b.
([MM] 78(5)(2005) 397).

9.7.3. Dla danych liczb rzeczywistych 0 < a < b rozpatrzmy funkcję


Z b
xt+1 dx
a
f (t) = Z b
.
t
x dx
a

Jest to funkcja rosnąca. Zachodzą równości:

f (−3) = H(a, b), f (−3/2) = G(a, b), f (−1) = L(a, b),


f (−1/2) = N (a, b), f (0) = A(a, b), f (1) = T (a, b).
([MM] 78(5)(2005) 397).

9.7.4. p log p + q log q > − log 2, dla p > 0, q > 0 i p + q = 1. ([Br80] 68).
 
logb a logc b loga c 9
9.7.5. 2 + + > , dla a, b, c > 1.
a+b b+c a+c a+b+c
Nierówności 9. Różne nierówności 129

ab bc ca 3
9.7.6. log2 + log2 + log2 + > log(abc), dla a, b, c > 0. ([MOc] z.507).
c a b 4
3
9.7.7. log10 (n + 1) > + log10 n, dla n ∈ N. ([IMO] Longlist 1959-1966, [Djmp] s.37).
10n
 
3n 1 1 1
9.7.8. log10 (n!) > + + · · · + − 1 , dla n ∈ N. ([IMO] Longlist 1959-1966, [Djmp] s.37).
10 2 3 n

9.7.9. Niech (an ) będzie ciągiem arytmetycznym o wyrazach dodatnich i różnicy r > 0. Wtedy

1 an+1 1 1 1 1 1 an
ln < + + ··· + 6 + ln . ([Bedn] 186-7).
r a1 a1 a2 an a1 r a1

⋆ H. Chen, Means generated by an integral, [MM] 78(5)(2005) 397-399.


oooooooooooooooooooooooooooooooooooooooooooooooooooooooooooooooooooooo
9.8 Nierówności z sumą lub iloczynem cyfr
oooooooooooooooooooooooooooooooooooooooooooooooooooooooooooooooooooooo
Przez s(n) i p(n) oznaczamy odpowiednio sumę i iloczyn cyfr liczby naturalnej n. Przed-
stawiamy nierówności, którymi zajmowaliśmy się w [N-2].

9.8.1. s(n + m) 6 s(n) + s(m) dla n, m ∈ N.


([Kw] 5/1972 31, [GaT] s.236, [ME] 7(4)(2002) z.157, [N-2]).

9.8.2. s(nm) 6 s(n)s(m) dla m, n ∈ N. ([Kw] 5/1972 31, [GaT] s.236, [N-2]).

9.8.3. s(n2 ) 6 s(n)2 dla n ∈ N. ([OM] Rosja 1996, wynika z 9.8.2).

9.8.4. s(2n) 6 2s(n) 6 10s(2n) dla n ∈ N. ([OM] Irlandia 1996, [N-2]).

9.8.5. s(n(n − 1)) 6= s((n + 1)2 ). ([OM] St Petersburg 1992, [Fom] 3/92).

9.8.6. s(1981n ) > 19. ([Kw] 12/1981 26).

9.8.7. Jeśli 41 | n, to s(n) > 5. ([Miss] 1994(3) z.74).

9.8.8. s(2n+1 ) 6= s(2n ) dla n ∈ N. ([MM] Q679, [N-2]).

9.8.9. s(2n ) > log4 n − 1. ([S-kc]).

9.8.10. s(2n ) < 3n + 9. ([N-2]).

9.8.11. Istnieje nieskończenie wiele liczb naturalnych n takich, że

s(2n ) > s(2n+1 ).

([Kw] 2/1977 26, [N-2]).


130 Nierówności 9. Różne nierówności

9.8.12. Istnieje nieskończenie wiele liczb naturalnych n takich, że

s(3n ) > s(3n+1 ).

([OM] Rosja 1997, [Kw] 5/97 s.47, 60, [N-2]).

9.8.13. Istnieją liczby naturalne n takie, że

s(3n ) > s(3n+1 ).

Najmniejszą z nich jest n = 11.

9.8.14. s(9n ) 6= 9 dla n > 2. ([Mon] 10/2001 z.10758).

9.8.15. p(n) < n dla n > 10. ([Ismj]).

9.8.16. Jeśli n > 1060 , to s(n) · p(n) < n. ([MG] 491(1997) 263, [MG] 493(1998) 72-75).

oooooooooooooooooooooooooooooooooooooooooooooooooooooooooooooooooooooo
9.9 Nierówności z silniami
oooooooooooooooooooooooooooooooooooooooooooooooooooooooooooooooooooooo
Podane w tym podrozdziale nierówności pojawiły się w [N11]
 50
100 51 · 151
9.9.1. 80! > 10! · 10 , 100! 6 , 300! > 100300 .
4
([Dlt] 8/1994, [MG] 83(496)(1999) 111, [Mat] 2/1950 53, 3/1950 54).

 n  n
n n
9.9.2. > n! > dla n > 6. ([BaL] 469, [BoL] 62 s.56).
2 3

9.9.3. n! > 2n , dla n > 5.


 n
n+1
9.9.4. n! 6 . ([OM] Belgia 1985/1986, [MG] 83(496)(1999) 111, [N11]).
2

9.9.5. (2n)! < nn (n + 1)n , dla n > 2. ([Mon] 15(10)(1908) 186-187).

 n  n
3n + 1 n+1
9.9.6. (2n)! 6 . ([MG] 83(496)(1999) 111).
2 2

(m + n)!
9.9.7. 2n n! 6 6 (m2 + m)n , gdy n 6 m. ([A-P] 1996, [OMm] 1997/1998).
(n − m)!

(2n)! 4n
9.9.8. > . ([San2] 9, [Ko03] 112).
(n!)2 n+1

9.9.9. n3n < (3n)!. ([BoL] 63 s.56, [Ko03] 111).


Nierówności 9. Różne nierówności 131

(3n)!
9.9.10. 32n < < 33n , dla n > 2. Jest to szczególny przypadek nierówności:
(n!)3

(kn)!
k (k−1)n < < k kn ,
(n!)k

dla 1 6 k − 1 6 n. ([Cmj] 20(2)(1989) 171-173).


 n  n  n
5n + 1 3n + 1 n+1
9.9.11. (3n)! 6 . ([MG] 83(496)(1999) 111).
2 2 2
 n
n 2 (n + 1)(n + 2)
9.9.12. n 6 (n!) 6 , dla n > 4. ([OM] Irlandia 1995, [Pa97]).
6

q √
n
9.9.13. Ciąg an = n
(n + 1)! − n! jest nieograniczony. ([KoM] 2005 A365).

√ q
n
9.9.14. n! < n+1
(n + 1)!. ([Jedr] B.14).

 √ 2 q q
n
9.9.15. n! > n+1
(n + 1)! · n−1
(n − 1)!, dla n > 2. ([Kw] 3/1995 M1462).

 √ 2 1 1 1
6 n2 + n + , dla n > 2.
n
9.9.16. n! ([IMO] Longlist 1967, [Djmp] s.43(347)).
3 2 6
q
9.9.17. 3 n (n!)2 (n + 1) < (n + 1)(n + 2), dla n > 2. ([Ko03]).

√ q q
n
9.9.18. n! + n+2
(n + 2)! < 2 n+1 (n + 1)!, dla n > 1. ([Crux] 1992 s.18 z.1589).


n
9.9.19. n! > (n + 1)/e. ([Dlt] 6/1989).

 n  n
n n
9.9.20. e 6 n! 6 en . ([Nath] 304).
e e

9.9.21. (n!)! > [(n − 1)!]n! , dla n > 1 ([Mon] 72(8)(1965) E1816, rozw. 74(2)(1967) s.202).

9.9.22. (n!)! > n [(n − 1)!]n! , dla n > 3 ([IMO] Longlist 1969).

9.9.23. (mn)! > (m!)n (n!)m , dla m, n ∈ N. ([OM] Czechosłowacja 1990/1991, [Par] 1998(2)).

(m + n)! m! n!
9.9.24. < m · n , dla m, n ∈ N. ([Putn] 2004 B1, [N11]).
(m + n)m+n m n

9.9.25. 1 · 3 · 5 · 7 · . . . · (2n − 1) < nn , dla n > 2. ([OM] Holandia 1990, [Pa97], [N11]).

9.9.26. 1 · 3 · 5 · 7 · . . . · (2n − 1) < 2nn−1 , dla n > 2. ([Crux] 1994 s.10).


132 Nierówności 9. Różne nierówności

    
1 1 1
9.9.27. 1+ 2+ ··· n + 6 2n!, dla n ∈ N. ([OM] St Petersburg 1998).
n n n

9.9.28. Jeśli a1 , . . . , an ∈ N i b = a1 +···+an


n , to a1 ! · · · an ! > ([b]!)n .
([IMO] Longlist 1969, [A-P] 2002).

3 4 n+2 1
9.9.29. + + ··· + < , dla n ∈ N.
1! + 2! + 3! 2! + 3! + 4! n! + (n + 1)! + (n + 2)! 2
([OM] Ukraina 2005).

oooooooooooooooooooooooooooooooooooooooooooooooooooooooooooooooooooooo
9.10 Nierówności z symbolami Newtona
oooooooooooooooooooooooooooooooooooooooooooooooooooooooooooooooooooooo
Podane w tym podrozdziale nierówności pojawiły się w [N11]
!
2n
9.10.1. < 4n , dla n ∈ N. ([Szni] 80, [S59] 403).
n
!
2n
9.10.2. < 4n−1 dla n > 5 ([S68], [N11]).
n
!
2n √
9.10.3. 3n < 4n , dla n ∈ N. ([OM] Polska 1990).
n
!
2n 4n
9.10.4. > √ , dla n ∈ N. ([Trost] 67, [S59] 395, [Br80] 55, [N11]).
n 2 n
!
2n 22n
9.10.5. > . ([S68], [N11]).
n 2n
!
4n 2n 4n
9.10.6. √ < < √ , dla n > 1. ([Chan] Ch.7 sect.3).
2 n n 2n
!
4n 2n 4n
9.10.7. √ p < <p , dla n ∈ N. ([Mon] 1/1977 z.6019).
π 4 n2 + n/2 + 1/8 n (n + 1/4)π
! !
2n 2n
9.10.8. Jeśli 0 ¬ k 6 2n, to 6 . ([N11]).
k n

!
2n + 1
9.10.9. < 4n , dla n ∈ N. ([Br80] 54).
n
! ! !
2n + 1 2n + 1 n + 1 2n + 1
9.10.10. + >2 , dla 1 6 k 6 2n. ([OM] Niemcy 1995).
k−1 k+1 n+2 k
Nierówności 9. Różne nierówności 133

v ! v ! v ! v !
u u u u
u n u n u n u n √
9.10.11. 1 · t +2· t +3· t + ... + n · t 6 2n−1 n3 , dla n > 2.
1 2 3 n
([OM] Hiszpania 88, [Crux] 1990 s.72, [Pa97]).

v !
u  
u 2n + 1 1
9.10.12. t
n
>2 1+ √ . ([OM] St Petersburg 1996).
n
n
n+1
v ! v ! v !
u u u
u n u n u n
9.10.13. 2 t
n
> t
n
+t
n
, dla 1 6 k 6 n. ([Mon] E2933).
k k−1 k+1
!
nn n nn
9.10.14. 6 6 , dla n, k ∈ N. ([A-P] 2000).
(n + 1)k k (n − k)n−k k k k (n − k)n−k

!
n
9.10.15 (E. F. Eclund, R. B. Eggleton). > nπ(k) , dla k > 202, n > 2k.
k
([Mon] 79(10)(1972) s.1083).

2n
X 2n
k k
9.10.16. (−1) > 0, dla n ∈ N, x ∈ R, x > 0. ([Crux] 2000 s.356 z.A236).
k=0
x+k

! !
n n
9.10.17. 2 > 3 , dla n > 3. ([Mon] 9(1980) 719-727).
3 2
!
b
9.10.18. Niech a, b będą liczbami naturalnymi takimi, że > ab. Istnieje wtedy liczba
a
naturalna c0 taka, że ! !
c
c b
b > ,
a a
dla c > c0 . ([Mon] 9(1980) 719-727).

oooooooooooooooooooooooooooooooooooooooooooooooooooooooooooooooooooooo
9.11 Nierówności z nwd i nww
oooooooooooooooooooooooooooooooooooooooooooooooooooooooooooooooooooooo
Przedstawiamy nierówności, którymi zajmowaliśmy się w [N-6].

9.11.1. a + b 6 (a, b) + [a, b], dla a, b ∈ N. ([PaT2], [N-6].).

9.11.2. (a, b) + (a, b + c) 6 a + c, dla a, b, c ∈ N. ([OM] St Petersburg 2005).


134 Nierówności 9. Różne nierówności

9.11.3. Niech a, b, c ∈ N, a | c, b | c, ab < c. Wtedy


 
c c
, > 1.
a b
([MOc] 2004 z.305).

a+1 b+1
9.11.4. Niech a, b ∈ N. Jeśli liczba + jest całkowita, to
b a

(a, b) 6 a + b.

([OM] Rosja 1994, [OM] Hiszpania 1996, [Pa97]).

a+1 b+1 c+1


9.11.5. Niech a, b, c ∈ N. Jeśli + + ∈ N, to
b c a

(a, b, c) 6 ab + bc + ca.
3

([MOc] 2005 z.406).

9.11.6. [1, 2, . . . , n] > 2n−1 . ([Mon] 89(1982) 12-129, [Zw] 1993).

9.11.7. [1, 2, . . . , n] 6 3n . (D. Hanson [CanB] 15(1972) 33-37).

9.11.8. [a, b] · [b, c] · [c, a] > [a, b, c]2 . ([OM] Rosja 1994, [Pa97]).

2ab
9.11.9. [a, b] + [a + 1, b + 1] > √ , dla a, b ∈ N, a > b. ([OM] St Petersburg, [Kw] 5/2001 s.56).
a−b

9.11.10. [a, b] + [a + 1, b + 1] > 2b a, dla a, b ∈ N, a > b. ([OM] St Petersburg).

9.11.11. Jeśli a1 < a2 < · · · < an są liczbami naturalnymi, to

[a1 , a2 , . . . , an ] > na1 .

([Kw] 10/1981 34).

9.11.12. Jeśli a < b < c < d < e są liczbami naturalnymi, to


1 1 1 1 15
+ + + 6 . ([OM] Kanada 1979).
[a, b] [b, c] [c, d] [d, e] 16

9.11.13. Jeśli a1 < a2 < · · · < an są liczbami naturalnymi, to


1 1 1 2n − 1
+ + ··· + 6 .
[a1 , a2 ] [a2 , a3 ] [an−1 , an ] 2n
([Kw] 8/1984 51, [AnAF] 168).
Nierówności 9. Różne nierówności 135

9.11.14 (Erdös). Niech a1 < a2 < · · · < an < 2n będzie skończonym ciągiem liczb natural-
nych, gdzie n > 6. Wtedy:
n o
(1) min [ai , aj ]; i, j = 1, 2, . . . , n 6 6([n/2] + 1).
n o
(2) max (ai , aj ); i, j = 1, 2, . . . , n > (38/147)n − c, gdzie c nie zależy od n.
([Mon] 44(6)(1937) z.3834-5, [Kw] 3/1979 36).

9.11.15. Niech n > a1 > a2 > · · · > ak oraz nwd(ai , aj ) 6 n dla wszystkich i, j. Wtedy

iai 6 n,

dla i = 1, 2, . . . , k. ([MOc] 2001).

9.11.16 (Erdös). Niech A(n, k) = nww(n, n − 1, n − 2, . . . , n − k + 1) dla n, k ∈ N, k 6 n.


(1) Liczba 14 jest największą liczbą naturalną n, dla której istnieje liczba naturalna k 6 n
taka, że A(n, 1) < A(n, 2) < A(n, 3) < · · · < A(n, k) = A(n, n).
(2) Niech f (n) oznacza największą liczbą naturalną k taką, że

A(n, 1) < A(n, 2) < A(n, 3) < · · · < A(n, k).



Wtedy f (n) < C n dla pewnej stałej C.
(3) lim f (n) = ∞. Dokładniej f (n) > k dla n > k! + k. ([Mon] 98(6)(1991) 555-556).
n→∞

9.11.17. Niech S będzie niepustym podzbiorem zbioru N zamkniętym ze względu na doda-


wanie. Następujące dwa warunki są równoważne.
(1) nwd(S) > 1.
(2) (a, b) > 1 dla wszystkich a, b ∈ S. ([Maza] s.13).

9.11.18. Jeśli a, b, c są liczbami naturalnymi takimi, że

(a + b) | ab, (b + c) | bc (c + a) | ca,

to nwd(a, b, c) > 2. ([Kw] 2/2005 14).

9.11.19. Niech a, b, c, d ∈ N. Jeśli a + b + c + d = 1995, to

(a, b, c, d) > 570.


19995
Równość zachodzi na przykład dla a = 285 = , b = c = d = 570. ([OM] Moskwa 1995/1996).
7

⋆ P. Bateman, J. Kalb, A. Stenger, A limit involving least common multiplies, [Mon] 109(2002)
393-394.
B. Farhi, Nontrivial lower bounds for the least common multiple of some finite sequences of integers,
[Jnum] 125(2007) 393-411.
D. Hanson, On the products of the primes, [CanB] 15(1972) 33-37.
M. Nair, On Chebyshev-type inequalities for primes, [Mon] 89(1982) 126-129.
136 Nierówności 9. Różne nierówności

oooooooooooooooooooooooooooooooooooooooooooooooooooooooooooooooooooooo
9.12 Nierówności z liczbami pierwszymi
oooooooooooooooooooooooooooooooooooooooooooooooooooooooooooooooooooooo
Przedstawiamy nierówności z liczbami pierwszymi, którymi zajmowaliśmy się w [N-4].
Przez π(x) oznaczamy liczbę liczb pierwszych nie większych od x.

π(n) 2
9.12.1. 6 dla n ∈ N. ([S50] 26).
n 3
9.12.2. Jeśli n jest liczbą pierwszą, to
π(n − 1) π(n)
< .
n−1 n
Jeśli n jest liczbą złożoną, to
π(n − 1) π(n)
> .
n−1 n
([S59] 367, [Mol2] 69).

π(n) 4 π(n)
9.12.3. < dla n ∈ N. Stąd wynika, że lim = 0. ([Mol2] 67).
n ln n n→∞ n
9.12.4. Jeśli n > 3 jest liczbą nieparzystą, to π(n) < ϕ(n). ([S59] 368).

9.12.5 (Leech 1957). Niech π1 (x), π3 (x) oznaczają liczby liczb pierwszych nie większych od
x postaci odpowiednio 4k + 1 i 4k + 3. Wtedy:
(1) π1 (x) < π3 (x) dla wszystkich x < 26861;
(2) π1 (26861) = 1473, π3 (26861) = 1472. ([Lion]).

9.12.6 (Twierdzenie Czebyszewa). Jeśli n > 1 jest liczbą naturalną, to istnieje liczba pierw-
sza p taka, że n < p < 2n.

9.12.7. Jeśli n > 5 jest liczbą naturalną, to pomiędzy n i 2n leżą co najmniej dwie liczby
pierwsze. ([S59] 400, [S68] 88, [S87] s.115).

9.12.8. Jeśli n > 2, to pomiędzy n i 32 n leży co najmniej jedna liczba pierwsza. ([Kw] 4/1998).

9.12.9. Jeśli n > 100, to pomiędzy n i 2n leży co najmniej 10 różnych liczb pierwszych.
([Kw] 4/1998).

9.12.10. Jeśli n > 4, to pomiędzy n i 2n leży co najmniej jedna liczba naturalna, która jest
iloczynem dwóch różnych liczb pierwszych. ([S64] 86, [DoC] 122).

9.12.11. Jeśli n > 15, to pomiędzy n i 2n leży co najmniej jedna liczba naturalna, która jest
iloczynem trzech parami różnych liczb pierwszych. ([S64] 86, [DoC] 123).

9.12.12. Dla każdej liczby naturalnej k istnieje liczba naturalna n taka, że pomiędzy n a 2n
leży co najmniej k liczb pierwszych. ([Mat] 4/1998 z.1422).
Nierówności 9. Różne nierówności 137

9.12.13. Dla dowolnej liczby naturalnej n iloczyn wszystkich liczb pierwszych, które spełniają
nierówność n < p < 2n, jest mniejszy od 4n . ([Zw] 1999, [N-4]).

9.12.14. Dla każdej liczby naturalnej n > 1 istnieje liczba pierwsza p taka, że n < p2 < 4n.
([Mat] 3/2002 z.1537).

9.12.15. Niech m ∈ N. Dla każdej liczby naturalnej n > 1 istnieje liczba pierwsza p taka,
że n < pm < 2m n. ([Mat] 3/2002 z.1537).

9.12.16. Istnieje taka liczba naturalna n, że w przedziale [n2 , (n + 1)2 ] jest co najmniej 1000
liczb pierwszych. ([MM] April 1992 s.130, [K-Me] z.217).

Przez pn oznaczamy n-tą liczbę pierwszą. Przykłady: p1 = 2, p2 = 3, p3 = 5, p4 = 7,


p5 = 11, p6 = 13, p7 = 17, p8 = 19, p9 = 23, p10 = 29.

9.12.17. Następujące dwa warunki są równoważne.


(1) pn+1 < 2pn , dla wszystkich n ∈ N.
(2) (Twierdzenie Czebyszewa) Dla każdej liczby naturalnej m > 1 istnieje liczba pierwsza p
taka, że m < p < 2m. ([N-4]).

9.12.18. pn+2 < 2pn , dla n > 3. ([S59] 401).

9.12.19. pn+2 < pn + pn+1 , dla n > 1. ([Trost] 73, [S59] 402).

9.12.20. pn+1 < p1 + p2 + · · · + pn , dla n > 2. ([S68] 95).

9.12.21. 2n < pn , dla n > 4.

9.12.22. 2n + 1 6 pn , dla wszystkich n ∈ N. Ponadto, 2n + 1 < pn , dla n > 6.

9.12.23. pn < 2n , dla n > 2.

n2 + 3n + 4
9.12.24. pn 6 . ([Mon] 79(1)(1972) E2274).
4
9.12.25. pn pm > pn+m . ([Trost] 73).

9.12.26. pn+2 < p1 p2 · · · pn − 1, dla n > 2 ([Wm] 7 32).

9.12.27. pn < p1 p2 · · · pn−1 + 1, dla n > 3. ([Mon] 96(4)(1989) 339-341).

9.12.28. p2n < p2 p3 · · · pn − 2, dla n > 3. ([Mon] 96(4)(1989) 339-341).

9.12.29. p3n−1 < p2 p3 · · · pn − 2, dla n > 4. ([Mon] 96(4)(1989) 339-341).

9.12.30. pn+1 + pn+2 6 p1 p2 · · · pn , dla n > 3. ([S64] 84).

9.12.31. p1 p2 · · · pn > p2n+1 , dla n > 4. ([RaT] 236, [N-4]).


138 Nierówności 9. Różne nierówności

9.12.32. Iloczyn wszystkich liczb pierwszych mniejszych lub równych n nie jest większy od
liczby 4n . ([Kw] 3/2003 s.44).

9.12.33. pn pn+1 pn+2 > p2n+3 , dla n > 3. Dla każdej liczby naturalnej k istnieje liczba
naturalna n(k) taka, że pn pn+1 · · · pn+k > pkn+k+1 dla wszystkich n > n(k). ([Cmj] 356).

9.12.34. Jeśli a > 0 jest liczbą rzeczywistą, to istnieje nieskończenie wiele liczb naturalnych
n takich, że
pn+1 < (1 + a)pn .
([Wino] 36, [Kw] 4/1998).

9.12.35. Dla dowolnych liczb naturalnych a, b istnieje liczba naturalna n0 taka, że

pn > an + b

dla wszystkich n > n0 . ([Kw] 6/87 38).

1 1 1 1 1
9.12.36. + 2 + ··· + 2 + < , dla n ∈ N. ([OM] Jugosławia 2001).
2
p1 p2 p n p 1 p 2 · · · pn 2

9.12.37. Dla każdej liczby naturalnej k istnieje n takie, że

pn+1 − pn > k.
n o n o
9.12.38. Zbiory n ∈ N; pn+1 − pn > 2 log pn oraz n ∈ N; pn+1 − pn < 2 log pn są
nieskończone. ([Mon] E2844).

9.12.39. Niech P (n) (dla n > 1) oznacza największą liczbę pierwszą dzielącą n. Istnieje
nieskończenie wiele liczb naturalnych n takich, że

P (n) < P (n + 1) < P (n + 2).

([OM] Bułgaria 1995).

9.12.40. Niech P (n) (dla n > 1) oznacza największą liczbę pierwszą dzielącą n. Jeśli n > 3,
to liczby P (n), P (n + 1) i P (n + 2) są parami różne. ([Gy04] 154).

oooooooooooooooooooooooooooooooooooooooooooooooooooooooooooooooooooooo
9.13 Nierówności z klasycznymi funkcjami arytmetycznymi
oooooooooooooooooooooooooooooooooooooooooooooooooooooooooooooooooooooo
Przedstawiamy nierówności, którymi zajmowaliśmy się w [N-5] i w których występują
następujące funkcje arytmetyczne: µ (funkcja Möbiusa), ϕ, τ oraz σ. Przypomnijmy, że
jeśli n jest liczbą naturalną, to τ (n) jest liczbą dzielników naturalnych liczby n oraz σ(n) jest
sumą dzielników naturalnych liczby n.

9.13.1. Jeśli nwd(a, b) > 1, to ϕ(ab) > ϕ(a)ϕ(b). ([S59] 207, [DoC] 367).


9.13.2. Jeśli n > 3 jest liczbą złożoną, to n − ϕ(n) > n. ([S59] 196, [San2] 89).
Nierówności 9. Różne nierówności 139


9.13.3 (A. M. Vaidya 1967). Jeśli n > 6, to ϕ(n) > n. ([MM] 45(4)(1972) 187-190, [San2] 89).

q q
9.13.4. ϕ(n) > n/2. Dokładniej, jeśli n parzyste, to ϕ(n) > n/2; jeśli n nieparzyste, to

ϕ(n) > n. ([Isaa] 356).

9.13.5. Istnieje nieskończenie wiele liczb naturalnych n takich, że ϕ(n) > ϕ(n + 1). ([Moll]).

9.13.6. Dla każdej liczby naturalnej m istnieje liczba naturalna n taka, że

ϕ(n) − ϕ(n − 1) > m oraz ϕ(n) − ϕ(n + 1) > m.

([S59] 207).

9.13.7.
(1) ϕ(n2 ) + ϕ(n2 + 2n + 1) < 2n2 , dla n > 2.
(2) ϕ(nk ) + ϕ((n + 1)k ) < knk , dla n > k > 2.
(3) ϕ(nk ) + ϕ((n + 1)k ) < 2n2 (n + 1)k−2 , dla n > 2, n > 2.
(4) ϕ(nm) + ϕ((n + 1)(m + 1)) < 2nm, dla n, m > 2. ([Mon] 79(8)(1972) E2316).

9.13.8. Jeśli (n − 1, n + 1) jest parą liczb pierwszych bliźniaczych, to 3ϕ(n) 6 n. ([San2] 88).

9.13.9 (D. J. Newman 1997). Niech a, b, c, d będą nieujemnymi liczbami całkowitymi takimi,
że a > 0, c > 0 oraz ad−bc 6= 0. Istnieje wtedy liczba naturalna n taka, że zachodzi nierówność

ϕ(an + b) < ϕ(cn + d).

9.13.10 (G. Martin 1999). Najmniejsza liczba naturalna n taka, że ϕ(30n + 1) < ϕ(30n)
ma 1116 cyfr. ([Gy04] 140).

9.13.11. 2 6 τ (n) < n, dla n > 3.

3
9.13.12. τ (n) 6 n, dla n > 2. ([N-5]).
4

9.13.13. τ (n) 6 2 n. ([S50] 114, [N-5]).

9.13.14. τ (n) 6 3n i równość zachodzi tylko dla n = 12. ([San2] 85).

9.13.15. Jeśli n > 4 jest liczbą parzystą, to

τ (n2 + 1) < n.

([OM] St Petersburg 1998).

9.13.16. τ (2n − 1) > τ (n). ([San2] 85).


140 Nierówności 9. Różne nierówności

9.13.17. τ (ab) > τ (a) + τ (b) − 1, dla a, b ∈ N. ([OM] St Petersburg 1996).

9.13.18. Jeśli a > 1, n ∈ N i an + 1 jest liczbą pierwszą, to

τ (an − 1) > n.

([Balt] 1996).

9.13.19. Dla każdej liczby naturalnej n istnieje liczba pierwsza p taka, że τ (p − 1) > n.
([S68] 146).

9.13.20. Dla każdej liczby naturalnej n istnieje liczba pierwsza p taka, że

τ (p − 1) > n oraz τ (p + 1) > n.

([S64] 96).

9.13.21. Niech f będzie wielomianem o nieujemnych współczynnikach całkowitych. Dla każ-


dej liczby naturalnej n istnieje liczba naturalna k taka, że

τ (f (k)) > n.

([Mat] 3/94 179).

9.13.22. Jeśli a1 , a2 , . . . , a200 są parami różnymi liczbami naturalnymi, to

τ (a1 a2 · · · a200 ) > 19901.

([OM] St Petersburg 2000).

9.13.23. τ (2p1 p2 ···pn + 1) 6 4n , gdzie p1 , . . . , pn są parami różnymi liczbami pierwszymi


większymi od 3. ([IMO] Shortlist 2002).

9.13.24. ϕ(n) > τ (n), dla n > 30. ([S50] 143).

9.13.25. ϕ(n)τ (n) > n, dla n ∈ N. ([Mon] 74(2)(1967) E1962).

9.13.26. ϕ(n)τ (n)2 6 n2 , dla n ∈ N, n 6= 4. Równość zachodzi tylko dla n = 1, 2, 8, 12.


([Mon] 74(2)(1967) E1962).

9.13.27. τ (n) + ϕ(n) = n, gdy n = 6, 8, 9; τ (n) + ϕ(n) = n + 1, gdy n = 1, 4 lub n jest


liczbą pierwszą; w pozostałych przypadkach

τ (n) + ϕ(n) < n.

([Crux] 1994 s.51 z.1817).


9.13.28. Jeśli n jest liczbą złożoną, to σ(n) > n + n. ([Crux] 2000 s.280, [N-5]).

9.13.29. σ(n) < n n, dla n > 2. ([MM] 45(4)(1972) 187-190).
Nierówności 9. Różne nierówności 141

9.13.30. Jeśli p1 , p2 , . . . , pn są parami różnymi liczbami pierwszymi większymi od 3, to

σ (2p1 p2 ···pn + 1) > 4n .

([IMO] Shortlist 2002).

(n + 1)!
9.13.31. σ(n!) 6 . ([Crux] 1990 s.58 z.1399).
2
 
1 1
9.13.32. σ(n!) > n! 1 + + · · · + . ([Crux] 1990 s.58 z.1399).
2 n

9.13.33. n2 /2 < ϕ(n)σ(n) < n2 , dla n > 1. ([S59] 245, [Dlt] 5/83 3).

9.13.34. ϕ(n)σ(n) 6 n2 − 1, dla n > 1. Równość zachodzi dokładnie wtedy, gdy n jest liczbą
pierwszą. ([Crux] 2000 s.280).

9.13.35. ϕ(n)σ(n) > n2 − n ⇐⇒ n = pk , gdzie p ∈ P, k > 1. ([Crux] 2000 s.280).

9.13.36. σ(n) + ϕ(n) > 2n, dla n > 1. ([Kw] 6/86 36).

 
σ(n)
9.13.37. ϕ(n ) < n. ([Mon] 76(4)(1969) z.5591).
n

6 σ(n)ϕ(n)
9.13.38. 2
< < 1, dla n > 2. ([HW4]).
π n2

σ(n) 6
9.13.39 (U. Annapurna 1972). √ < 2 , dla n > 13. ([MM] 45(4)(1972) 187-190).
n n π

n+1
9.13.40. σ(n) 6 τ (n). ([Crux] 2000 s.139).
2
3n
9.13.41. σ(n) 6 τ (n). ([Mon] 73(1)(1966) E1749).
4

9.13.42. σ(n) > nτ (n). ([Mon] 71(6)(1964) E1625).
√ q
9.13.43. nτ (n) < σ(n) < n 2τ (n). ([OM] Białoruś 1999).

9.13.44 (A. Mąkowski). τ (n)2 ϕ(n) > σ(n), dla n > 1. ([MM] 37(4)(1964) 538).

9.13.45. σ(1) + σ(2) + · · · + σ(n) 6 n2 . ([AnAF] 104).

9.13.46. Liczba σ(n)σ(n + 1)σ(n + 2) jest parzysta. ([AuP] 1992).


142 Nierówności 9. Różne nierówności

9.13.47. Istnieje nieskończenie wiele liczb naturalnych n takich, że


σ(n + 1) < σ(n).
Takimi są na przykład wszystkie liczby postaci p−1, gdzie p > 3 jest liczbą pierwszą. ([S59] 241).

9.13.48. Istnieje nieskończenie wiele liczb naturalnych n takich, że


σ(n + 1) > σ(n).
Takimi są na przykład wszystkie liczby pierwsze. ([S59] 241).

9.13.49. Dla każdej liczby naturalnej k istnieje liczba naturalna n taka, że


σ(n) − σ(n + 1) > k.
([S50] 509, [N-5]).

σ(n) √
9.13.50. < n, dla n > 2. ([Mon] 74(9)(1967) E1888, [B-zm] 69).
n
9.13.51. Jeśli n ma co najwyżej 5 różnych dzielników pierwszych, to
σ(n) 77
< .
n 16
σ(m) 76
Istnieje liczba naturalna m taka, że > . ([IMO] Longlist 1971).
m 16
σ(n) σ(n)
9.13.52. < log2 n + 2, n < ln n + 1. ([Kw] 6/83 47).
n
σ(1) σ(2) σ(n)
9.13.53. + + ··· + 6 2n. ([AnAF] 157).
1 2 n
σ(n) X 1
9.13.54. = . ([S59] 227).
n k|n
k

9.13.55. Dla każdej liczby naturalnej m > 1 istnieje nieskończenie wiele liczb naturalnych n
takich, że
σ(n)
> m.
n
([Mat] 6/88 354).

9.13.56. Istnieje nieskończenie wiele liczb naturalnych n takich, że


σ(n) σ(k)
>
n k
dla wszystkich k = 1, 2, . . . , n − 1. Angielska nazwa takich liczb: superabundant numbers
(Erdös 1944). Takimi liczbami n są:
1, 2, 4, 6, 12, 24, 36, 48, 60, 120, 180, 240, 360, 720, 840, 1260, 1680.
([IMO] Shortlist 1983, [Djmp] 165(457), [Mon] 116(3)(2009) 273).
Nierówności 9. Różne nierówności 143

oooooooooooooooooooooooooooooooooooooooooooooooooooooooooooooooooooooo
9.14 Nierówności i ciągi rekurencyjne
oooooooooooooooooooooooooooooooooooooooooooooooooooooooooooooooooooooo
Przedstawiamy pewne nierówności, które pojawiły się w [N-7]. Przez un oznaczamy n-tą
liczbę Fibonacciego; u1 = u2 = 1 oraz un+2 = un+1 + un .

9.14.1. 3un−1 6 2un 6 4un−1 , dla n > 2.

9.14.2. 4un−1 6 3un 6 5un−1 , dla n > 3.

9.14.3. un > (3/2)n−1 , dla n > 2. ([N-7]).

9.14.4. un+5m > 10m un dla n > 2. ([Nar03] 24).

9.14.5. u2n+1 > u2n , dla n ∈ N. ([FQ] B760).

un+1 um+1
9.14.6. − 6 1 dla n, m > 2. ([KoM] 7/96).
un um
9.14.7. ([IMO] Shortlist 1992). Dla każdego x1 ∈ [0, 1) definiujemy ciąg (xn ) przyjmując:

 0,
 gdy xn = 0,
xn+1 =  
 1 1
 − , gdy xn 6= 0.
xn xn
Wówczas dla każdej liczby naturalnej n zachodzi nierówność:
u1 u2 un
x1 + x2 + · · · + xn < + + ··· + .
u2 u3 un+1

X un
9.14.8. < 2. ([Ssm] 103(2)(2003) z.4753).
n=1
(un+2 − 1)2

√ 1
9.14.9. Niech a1 = 1, a2 = 2 oraz an+2 = an+1 + an dla n > 1. Wtedy n an+1 > 1 + na .

n
([OM] St Petersburg 1992).

9.14.10. Niech a1 = 1, a2 = 2, an+2 = 3an+1 − an . Wówczas


a2n+1
an+2 + an > 2 +
an
dla n ∈ N. ([OM] Wietnam 1999).

9.14.11. Niech a1 = 1, an+1 − 2an = 2(n + 1). Wtedy an 6 2n+2 dla n ∈ N.


([OM] Czechy-Słowacja 1996/97).

1 a2
9.14.12. Niech a0 = , ak = ak−1 + k−1 dla k = 1, 2, . . . , n. Wtedy
2 n
1
1− < an < 1.
n
([OM] Finlandia 1980, [Pa97]).
144 Nierówności 9. Różne nierówności

oooooooooooooooooooooooooooooooooooooooooooooooooooooooooooooooooooooo
9.15 Nierówności z funkcjami trygonometrycznymi
oooooooooooooooooooooooooooooooooooooooooooooooooooooooooooooooooooooo
20 21
9.15.1. 60 < sin 20o < 60 . ([IMO] Shortlist 1979, [Djmp] 140(438)).

9.15.2. tg 34o > 2/3. ([Kw] 9/1978 32).

9.15.3 (Fejér-Jackson). sin x+ 12 sin 2x+· · ·+ n1 sin nx > 0 dla x ∈ (0, π), n ∈ N. ([Dlt] 8/1995).

9.15.4. | sin 1| + | sin 2| + · · · + | sin(3n − 1)| + | sin(3n)| > 58 n, dla n ∈ N. ([OM] ZSRR 1986).

9.15.5. cos x + 12 cos 2x + · · · + 1


n cos nx > −1 dla x ∈ (0, π), n ∈ N. ([Dlt] 8/1995).

9.15.6. Niech a, b, c ∈ R, a2 +b2 6= 0. Istnieje liczba rzeczywista x taka, że c = a sin x+b cos x
wtedy i tylko wtedy, gdy a2 + b2 > c2 . ([AFe] 13).

9.15.7. sin x + cos x > 1, dla 0 < x < π2 .


  
1 1
9.15.8. 1+ 1+ > 5, dla 0 < x < π2 . ([CieS] 1986, [N10]).
sin x cos x

Z następnymi nierównościami spotkamy się w [N15]. Przez a, b, c oznaczamy boki trój-


kąta, a przez α, β, γ oznaczamy kąty wewnętrzne przy wierzchołkach leżących naprzeciw
odpowiednio boków a, b, c.

9.15.9.

(1) sin α + sin β + sin γ 6 3 3/2.
(2) 2 < sin α + sin β + sin γ, jeśli trójkąt jest ostrokątny.

(3) sin α + sin β + sin γ 6 1 + 2, jeśli trójkąt jest rozwartokątny. ([Khr2], [Khr1]).

9.15.10.
α a β b γ c
(1) sin 6 , sin 6 , sin 6 , ([AFe] 87);
2 b+c 2 c+a 2 a+b
√ √
3 3 3 3
(2) − 6 sin(α − β) + sin(β − γ) + sin(γ − α) 6 , ([Crux] 2001 s.140);
8 8
α β γ 3
(3) sin2 + sin2 + sin2 > , ([AFe] 96);
2 2 2 4

(4) sin α sin β sin γ 6 3 3/8, ([Khr2], [Khr1]);
α β γ 1
(5) sin sin sin 6 . ([AFe] 96);
2 2 2 8
1 1 1
(6) 2 α + + > 12, ([OM] Korea Pd. 1994).
sin 2 2
sin 2β sin2 γ2
Nierówności 9. Różne nierówności 145

9.15.11.
3
(1) 1 < cos α + cos β + cos γ 6 , ([Crux] 2001 45-47).
2
α β γ 9
(2) cos2 + cos2 + cos2 6 , ([AFe] 96).
2 2 2 4
(3) cos α cos β cos γ 6 1/2, ([Khr2], [Khr1]).
(4) cos α cos β cos γ 6 1/8, ([OM] Czechosłowacja 1968, [Pa03] s.19).

α β γ 3 3
(5) cos cos cos 6 , ([AFe] 96).
2 2 2 8
1 1 8
(6) + > , ([OM] Macedonia 1999).
sin α sin β 3 + 2 cos γ

9.15.12. Jeśli trójkąt jest ostrokątny, to tg α tg β tg γ > 3 3. ([AFe] 94).

oooooooooooooooooooooooooooooooooooooooooooooooooooooooooooooooooooooo
9.16 Inne nierówności
oooooooooooooooooooooooooooooooooooooooooooooooooooooooooooooooooooooo
Poniższe nierówności znajdziemy w [N10].

9.16.1. eπ > π e . ([Uiuc] 2002, [MG] 87(509)(2003) s.306).

9.16.2. (3.14)π > π 3.14 . ([Uiuc] 2007).

9.16.3. Która z liczb (2.71)e i e2.71 jest większa ?

1 1 1 1
9.16.4. < 1 + + · · · + − log n − γ < , gdzie
2n + 2/5 2 n 2n + 1/3
 1 1 1 
γ = lim 1 + + + · · · + − ln n = 0, 5772156649 · · · .
2 3 n
([Mon] 99(7)(1992) E3432).

9.16.5. Ciąg (a1 , . . . , an ) jest arytmetyczny, a ciąg (b1 , . . . , bn ) jest geometryczny. Jeśli a1 =
b1 i an = bn , to a1 + · · · + an > b1 + · · · + bn . ([MOc] 2003 z.201).

9.16.6. Niech f : R → R będzie nieparzystą funkcją rosnącą. Wtedy

f (a)f (b) + f (b)f (c) + f (c)f (a) 6 0

dla wszystkich liczb rzeczywistych a, b, c takich, że a + b + c = 0. ([Kw] 1/1998 M1601).


146 Nierówności Literatura

Literatura
[A-P] Asian Pacific Mathematical Olympiad. h61, 74, 97, 117, 130, 132, 133i.
[AF00] T. Andreescu, Z. Feng, G. Lee Jr., Mathematical Olympiads 1999 − 2000. Problems and Solu-
tions From Around the World, The Mathematical Association of America, 2003. h80i.
[AFe] T. Andreescu, Z. Feng, 103 Trigonometry Problems. From the training of the USA IMO team,
Birkhäuser, Boston - Basel - Berlin, 2005. h144, 145i.
[AnAC] T. Andreescu, D. Andrica, I. Cucurezeanu, An Introduction to Diophantine Equations,
Birkhäuser, 2010. h68i.
[AnAF] T. Andreescu, D. Andrica, Z. Feng, 104 Number Theory Problems. From the training of the
USA IMO team, Birkhäuser, Boston - Basel - Berlin, 2007. h134, 141, 142i.
[AnC] T. Andreescu, V. Cirtoaje, G. Dospinescu, M. Lascu, Old and New Inequalities, GIL Publishing
House, 2004. h57, 61–63, 77, 81, 82, 85, 87, 98i.
[AnE] T. Andreescu, B. Enescu, Mathematical Olympiad Treasures, Birkhäuser, Boston - Basel -
Berlin, 2006. h13, 15i.
[AuP] Austriacko-Polskie Zawody Matematyczne. h58, 62, 78, 141i.
[B-mv] P. S. Bullen, D. S. Mitrinović, P. M. Vasić, Means and Their Inequalities, Reider, Dordrecht,
1988. h13, 22, 24, 41i.
[B-rs] J. Browkin, J. Rempała, S. Straszewicz, 25 lat Olimpiady Matematycznej, WSiP, Warszawa,
1975. h66, 108, 120i.
[B-zm] V. I. Bernik, I. K. Żuk, O. W. Melnikow, Zbiór Zadań Olimpijskich z Matematyki (po rosyjsku),
Narodnaja Aswieta, Minsk, 1980. h120, 124, 125, 142i.
[BaL] I. W. Baranowa, C. E. Lapin, Zadania z Algebry (po rosyjsku), Leningrad, 1954. h106, 130i.
[Balk] Balkan Mathematical Olympiad. h79, 93i.
[Balt] Zawody Matematyczne Państw Bałtyckich. h52, 81, 87, 106, 124, 140i.
[BeB] E. F. Beckenbach, R. Bellman, Inequalities, Springer-Verlag 1961 (tłumaczenie rosyjskie 1965);
fourth ed., Springer-Verlag, Berlin, 1983. h16, 29i.
[Bedn] W. Bednarek, Zbiór Zadań dla Uczniów Lubiących Matematykę, Gdańskie Wydawnictwo
Oświatowe, Gdańsk, 1995. h66, 96, 104, 107, 120, 127, 129i.
[Berk] V. I. Bernik, Byelorussian Mathematical Olympiads, 1992-1993, Minsk, 1993. h54, 60i.
[BiYel] X. Bin, L. P. Yee, Mathematical Olympiad in China, Problems and Solutions, East China
Normal University Press, 2007awa, 2007. h i.
[BoL] W. G. Bołtiański, W. G. Leman, Zbiór Zadań Moskiewskich Olimpiad Matematycznych (po
rosyjsku), Moskwa, 1965. h107, 130i.
[BoW] W. G. Bołtiański, I. J. Wilenkij, Symetria w Algebrze (po rosyjsku), Nauka, Moskwa, 1967.
h76, 82i.
[Br80] J. Browkin, Zadania z Olimpiad Matematycznych, tom 5, 21-25, 69/70 - 73/74, WSiP, War-
szawa, 1980. h87, 108, 119, 128, 132i.
[Br83] J. Browkin, Zbiór Zadań z Olimpiad Matematycznych, tom 6, 26-30, 74/75 - 78/79, WSiP,
Warszawa, 1983. h53, 108i.
[Brad] C. J. Bradley, Introduction to Inequalities, The United Kingdom Mathematics Trust, Handbo-
oks 2, 2006. h76i.
Nierówności Literatura 147

[Bryn] M. Bryński, Olimpiady Matematyczne, tom 7, 31-35, 79/80 - 83/84, WSiP, Warszawa, 1995.
h96, 103, 119, 120i.
[Bull] P. S. Bullen, A Dictionary of Inequalities, Pitnam Monographs and Surveys in Pure and Ap-
plied Mathematics 97, 1998; (Supplement 2004). h31i.
[Buln] P. S. Bullen, Handbook of Means and Their Inequalities, Kluwer Academic Publishers, 2003.
h6, 9, 10, 13, 22, 24, 41i.
[CanB] Canadian Mathematical Bulletin, (Canad. Math. Bull.), kanadyjskie czasopismo matematycz-
ne. h134, 135i.
[Chan] K. Chandrasekharan, Introduction to Analityc Number Theory. h132i.
[ChKh] Le Hai Chau, Le Hai Khoi, Selected Problems of the Vietnamese Masthematical Olympiad
(1962 − 2009), Mathematical Olympiad Series 5, World Scietific 2010. h84, 88i.
[CiCP] D. Ciesielska, K. Ciesielski, Z. Pogoda, Epsilon, Wydawnictwo Szkolne Omega, Kraków, 2002.
h i.
[CieS] K. Ciesielski, J. Szczepański, Egzaminy Wstępne z Matematyki na Uniwersytet Jagielloński
1986−2003, Zadania i Odpowiedzi, Wydawnictwo Uniwersytetu Jagiellońskiego, Kraków, 2004.
h51, 144i.
[Cmj] The College Mathematics Journal, The Mathematical Association of America. h48, 75, 88, 93,
98, 120, 123, 131, 138i.
[Crux] Crux Mathematicorum, Canadian Mathematical Society, popolarne matematyczne czasopismo
kanadyjskie. h1, 16, 21, 48–53, 55, 57, 62, 63, 67, 71, 74–76, 78, 81, 83–85, 90, 95, 97–101, 105,
107, 109–113, 115, 116, 118, 122, 124, 126, 127, 131, 133, 140, 141, 144, 145i.
[Djmp] D. Djukić, V. Janković, I. Matić, N. Petrović, The IMO Compendium. A Collection of Pro-
blems Suggested for the International Mathematical Olympiads: 1959-2004, Problem Books in
Mathematics, Springer, 2006. h20, 21, 48, 58, 59, 64, 65, 74, 76, 81, 85, 91, 93, 96, 103, 123,
129, 131, 142, 144i.
[Dlt] Delta, popularny polski miesięcznik matematyczno-fizyczno-astronomiczny. h1, 18, 22, 31, 46,
53, 57, 67, 71, 78, 85, 87, 107, 121, 122, 125, 130, 131, 141, 144i.
[DoC] S. Doduniekow, K. Czakyrjan, Zadania z Teorii Liczb (po rosyjsku), Narodna Poswieta, Sofia,
1985. h125, 136, 138i.
[Druz] L. M. Drużkowski, Analiza Matematyczna. Podstawy, Wydawnictwo Uniwersytetu Jagielloń-
skiego, Kraków, 1998. h13i.
[DwJ] S. W. Dworianinow, E. A. Jasinowyj, Jak otrzymuje się nierówności symetryczne, (po rosyjsku),
Kwant, 7(1985), 33-36. h39i.
[Fich] G. M. Fichtenholz, Rachunek Różniczkowy i Całkowy, tom 1, PWN, Warszawa 1980. h9, 13,
18, 120i.
[Fom] D. V. Fomin, Sankt-Petersburskie Olimpiady Matematyczne (po rosyjsku), Politechnika, Sankt-
Petersburg, 1994. h28, 47, 94, 100, 106, 115, 119, 122, 129i.
[FQ] The Fibonacci Quarterly, czasopismo matematyczne. h143i.
[Fu47] L. Fuchs, A new proof of an inequality of Hardy - Littlewood - Polya, Mat. Tidsskr. B., 1947,
53-54. h15i.
[G-if] S. A. Genkin, I. W. Itenberg, D. V. Fomin, Leningradzkie Kółka Matematyczne (po rosyjsku),
Kirow, ASA, 1994. h106, 118, 119i.
148 Nierówności Literatura

[GaT] G. A. Galpierin, A. K. Tołpygo, Moskiewskie Olimpiady Matematyczne (po rosyjsku), 1935-


1985, Moskwa, 1986. h129i.
[Gorn] J. Górnicki, Okruchy Matematyki, PWN, Warszawa 1995. h10, 26, 46, 78i.
[Gy04] R. K. Guy, Unsolved Problems in Number Theory, Third edition, Springer-Verlag, New York,
2004. h138, 139i.
[H-48] G. H. Hardy, J. E. Littlewood, G. Polya, Nierówności, (po rosyjsku), Moskwa, 1948. h39i.
[H-52] G. H. Hardy, J. E. Littlewood, G. Polya, Inequalities, Cambridge University Press, 2nd edition,
1952. h11, 13, 22, 24, 29, 41i.
[H-lp] G. H. Hardy, J. E. Littlewood, G. Polya, Some simple inequalities satisfied by convex function,
Messenger Math., 58(1928/29), 145-152. h16i.
[HW4] G. H. Hardy, E. M. Wright, An Introduction to the Theory of Numbers, Fourth edition, Oxford
at the Clarendon Press, 1960. h141i.
[Ibe] Iberoamerican Mathematical Olympiad. h101i.
[IMO] Międzynarodowa Olimpiada Matematyczna. h10, 20–22, 48, 52, 57–59, 62–65, 67, 74, 76, 78,
81, 85, 87, 88, 90–94, 96, 103, 107–109, 111, 112, 123, 125, 126, 129, 131, 132, 140–144i.
[Isaa] I. M. Isaacs, Algebra, A Graduate Course, Brooks/Cole Publishing Company, Pacific Grove,
California, 1994. h139i.
[Ismj] Indiana School Mathematics Journal. h31, 130i.
[Jedr] P. Jędrzejewicz, Bukiety Matematyczne dla Gimnazjum, Gdańskie Wydawnictwo Oświatowe,
2002. h131i.
[JeL] L. Jeśmanowicz, J. Łoś, Zbiór Zadań z Algebry, (wydanie 2), PWN, Warszawa, 1965. h17, 25,
26i.
[Jlms] Journal of the London Mathematical Society, (J. London. Math. Soc.) h78i.
[Jnum] Journal of Number Theory, Academic Press, Orlando, (J. Number Theory). h135i.
[K-Me] J.-M. De Koninck, A. Mercier, 1001 Problems in Classical Number Theory, AMS, 2007. h137i.
[Kara] J. Karamata, Sur une inégalité relative aux fonctions convexes, Publ. Math. Univ. Belgrade,
1(1932), 145-148. h16i.
[Khr1] A. I. Khrabrov, Around mongolian inequality, (Russian), Matemat. Prosv., 3(7)(2003), 149-
162. h13–16, 49, 54, 63, 74, 81, 117, 144, 145i.
[Khr2] A. I. Khrabrov, Around mongolian inequality, (Russian), Appendix to: St Petersburg mathe-
matical olympiad, 2002. Nevsky Dialekt, St Petersburg, 2002, 146-167. h9, 13–16, 49, 74, 81,
117, 144, 145i.
[Khr3] A. I. Khrabrov, Cauchy–Bunyakovsky inequality, (Russian). Appendix to: St Petersburg ma-
thematical olympiad, 2003. Nevsky Dialekt, St Petersburg, 2003, 118-152. h29, 48, 77, 109i.
[Ko00] L. Kourliandtchik, Wędrówki po Krainie Nierówności, Aksjomat, Toruń, 2000. h22, 24, 26, 27,
29, 52, 54, 55, 57, 60, 66, 77, 79, 82, 85, 91, 110, 114i.
[Ko03] L. Kourliandtchik, Powrót do Krainy Nierówności, Aksjomat, Toruń, 2001. h24, 39, 46, 48, 60,
62, 64, 71, 75, 80, 85, 87, 94, 101, 102, 130, 131i.
[Ko04] L. Kourliandtchik, Słynne Nierówności, Aksjomat, Toruń, 2002. h50, 51, 75–78, 80, 82, 86–88i.
[KoM] KöMal, Kozepiskolai Matematikai Lapok, węgierskie czasopismo matematyczne, 1894-2012.
h65, 75, 85, 94, 97, 107, 110, 115, 120, 125, 126, 131, 143i.
Nierówności Literatura 149

[KoMe] J.-M. De Koninck, A. Mercier, 1001 Problémes en Théorie Classique des Nombres, Ellipses,
2004. h119, 125i.
[Kras] T. Krasiński, Analiza Matematyczna. Funkcje jednej zmiennej, Wydawnictwo UŁ, Łódź, 2003.
h11–13i.
[Kry1] W. Kryszewski, Wykład Analizy Matematycznej, Część I. Funkcje Jednej Zmiennej, Wydaw-
nictwo Naukowe UMK, Toruń, 2009. h13i.
[Kurs] J. Kürschak, Węgierskie Olimpiady Matematyczne (po rosyjsku), MIR, Moskwa, 1976. h95,
106i.
[Kurt] K. Kuratowski, Rachunek Różniczkowy i Całkowy, PWN, Warszawa, 1999. h13i.
[Kw] Kwant, popularne czasopismo rosyjskie. h1, 10, 29, 31, 47, 50, 51, 53–55, 57–61, 63–66, 71, 74,
77–79, 81, 85, 87, 91, 94–98, 100, 107–112, 118, 119, 123, 126, 129–131, 134–136, 138, 141, 142,
144, 145i.
[LeH2] H. Lee, Topics in Inequalities - Theorems and Techniques, Internet 2009. h10, 24, 29, 53, 54,
61, 65, 76, 91i.
[Lion] F. Le Lionnais, Les nombres remarquables, Herman, Paris, 1983. h136i.
[Liu1] A. Liu, Chinese Mathematics Competitions and Olympiads 1981−1993, Australian Mathematic
Trust Publications, 1998. h20, 75, 109i.
[Liu2] A. Liu, Chinese Mathematics Competitions and Olympiads 1993−2005, Australian Mathematic
Trust Publications, 2005. h109i.
[M-pf] D. S. Mitrinović, J. E. Pećarić, A. M. Finc, Classical and New Inequalities in Analysis, Kluwer
Academic, Dordrecht, 1993. h10, 22, 24, 26, 29, 77, 78i.
[MaO] A. W. Marshall, I. Olkin, Inequalities: Theory of Majorization and its Applications, Mathe-
matics in Science and Engineering vol. 143, 1979 (tłumaczenie rosyjskie 1983). h39, 41i.
[MaOD] R. B. Manfrino, J. A. G. Ortega, R. V. Delgado, Inequalities. A Mathematucal Olympiad
Approach, Birkhäuser, Boston - Basel - Berlin, 2009. h61, 64, 73, 74, 79, 82, 84, 91, 92, 96–98,
110, 113–115, 119, 123, 124i.
[MaS] Matematyka w Szkole, popularne czasopismo rosyjskie. h82, 84, 86, 89, 96, 113i.
[Mat] Matematyka, polskie czasopismo dla nauczycieli. h1, 48, 54, 57, 63, 74, 75, 79, 80, 93, 97, 101,
120, 122, 124, 125, 130, 136, 137, 140, 142i.
[Math] The Mathscope. All the best from Vietnamese Problem Solving Journals. http://
imocompendium.com/othercomp/Journ/mathscope.pdf. h54, 56, 57, 61–63, 65, 79, 87, 91i.
[Mau1] K. Maurin, Analiza, Część I, Elementy, PWN, Warszawa, 1973. h13i.
[Maza] W. Marzantowicz, P. Zarzycki, Elementarna Teoria Liczb, Wydawnictwo Naukowe PWN,
Warszawa, 2006. h135i.
[MC] Mathematics Competitions, popularne czasopismo matematyczne h51, 59–61, 76, 78, 94i.
[ME] Mathematical Excalibur, chińskie popularne czasopismo matematyczne, Hong Kong. h1, 41,
54, 59, 63, 90–93, 95, 97, 111, 114–117, 122, 129i.
[MG] The Mathematical Gazette, angielskie popularne czasopismo matematyczne. h1, 22, 31, 53, 62,
116, 130, 131, 145i.
[Mild] T. J. Mildorf, Olympiad inequalities, August 4, 2006, http://web.mit.edu/tmildorf/www.
h21, 28, 29, 53, 55, 57, 58, 62–65, 75, 81, 85, 89–92, 94, 100, 109, 111, 114, 115, 117i.
150 Nierówności Literatura

[Miss] Missouri Journal of Mathematical Sciences. h103, 129i.


[Mit2] D. S. Mitrinović, Elementarne Nierówności, PWN, Warszawa, 1972. h10, 13, 21–24, 26, 29, 31,
60, 78, 86i.
[Mitr] D. S. Mitrinović, Elementary Inequalities, P. Noordhoff LTD - Groningen, The Netherlands,
1964. h10, 13, 24, 26, 29, 78i.
[MiV] D. S. Mitrinović, P. M. Vasić, Analytic Inequalities, Springer-Verlag, 1970. h9–11, 13, 24, 26,
29, 77, 78i.
[MM] Mathematics Magazine, popularne czasopismo matematyczne. h21, 22, 58, 59, 65, 74, 77, 101,
108, 110, 125, 128, 129, 137, 139–141i.
[MOc] Mathematical Olympiads’ Correspondence Program, Canada, 1997-2012. h47, 58, 61, 68, 80,
81, 98, 100, 110, 112, 118, 129, 134, 135, 145i.
[Mock] Mock Putnam Exam. h59, 119, 120i.
[Mol2] R. A. Mollin, Fundamental Number Theory with Applications, Second Edition, CRC Press,
Boca Raton, London, New York, 2008. h136i.
[Moll] R. A. Mollin, Fundamental Number Theory with Applications, CRC Press, Boca Raton, London,
New York, 2000. h139i.
[Mon] The American Mathematical Monthly, Mathematical Association of America. h1, 10, 65, 67,
77, 78, 104, 111, 120, 122, 123, 127, 130–135, 137–142, 145i.
[MoP] E. A. Morozowa, I. S. Pietrakow, Międzynarodowe Olimpiady Matematyczne (po rosyjsku),
Moskwa 1971. h64i.
[Msn] Matematyka Społeczeństwo Nauczanie, Ośrodek Kultury Matematycznej w Mordach. h13i.
[N-1] A. Nowicki, Liczby Wymierne, Podróże po Imperium Liczb, cz.1, Wydawnictwo OWSIiZ, To-
ruń, Olsztyn. Wydanie pierwsze 2008; Wydanie drugie 2012. h106i.
[N-2] A. Nowicki, Cyfry Liczb Naturalnych, Podróże po Imperium Liczb, cz.2, Wydawnictwo
OWSIiZ, Toruń, Olsztyn. Wydanie pierwsze 2008; Wydanie drugie 2012. h3, 129, 130i.
[N-4] A. Nowicki, Liczby Pierwsze, Podróże po Imperium Liczb, cz.4, Wydawnictwo OWSIiZ, Toruń,
Olsztyn. Wydanie pierwsze 2009; Wydanie drugie 2012. h136, 137i.
[N-5] A. Nowicki, Funkcje Arytmetyczne, Podróże po Imperium Liczb, cz.5, Wydawnictwo OWSIiZ,
Toruń, Olsztyn, 2009. h3, 138–140, 142i.
[N-6] A. Nowicki, Podzielność w Zbiorze Liczb Całkowitych, Podróże po Imperium Liczb, cz.6, Wy-
dawnictwo OWSIiZ, Toruń, Olsztyn, 2009. h3, 133i.
[N-7] A. Nowicki, Ciągi Rekurencyjne, Podróże po Imperium Liczb, cz.7, Wydawnictwo OWSIiZ,
Toruń, Olsztyn, 2010. h3, 143i.
[N10] A. Nowicki, Liczby i Funkcje Rzeczywiste, Podróże po Imperium Liczb, cz.10, Wydawnictwo
OWSIiZ, Toruń, Olsztyn, 2010. h3, 124–127, 144, 145i.
[N11] A. Nowicki, Silnie i Symbole Newtona, Podróże po Imperium Liczb, cz.11, Wydawnictwo
OWSIiZ, Toruń, Olsztyn, 2011. h3, 130–132i.
[N15] A. Nowicki, Liczby, Funkcje, Ciągi, Zbiory, Geometria, Podróże po Imperium Liczb, cz.15,
Wydawnictwo OWSIiZ, Toruń, Olsztyn, 2011. h144i.
[Nar03] W. Narkiewicz, Teoria Liczb, PWN, Wydanie trzecie, Warszawa, 2003. h143i.
[Nath] M. B. Nathanson, Additive Number Theory, Graduate Texts in Mathematics 164, Springer,
1996. h131i.
Nierówności Literatura 151

[NoO] Nowe Olimpiady Matematyczne (po rosyjsku), Rostov-na-Donu, Feniks, 2005. h57, 125i.
[Nord] Nordic Mathematical Competition. h24, 74, 79, 84i.
[OM] Olimpiada Matematyczna. h20, 22, 24, 47–50, 52, 54, 55, 57–66, 68, 71, 74, 75, 78–88, 90–104,
106–110, 112–120, 123–135, 138–141, 143–145i.
[OMm] Mała Olimpiada Matematyczna. h19, 55, 88, 90, 91, 98, 101, 130i.
[Pa03] H. Pawłowski, Zadania z Olimpiad Matematycznych z Całego Świata. Trygonometria i Geo-
metria, Tutor, Toruń, 2003. h145i.
[Pa94] H. Pawłowski, Kółko Matematyczne dla Olimpijczyków, Turpress, Toruń, 1994. h125i.
[Pa97] H. Pawłowski, Zadania z Olimpiad Matematycznych z Całego Świata, Tutor, Toruń, 1997. h54,
63, 64, 79, 86, 87, 100, 131, 133, 134, 143i.
[Pach] B. G. Pachpatte, Mathematical Inequalities, North-Holland Mathematical Library, Elsevier,
2005. h11, 13, 31i.
[Par] Parabola, australijskie czasopismo matematyczne. h131i.
[PaT2] H. Pawłowski, W. Tomalczyk, Zadania z Matematyki dla Olimpijczyków, Index Books, Toruń,
1992. h133i.
[Pcam] Mathematical Proceedings of the Cambridge Philosophical Society, (Proc. Cambridge Ph.
Soc.). h78i.
[Pie1] E. Piegat, Jeszcze 105 zadań Hugona Steinhausa, Oficyna Wydawnicza GiS, Wrocław 2000.
h120i.
[Pie2] E. Piegat, Zadania Hugona Steinhausa Znane i Nieznane, Opracował Edward Piegat, Oficyna
Wydawnicza GiS, Wrocław 2005. h82, 120i.
[Pkh] Pham Kim Hung, Secrets in Inequalities, Vol. 1. Basic Inequalities, GIL Publishing House,
Romania 2007. h55, 58, 61, 63, 64, 66, 79, 81–88, 90–93, 95–103, 109–118, 122, 123, 127i.
[Plos] A. Płoski, Wstęp do Analizy Matematycznej, Politechnika Świętokrzyska, Kielce, 1997. h13i.
[Pmgr] Praca magisterska, Uniwersytet Mikołaja Kopernika w Toruniu, Wydział Matematyki i Infor-
matyki. h41i.
[Putn] Putnam (William Lowell) Mathematical Competition. h1, 104, 107, 119, 120, 131i.
[RaT] H. Rademacher, O. Toeplitz, O Liczbach i Figurach, PWN, Warszawa, 1956. h137i.
[Rias] S. Riasat, Basics of Olympiad Inequalities, Preprint, 2008. h20, 80, 90, 112, 122i.
[RiM] R i M, rumuńskie czasopismo matematyczne. h53, 80–83i.
[S-kc] M. Saadatmanesh, R. E. Kennedy, C. Cooper, Super Niven numbers, preprint. h129i.
[S-kg] W. A. Sadowniczij, A. A. Grigorjan, S. W. Konjagin, Zadania Studenckich Olimpiad Matema-
tycznych (po rosyjsku), Moskwa, 1987. h53i.
[S50] W. Sierpiński, Teoria Liczb, Warszawa - Wrocław, 1950. h124, 125, 136, 139, 140, 142i.
[S59] W. Sierpiński, Teoria Liczb II, PWN, Warszawa, 1959. h132, 136–139, 141, 142i.
[S64] W. Sierpiński, 200 Zadań z Elementarnej Teorii Liczb, Biblioteczka Matematyczna 17, PZWS,
Warszawa, 1964. h136, 137, 140i.
[S68] W. Sierpiński, Arytmetyka Teoretyczna, (wydanie 4), Biblioteka Matematyczna 7, PWN, War-
szawa, 1968. h132, 136, 137, 140i.
152 Nierówności Literatura

[S87] W. Sierpiński, 250 Zadań z Elementarnej Teorii Liczb, (wydanie 2), Biblioteczka Matematyczna
17, PZWS, Warszawa, 1987. h136i.
[San2] D. A. Santos, Elementary Number Theory Notes, Preprint, Internet 2002. h67, 130, 138, 139i.
[Siw] I. H. Siwaszinskij, Nierówności w Zadaniach (po rosyjsku), Nauka, Moskwa, 1967. h59, 76, 81,
107, 119i.
[Ssm] School Science and Mathematics Journal, School Science and Mathematics Association. h91,
97, 143i.
[StaZ] E. Stachowski, A. Zalewska, Zbiór Zadań dla Nauczycieli Prowadzących Koła Matematyczne,
Instytut Kształcenia Nauczycieli, Warszawa, 1984. h54, 66i.
[Stee] J. M. Steele, The Cauchy–Schwarz Master Class. An Introduction to the Art of Mathematical
Inequalities, Cambridge University Press, 2004. h12, 18, 19, 24, 28–31, 46, 84, 112i.
[Str] S. Straszewicz, Zadania z Olimpiad Matematycznych, tom I, 1-5, 49/50 - 53/54, PZWS, War-
szawa, 1960. h48, 54i.
[Str1] S. Straszewicz, Zadania z Olimpiad Matematycznych, tom II, 6-10, 54/55 - 58/59, PZWS,
Warszawa, 1961. h66, 76, 78, 87i.
[Str67] S. Straszewicz, Zadania z Olimpiad Matematycznych, tom III, 11-15, 59/60 - 63/64, PZWS,
Warszawa, 1967. h54, 119, 120i.
[Str72] S. Straszewicz, Zadania z Olimpiad Matematycznych, tom IV, 16-20, 64/65 - 68/69, PZWS,
Warszawa, 1972. h66i.
[Sup] W. P. Suprun, Wybrane Trudniejsze Zadania z Matematyki (po rosyjsku), Minsk, 1998. h58i.
[Szn] L. B. Szneperman, Zbiór Zadań z Algebry i Teorii Liczb (po rosyjsku), Minsk, 1982. h106, 107i.
[Szni] L. Sznirelman, Liczby pierwsze, PWN, Warszawa, 1954. h132i.
[Trost] E. Trost, Primzahlen, Verlag Birkhauser, Basel - Stuttgard. Tłumaczenie rosyjskie, Moskwa
1959. h132, 137i.
[TT] Tournament of the Towns. h59, 78i.
[TTjs] Tournament of the Towns, Junior, Spring. h64i.
[Uiuc] UIUC Undergraduate Math Contest, University of Illinois at Urbana-Champaign. h75, 145i.
[WaJ] N. B. Wasilev, A. A. Jegorow, Zadania Olimpiad Matematycznych Związku Radzieckiego (po
rosyjsku), 1961-1987, Moskwa, Nauka, 1988. h53, 66, 67, 74, 78, 127i.
[Wino] I. Winogradow, Elementy Teorii Liczb, PWN, Warszawa, 1954. h138i.
[Wm] Wiadomości Matematyczne, Roczniki Polskiego Towarzystwa Matematycznego, 1956-2012.
h137i.
[WyKM] W. A. Wyszenskij, I. W. Kartaszow, W. I. Michaiłowskij, M. I. Jadrenko, Zbiór Zadań
Kijowskich Olimpiad Matematycznych (po rosyjsku), 1935-1983, Kijów, 1984. h51, 99, 103,
104i.
[Zw] Zwardoń, Obóz Naukowy Olimpiady Matematycznej. h49, 58, 64, 80, 92, 100, 113, 118, 134,
137i.

Uniwersytet Mikołaja Kopernika, Wydział Matematyki i Informatyki, Toruń


Olsztyńska Wyższa Szkoła Informatyki i Zarządzania, Olsztyn
e-mail: anow@mat.uni.torun.pl
Skorowidz
Aassila M., 97 Eclund E.F., 133
Aleksiejew P., 102 Eggleton R.B., 133
Andreescu T., 13, 146 Enescu B., 13, 146
Andrica D., 146 Erdös P., 135, 142
Annapurna U., 141
Arslanagić S., 57 Farhi B., 135
Ashiba I., 31 Fejér, 144
Feng Z., 146
Baranowa I.W., 146 Fichtenholz G.M., 147
Bateman P., 135 Finc A.M., 10, 24, 26, 29, 78, 149
Beckenbach E.F., 10, 16, 29, 146 Fomin D.V., 147
Bednarek W., 146 Fuchs L., 147
Bellman R., 16, 29, 146
Bencze M., 74 Galpierin G.A., 148
Berent A., 41 Genkin S.A., 147
Bernik V.I., 146 Godunova E., 77
Bin X., 146 Górnicki J., 10, 13, 26, 46, 78, 148
Boltianski W.G., 146 Grigorjan A.A., 151
Bos L., 48, 55 Grinberg D., 61, 83, 111
Bradley C.J., 146 Gueron S., 16
Browkin J., 146 Guy R.K., 148
Bryński M., 147
Hadamard J., 11
Bullen P.S., 13, 22, 24, 31, 41, 146, 147
Hajłasz P., 31
Hanson D., 134, 135
Carlitz L., 65
Hardy G.H., 11, 13, 16, 22, 24, 41, 148
Cauchy A., 26, 29
Hegedüs Y., 71
Chandrasekharan K., 147
Hin L.C., 41
Chau L.H., 147
Chen H., 129 Isaacs I.M., 148
Ciesielska D., 147 Itenberg I.W., 147
Ciesielski K., 147 Izboldin O.T., 10
Cirtoaje V., 55, 65, 81, 86, 87, 99–102, 112, 115,
146 Jackson T., 144
Cmyszlajew W.K., 29 Jadrenko M.I., 152
Cooper C., 151 Janković V., 147
Critch A., 68 Janous W., 49, 85, 97
Cucurezeanu I., 146 Jarski A., 71
Cynk S., 18 Jasinowyj E.A., 147
Czakyrjan K., 147 Jeśmanowicz L., 148
Jędrzejewicz P., 148
Delgado R.V., 149 Jegorow A.A., 152
Deykin L., 77 Jensen K.L., 8, 10
Diananda P.H., 78
Djekovicz D., 77 Kalb J., 135
Djukić D., 147 Karamata J., 16, 148
Doduniekow S., 147 Kartaszow I.W., 152
Dospinescu G., 98, 109, 146 Kedlaya K., 21
Drużkowski L.M., 147 Kennedy R.E., 151
Dworianinow S.W., 71, 147 Khoi L.H., 147

153
154 Nierówności Skorowidz nazwisk

Khrabrov A.I., 31, 148 Muirhead R.E., 33, 38


Kin-Yin Li, 95 Murty V.N., 52, 85, 116
Klamkin M.S., 57, 65, 113, 122 Mushkarov O., 115
Koninck De J.-M., 148, 149
Konjagin S.W., 151 Nair M., 135
Krasiński T., 149 Narkiewicz W., 150
Kryszewski W., 149 Nathanson M.B., 150
Kuczma M., 50, 76 Nesbitt A.M., 76–78
Kumor P., 51, 101 Newman D.J., 139
Kuratowski K., 149 Nguyen Anh Tuan, 114
Kurlandczyk L., 10, 22, 24, 26, 29, 51, 78, 102, Nguyen Van Thach, 84
122, 148 Nguyen Viet Anh, 79, 81, 85
Kurschak J., 149 Nicula V., 123
Nihei M., 31
Lapin C.E., 146 Nikolov N., 115
Lascu M., 146 Northover F.H., 78
Le Huu Dien Khue, 84 Novosad P., 77
Le Trung Kien, 113, 117 Nowicki A., 41, 150
Lee G., 146
Lee H., 10, 24, 31, 65, 76, 78, 81, 84, 85, 101, 149 Olkin I., 41, 149
Leech J., 136 Opial Z., 31
LeLionnais F., 149 Ortega J.A.G., 149
Leman W.G., 146
Pachpatte B.G., 11, 13, 31, 151
Levin V., 31, 77
Pawłowski H., 22, 151
Li K.Y., 41
Pećarić J.E., 10, 24, 26, 29, 78, 149
Lighthill M., 77
Perz G., 83
Littlewood J.E., 11, 13, 16, 22, 24, 41, 148
Petrović N., 147
Liu A., 149 Pham Kim Hung, 63, 66, 81–84, 86, 87, 93, 99,
Łoś J., 148 102, 103, 111–113, 116–118, 122, 151
Lord N., 22 Phan Hong Son, 116
Lore M., 92 Phan Thanh Nam, 110, 115
Łukaszewicz G., 31 Phan Thanh Viet, 116
Piegat E., 151
Mąkowski A., 141
Pietrakow I.S., 150
Malikic S., 93
Pinter L., 71
Manfrino R.B., 149 Pixton A., 90
Marghidanu D., 105 Płoski A., 151
Marshall A.W., 41, 149 Pogoda Z., 147
Martin G., 139 Polya G., 11, 13, 16, 22, 24, 41, 148
Marzantowicz W., 149 Pompe W., 50
Matić I., 147
Maurin K., 149 Rabinowicz W.L., 50
Melnikow O.W., 146 Rademacher H., 151
Mercier A., 148, 149 Rempała J., 146
Michaiłowskij W.I., 152 Riasa S., 100
Mildorf T.J., 114, 149 Riasat S., 112, 122, 151
Minkowski, 30 Rozenberg M., 63, 85
Mitev T., 21, 75
Mitrinović D.S., 10, 11, 13, 22–24, 26, 29, 41, 78, Saadatmanesh M., 151
146, 149, 150 Sadowniczij W.A., 151
Mollin R.A., 150 Santos D.A., 152
Morozowa E.A., 150 Sato N., 71, 95
Nierówności Skorowidz nazwisk 155

Sauvé L., 97
Scholze P., 83
Schur I., 16
Shapiro H.S., 77
Siebriuk M., 31
Siedrakjan N., 95
Sierpiński W., 22, 151, 152
Siwaszinskij I.H., 152
Stachowski E., 152
Steele J.M., 24, 29, 31, 46, 152
Steinhaus H., 151
Stellenbosch, 97
Stenger A., 135
Straszewicz S., 146, 152
Suprun W.P., 152
Szczepański J., 147
Szego G., 15
Szneperman L.B., 152
Sznirelman L., 152

Tessler R., 16
Tołpygo A.K., 148
Toeplitz O., 151
Tomalczyk W., 151
Trosh K., 77
Trost E., 152
Tsaoussoglou P.E., 63, 101

Vaidya A.M., 139


Vasić P.M., 11, 13, 22, 24, 26, 29, 41, 78, 146, 150
Vo Quoc Ba Can, 111

Wang E.T.H., 62
Wasilev N.B., 152
Wilenkij I.J., 146
Winogradow I., 152
Wright E.M., 53, 148
Wyszenskij W.A., 152

Yasinovyi E.A., 71
Yee L.P., 146
Young R.M., 22

Zalauf A., 77
Zalewska A., 152
Zarzycki P., 149
Żuk I.K., 146
Zvonaru T., 84, 101
Skorowidz
bezwzględna wartość, 10, 12, 27–29, 31, 67, 86, iloczyn cyfr, 129, 130
101, 123–127, 144 IMO, 1, 48, 62–65, 87, 92, 107–109, 111, 112
Longlist, 10, 20–22, 48, 52, 57, 58, 64, 76, 78,
całka, 11, 31, 128 81, 85, 93, 94, 125, 126, 129, 131, 132,
ciąg, 47 142
arytmetyczny, 67, 107, 120, 129, 145 Shortlist, 59, 64, 65, 67, 74, 87, 88, 90, 91, 93,
Fibonacciego, 143 96, 103, 123, 140–144
geometryczny, 145 indukcja matematyczna, 8, 17, 73
kolejnych liczb pierwszych, 137, 138 indukcja wsteczna, 18
nieograniczony, 131
nieskończony, 21, 23, 120 kolejne liczby naturalne, 106–108
ograniczony, 127
rekurencyjny, 106, 143 liczba
skończony, 13–15, 22–24, 30, 33, 35, 36, 39, γ, 145
41–46 π, 12, 109, 120, 132, 133, 141, 144, 145
wagowy, 43, 45, 48 e, 12, 13, 131, 145
cyfry, 108, 129, 130 dzielników naturalnych, 138
część całkowita, 2, 59, 124–126, 141, 143 Fibonacciego, 143
część ułamkowa, 126, 127 kwadratowa, 107, 131, 137–140, 143
nieparzysta, 136
dwumian Newtona, 25 pierwsza, 127, 136–140
postaci an + bn , 66
funkcja złożona, 140
ϕ, 2, 107, 136, 138–141 zespolona, 29, 44, 124
µ Möbiusa, 138 liczby względnie pierwsze, 107
π, 136 logarytm, 54, 120, 125, 126, 128, 129, 136, 138,
σ, 138, 140–142 142, 145
τ , 138–141
arytmetyczna, 138 macierz, 44
ciągła, 10, 11 Maple, 1
J-wklęsła, 10 max, 23, 24, 31, 49, 50, 53, 59, 122, 123
J-wypukła, 10 min, 23, 24, 31, 41, 59, 120, 122, 123
kosinus, 144, 145
liniowa, 6 największy dzielnik pierwszy, 138
malejąca, 69 nierówność
nieparzysta, 145 Bernoulliego, 25, 26
parzysta, 145 Cauchy’ego, 26, 27, 29, 55, 74, 76, 84, 88, 112,
przeciwna, 6 118
rosnąca, 7, 11, 12, 96, 127, 128 Cauchy’ego-Buniakowskiego, 26
różniczkowalna, 31, 121 Cauchy’ego-Buniakowskiego-Schwarza, 26
ściśle wklęsła, 6 Cauchy’ego-Schwarza, 26
ściśle wypukła, 6, 11, 12 cykliczna, 43, 46, 78, 115, 117, 120
sinus, 12, 144 Czebyszewa, 30
stała, 6, 7 dla boków trójkąta, 15, 64, 65, 100, 101, 117,
tangens, 144, 145 127, 144, 145
wklęsła, 6, 7, 10 dla liczb całkowitych, 67–71, 103–108, 119
wypukła, 6–15, 27, 41, 47, 53, 54, 64, 65, 81, dla liczb z odcinka, 63, 95, 100, 103, 110
113, 117 Höldera, 29, 30, 80
Hadamarda, 11
granica ciągu, 24, 135, 136, 145 Huyhensa, 20, 21

156
Nierówności Skorowidz 157

Janousa, 50 Kijów, 75
jednorodna, 47, 49–52, 57, 76, 79, 85, 111 Korea, 47, 144
Jensena, 8, 10, 27, 47 Litwa, 107
Karamaty, 13–16, 54, 64, 65, 81, 113, 117 Macedonia, 145
Minkowskiego, 30 Meksyk, 115
Nesbitta, 76, 77 Mołdawia, 22, 47, 60, 81–83, 86, 94, 102, 104,
Opiala, 31 107, 109
rearrangement, 30 Moskwa, 24, 62, 64, 71, 79, 83, 101, 124, 126,
Schura, 53, 55–57, 114 135
trójkąta, 27, 28 Niemcy, 132
z iloczynem równym jeden, 17, 19, 20, 63, 66, Norwegia, 58, 100
90, 92, 109, 116 Polska, 58, 60, 61, 66, 74, 98, 100, 113, 115,
z zerową sumą, 62, 65, 145 123, 124,
ze stałą sumą, 50, 61–63, 65, 66, 93, 94, 97, Rosja, 20, 55, 61, 68, 79, 83, 90, 92, 93, 96–98,
98, 102, 109, 115, 116, 118, 127 110,
ze stałą sumą kwadratów, 64, 94, 96, 103, 117, Rumunia, 63, 68, 91, 92, 100, 112, 117, 124
123 Słowenia, 117, 119
ze stałym iloczynem, 66, 89, 90, 92, 116, 117 Serbia-Czarnogóra, 62, 86, 91, 97, 113
nierówności równoważne, 18, 19 St Petersburg, 48, 49, 58, 61, 62, 66, 75, 83,
nwd, 2, 108, 133, 135, 138 96, 110,
nww, 2, 133–135 Szwecja, 62, 127
Turcja, 94
Olimpiada Matematyczna Ukraina, 64, 80, 85, 86, 91, 100, 103, 116, 132
Anglia, 62 USA, 57, 61, 66, 80, 83, 87, 123, 125, 127
Australia, 78 W.Brytania, 55, 62, 65, 93
Austria, 86, 96 Węgry, 64
Belgia, 130 Węgry-Izrael, 75, 95, 114, 126
Białoruś, 63, 80, 102, 126, 141 Wietnam, 64, 80, 84, 88, 100, 104, 110, 143
Bośnia Hercegowina, 82, 101, 115, 126 ZSRR, 60–62, 93, 96, 114, 128, 144
Brazylia, 101
Bułgaria, 91, 99, 138 permutacja, 31, 36, 37, 42, 43, 68–70, 95, 104
Chiny, 20, 59, 61, 62, 65, 75, 92, 109, 112, 116 pierścień wielomianów, 39, 61
Chorwacja, 87, 98, 110 pierwiastek z jedynki, 44
Czechosłowacja, 110, 131, 145 pochodna, 11, 12, 31, 121, 122
Czechy-Słowacja, 65, 82, 90, 91, 143 podzbiór, 5, 135
Estonia, 102, 110 podziały, 33, 34, 36–39, 41–43
Finlandia, 143 sąsiednie, 35, 36, 38
Grecja, 86, 96, 97, 108 podzielność, 129, 134, 135, 142
Hiszpania, 133, 134 postulat Bertranda, 136
Holandia, 131 potęga
Hong Kong, 63, 65, 68 dwójki, 18, 20, 67, 84, 92, 104, 119, 120, 129,
IMSA Intramural, 85 130, 132, 134, 137, 139–141, 143
Indie, 50, 61, 63, 82, 96, 97, 100, 102, 109 dziewiątki, 130
Iran, 48, 83, 92, 123 liczby pierwszej, 137, 141
Irlandia, 52, 61, 81, 95, 100, 102, 108, 113, trójki, 20, 104, 130, 134
129, 131 problem Janousa, 49, 51
Izrael, 106 problem Shapiro, 76–78
Japonia, 83, 98 przedział, 5, 6, 8–15, 53
Jugosławia, 138 (−∞, a), 5
Kanada, 54, 62, 71, 80, 98, 103, 134 (−∞, a], 5
Kanada-USA, 120 (0, 1), 63, 100, 110, 117
Kazachstan, 91, 98, 101 (0, 1], 100
158 Nierówności Skorowidz

(0, ∞), 12, 15, 47, 54, 64, 65, 81, 113, 117 wielomian, 28, 61, 140
(a, ∞), 5 symetryczny, 36, 37
(a, b), 5 wklęsłość w sensie Jensena, 10
(a, b], 5 wypukłość w sensie Jensena, 10
[0, 1], 11, 14, 59, 63, 100, 110, 117 wyznacznik, 9, 44, 45
[0, 2], 109
[a, ∞), 5, 115 zbiór
[a, b), 5 N0 , 1
[a, b], 5, 11, 31, 63, 74, 95 P(n, k), 33, 34, 41–46
pytanie, 95, 145 R+ , 5, 10, 55, 96, 121, 127
liczb całkowitych, 1, 96, 127
rearrangement inequality, 30 liczb naturalnych, 1, 67, 103, 119
relacja, 34, 44 liczb pierwszych, 1
częściowego porządku, 13, 44 liczb rzeczywistych, 1, 5, 6, 8–15, 23, 31, 39,
równanie diofantyczne, 108 47, 54, 64, 65, 81, 113, 117, 145
rozwinięcie dziesiętne, 108, 145 liczb wymiernych, 1, 10, 42–46
liczb zespolonych, 1, 29, 124
silnia, 36, 129–132, 135, 141 nieskończony, 125, 129, 130, 138, 139, 142
średnia, 110, 118, 128
arytmetyczna, 17, 20–23, 40, 46, 47, 68, 128
geometryczna, 10, 17, 20–23, 40, 46, 128
harmoniczna, 22, 23, 128
kwadratowa, 22, 23, 68
logarytmiczna, 128
potęgowa, 23, 24
złożona, 23
stopień wielomianu, 39
suma cyfr, 129, 130
superabundant numbers, 142
symbol Newtona, 25, 76, 132, 133
sześcian liczby całkowitej, 108
szereg, 28, 108, 119, 143

tożsamość Abela, 14
trójkąt, 15, 64, 65, 100–102, 117, 123, 144, 145
trójmian kwadratowy, 61
twierdzenie
Czebyszewa, 136, 137
Jensena, 8, 10
Karamaty, 14
Lagrange’a o wartości średniej, 11
Muirheada, 33, 38–43, 51–53, 55, 56
o nierówności cyklicznej, 46, 48, 53, 55–57,
79, 80, 87

ułamek prosty, 106–108, 132, 138, 141


układ Cramera, 45

warunek Lipschitza, 10
warunki równoważne, 6, 8, 9, 12, 22–26, 30, 38,
41, 42, 44, 45, 62, 65, 67, 127, 135, 137,
141, 144
wielokąt, 94

View publication stats

You might also like